/ Forside / Karriere / Uddannelse / Højere uddannelser / Nyhedsindlæg
Login
Glemt dit kodeord?
Brugernavn

Kodeord


Reklame
Top 10 brugere
Højere uddannelser
#NavnPoint
Nordsted1 1588
erling_l 1224
ans 1150
dova 895
gert_h 800
molokyle 661
creamygirl 610
berpox 610
jomfruane 570
10  3773 570
At se tilbage til universets begyndelse
Fra : Finn Guldmann


Dato : 22-10-04 03:39

Det har længe været mig til undren at man snakker om at man kan se så og
så mange mia. år tilbage i tiden. Og at man håber at kunne se helt,
eller næsten helt, tilbage til Big Bang (BB)

Under forudsætningerne at universet er skabt ved et BB og universet
udvider sig lige meget i alle retninger, vil det jo aldrig kunne lade
sig gøre at se længere tilbage i tiden end et vist tidspunkt.

Ved en BB-teori må der jo være et centrum hvor alting udvider sig fra.

Fra dette punkt vil alting, både lys og masse komme fra.

Men da lys formodes at bevæge sig hurtigere end massen vil det lys vi
skulle se som værende stammende fra BB jo forlængst være passeret det
punkt vi befinder os i, i forhold til BB-punktet.

Nu kender jeg ikke den hastighed hvormed universet udvider sig, nogen
hævder at den er stigende, men forudsat at den er mindre end lysets
hastighed vil der være lys vi aldrig vil kunne få at se.

Teorierne siger at universet er omkring 13 mia år gammel. Hvis vi viste
i hvilken retning "BB-punktet" var, og vi kunne se mere end 13 mia.
lysår tilbage, ville det vi fik øje på jo være masse (sole, planeter,
m.v.) der var på vej væk fra "BB-punktet" i modsat retning.

Lidt kringlet, med forhåbentlig så forståelig at det er til at regne ud
hvor jeg vil hen.

--
MVH Finn
To, på hinnanden følgende, undertekster i en udsendelse om
operasangere; "Jeg laver selv min mad" "og beder ofte til Gud"

 
 
Martin Andersen (22-10-2004)
Kommentar
Fra : Martin Andersen


Dato : 22-10-04 07:03

Finn Guldmann wrote:

>
> Ved en BB-teori må der jo være et centrum hvor alting udvider sig fra.

Forkert antagelse. BB skete overalt.

Det er teoretisk muligt at se tilbage til det tidspunkt hvor elektronerne
blev låst fast sammen med protonerne og neutronerne, og dannede brint og
helium.
Efter det skete så kunne fotonerne frit bevæge sig rundt og vi kan observere
dem.
Tidspunktet for dannelse af neutralt brint og helium er ca 300.000 år
efter BB.

Martin.

Martin Andersen (22-10-2004)
Kommentar
Fra : Martin Andersen


Dato : 22-10-04 07:15

Martin Andersen wrote:


> Tidspunktet for dannelse af neutralt brint og helium er ca 300.000 år
> efter BB.
>
> Martin.

Lige en rettelse: Det var neutralt brint der blev dannet ca 300.000 år
efter BB, og den stråling der så kom ud har vi observeret i 45 år.
Det er selvfølgelig den kosmiske baggrundsstråling, som er den
bedste indicium på at BB har fundet sted.

http://en.wikipedia.org/wiki/Cosmic_microwave_background_radiation


Martin.

Finn Guldmann (22-10-2004)
Kommentar
Fra : Finn Guldmann


Dato : 22-10-04 15:38

Martin Andersen wrote:

>> Ved en BB-teori må der jo være et centrum hvor alting udvider sig fra.
> Forkert antagelse. BB skete overalt.
>
Det passer ikke med det jeg har læst mig til. Iflg. den teori skulle BB
vare startet på den måde at det var et "uendelig" lille punkt hvor al
masse var samlet under et "uendelig" stort tryk.

Det må vel også være sådan at dit "over alt" må have en definerbar
størrelse i forhold til det nuværende univers?

> Det er teoretisk muligt at se tilbage til det tidspunkt hvor elektronerne
> blev låst fast sammen med protonerne og neutronerne, og dannede brint og
> helium.
> Efter det skete så kunne fotonerne frit bevæge sig rundt og vi kan
> observere
> dem.
> Tidspunktet for dannelse af neutralt brint og helium er ca 300.000 år
> efter BB.
>
Men hvor vil det lys komme fra?

Det blev udsendt for 13,63 mia år siden. Og medmindre universet udvider
sig med en hastighed der svarer til lysets hastighed er det jo forlængst
passeret ethvert punkt i universet.



--
MVH Finn
To, på hinnanden følgende, undertekster i en udsendelse om
operasangere; "Jeg laver selv min mad" "og beder ofte til Gud"

Martin Andersen (22-10-2004)
Kommentar
Fra : Martin Andersen


Dato : 22-10-04 18:42

Finn Guldmann wrote:
> Martin Andersen wrote:
>
>>> Ved en BB-teori må der jo være et centrum hvor alting udvider sig fra.
>>
>> Forkert antagelse. BB skete overalt.
>>
> Det passer ikke med det jeg har læst mig til. Iflg. den teori skulle BB
> vare startet på den måde at det var et "uendelig" lille punkt hvor al
> masse var samlet under et "uendelig" stort tryk.

Det passer også med det jeg skriver.
>
> Det må vel også være sådan at dit "over alt" må have en definerbar
> størrelse i forhold til det nuværende univers?
Nej, fordi det nuværende Univers blev skabt i BB. Der er ikke noget
udenfor Universet. Universet er alt. Ved tid 0 havde Universet
udstrækning 0.

> Men hvor vil det lys komme fra?
>
> Det blev udsendt for 13,63 mia år siden. Og medmindre universet udvider
> sig med en hastighed der svarer til lysets hastighed er det jo forlængst
> passeret ethvert punkt i universet.

Lyset, som nu er stærkt rødforskudt, kommer fra alle steder på himlen,
og har
været undervejs i ca 13 mia år. Universet behøver ikke at udvide sig med
større
end lysets hastighed, men bare være stort nok så strålingen først kommer
frem til
os nu. Hvis vi kører tiden bagover her på jorden, vil
baggrundsstrålingen stadig
være her, men komme fra områder der ligger tættere på.

Martin.

Finn Guldmann (22-10-2004)
Kommentar
Fra : Finn Guldmann


Dato : 22-10-04 20:24

Martin Andersen wrote:

>>>> Ved en BB-teori må der jo være et centrum hvor alting udvider sig fra.
>>> Forkert antagelse. BB skete overalt.
>> Det passer ikke med det jeg har læst mig til. Iflg. den teori skulle
>> BB vare startet på den måde at det var et "uendelig" lille punkt hvor
>> al masse var samlet under et "uendelig" stort tryk.
> Det passer også med det jeg skriver.
>
For den tid BB skete kan vi godt blive enige om det der var dengang var
alt. Men i udstrækning var det ikke meget i forhold til hvad der er i dag.

>> Det må vel også være sådan at dit "over alt" må have en definerbar
>> størrelse i forhold til det nuværende univers?
> Nej, fordi det nuværende Univers blev skabt i BB. Der er ikke noget
> udenfor Universet. Universet er alt. Ved tid 0 havde Universet
> udstrækning 0.
>
Hvilken udstrækning har så ved tid 13,7 mia år?

>> Men hvor vil det lys komme fra?
>> Det blev udsendt for 13,63 mia år siden. Og medmindre universet
>> udvider sig med en hastighed der svarer til lysets hastighed er det jo
>> forlængst passeret ethvert punkt i universet.
> Lyset, som nu er stærkt rødforskudt, kommer fra alle steder på himlen,
> og har været undervejs i ca 13 mia år. Universet behøver ikke at udvide
> sig med større end lysets hastighed, men bare være stort nok så
> strålingen først kommer frem til os nu. Hvis vi kører tiden bagover
> her på jorden, vil baggrundsstrålingen stadig være her, men komme fra
> områder der ligger tættere på.
>
Som det vistnok blev beskrevet tidligere fik udvidelsen et forspring på
300.000 år inden det første lys blev udsendt. Men siden har
udvidelseshastigheden da vist været væsentlig lavere end lysets
hastighed. Så hvordan kan universet havde udvidet sig så meget at fra
f.eks. år 1 mia. endnu ikke er kommet forbi os?

--
MVH Finn
To, på hinnanden følgende, undertekster i en udsendelse om
operasangere; "Jeg laver selv min mad" "og beder ofte til Gud"

Per Rønne (23-10-2004)
Kommentar
Fra : Per Rønne


Dato : 23-10-04 18:31

Finn Guldmann <n@a.invalid> wrote:

> Martin Andersen wrote:

> > Nej, fordi det nuværende Univers blev skabt i BB. Der er ikke noget
> > udenfor Universet. Universet er alt. Ved tid 0 havde Universet
> > udstrækning 0.

> Hvilken udstrækning har så ved tid 13,7 mia år?

Det aner man ikke; man kan jo kun se det »synlige« univers.

> >> Men hvor vil det lys komme fra?
> >> Det blev udsendt for 13,63 mia år siden. Og medmindre universet
> >> udvider sig med en hastighed der svarer til lysets hastighed er det jo
> >> forlængst passeret ethvert punkt i universet.

> > Lyset, som nu er stærkt rødforskudt, kommer fra alle steder på himlen,
> > og har været undervejs i ca 13 mia år. Universet behøver ikke at udvide
> > sig med større end lysets hastighed, men bare være stort nok så
> > strålingen først kommer frem til os nu. Hvis vi kører tiden bagover
> > her på jorden, vil baggrundsstrålingen stadig være her, men komme fra
> > områder der ligger tættere på.

> Som det vistnok blev beskrevet tidligere fik udvidelsen et forspring på
> 300.000 år inden det første lys blev udsendt. Men siden har
> udvidelseshastigheden da vist været væsentlig lavere end lysets
> hastighed. Så hvordan kan universet havde udvidet sig så meget at fra
> f.eks. år 1 mia. endnu ikke er kommet forbi os?

Det er jo ikke som sådan de fysiske objekter der udvider sig. Det er
selve rummet der ekspanderer. Og det kan ekspandere hurtigere end lysets
hastighed.
--
Per Erik Rønne

Finn Guldmann (25-10-2004)
Kommentar
Fra : Finn Guldmann


Dato : 25-10-04 07:02

Per Rønne wrote:

>>>Nej, fordi det nuværende Univers blev skabt i BB. Der er ikke noget
>>>udenfor Universet. Universet er alt. Ved tid 0 havde Universet
>>>udstrækning 0.
>>Hvilken udstrækning har så ved tid 13,7 mia år?
> Det aner man ikke; man kan jo kun se det »synlige« univers.
>
Det gør jo ikke tingene nemmere at forstå.

>>Som det vistnok blev beskrevet tidligere fik udvidelsen et forspring på
>>300.000 år inden det første lys blev udsendt. Men siden har
>>udvidelseshastigheden da vist været væsentlig lavere end lysets
>>hastighed. Så hvordan kan universet havde udvidet sig så meget at fra
>>f.eks. år 1 mia. endnu ikke er kommet forbi os?
> Det er jo ikke som sådan de fysiske objekter der udvider sig. Det er
> selve rummet der ekspanderer. Og det kan ekspandere hurtigere end lysets
> hastighed.
>
Den kan jeg godt følge. Objekt a og objekt b udvider sig ikke. Men
afstanden mellem dem gør.

Men for at lys fra objekt a, der er udsendt 700.000 "inde i" universets
eksistens, først når frem til os, på objekt b, 13 mia. år senere skal de
to objekter jo havde fjernet sig fra hinnanden med en hastighed der
ligget tæt på lysets.

Har man nogen mulighed, f.eks. ved hjælp af måling af Dopplereffekten,
at afgøre med hvilken hastighed det objekt (a) der har udsendt det lys
man hævder er 13 mia. år gammelt fjerner sig fra os med?

--
MVH Finn
To, på hinnanden følgende, undertekster i en udsendelse om
operasangere; "Jeg laver selv min mad" "og beder ofte til Gud"

Preben Riis Sørensen (25-10-2004)
Kommentar
Fra : Preben Riis Sørensen


Dato : 25-10-04 19:12


"Finn Guldmann" <n@a.invalid> skrev > Har man nogen mulighed, f.eks. ved
hjælp af måling af Dopplereffekten,
> at afgøre med hvilken hastighed det objekt (a) der har udsendt det lys
> man hævder er 13 mia. år gammelt fjerner sig fra os med?

Man måler rødforskydningen, den eneste målemetode for disse afstande. Og en
13 mia. ly fjern galakse er ca. dobbelt så langt væk når vi modtager dens
gamle lys, om teorierne holder desangående.....så nu er farten det dobbelte.
--
M.V.H.
Preben Riis Sørensen
preben@esenet.dk




Regnar Simonsen (25-10-2004)
Kommentar
Fra : Regnar Simonsen


Dato : 25-10-04 22:01

Preben Riis Sørensen
> Man måler rødforskydningen, den eneste målemetode for disse afstande. Og
en
> 13 mia. ly fjern galakse er ca. dobbelt så langt væk når vi modtager dens
> gamle lys, om teorierne holder desangående.....så nu er farten det
dobbelte

Afstandensvurderingen passer meget godt, men hvad er farten det dobbelte
af??
Jeg har prøvet at regne på en galakse med en rødforskydning på 100 - dvs.
afstandene i universet er øget med en faktor 100.
Universets alder : 13 milliarder år
Lysets rejsetid : 12,987 milliarder år
Hastigheden af galaksen i dag : 1,80·c (c=lysets hastighed i vacuum)
Hastigheden af galaksen dengang lyset blev udsendt : 18·c
Afstand til galaksen i dag : 35 milliarder lysår
Afstand til galaksen, da lyset blev udsendt : 0,35 milliarder lysår.

Galakser med en anden rødforskydning har selvfølgelig lidt andre værdier -
specielt hvis man udregner hastigheden "dengang".

--
Hilsen
Regnar Simonsen



Preben Riis Sørensen (26-10-2004)
Kommentar
Fra : Preben Riis Sørensen


Dato : 26-10-04 00:27


"Regnar Simonsen" <regnar.simo@image.dk> skrev >
> Afstandensvurderingen passer meget godt, men hvad er farten det dobbelte
> af??

Farten ved udsendelsestidspunktet.

Det er forresten ikke så ligetil at beregne. Afstanden mellem to galakser
stiger eksponentielt, for jo mere afstand, jo større hastighed fjerner de
sig fra hinanden med. Hvis der altså e r tale om bevægelse. Det var sjovt
om man kunne måle at udstrækningen på en af de fjerneste galakser rent
faktisk blev mindre. Det tror jeg dog ikke er tilfældet, for jeg tror heller
ikke på at der er tale om bevægelse.

--
M.V.H.
Preben Riis Sørensen
preben@esenet.dk




Henning Makholm (26-10-2004)
Kommentar
Fra : Henning Makholm


Dato : 26-10-04 01:47

Scripsit "Preben Riis Sørensen" <preben@esenet.dk>

> Det er forresten ikke så ligetil at beregne. Afstanden mellem to galakser
> stiger eksponentielt, for jo mere afstand, jo større hastighed fjerner de
> sig fra hinanden med.

Tja, tjo. Kun hvis man antager at Hubble-parameteren er konstant i tid.
Det gør man normalt ikke i BB-modeller.

--
Henning Makholm "... popping pussies into pies
Wouldn't do in my shop
just the thought of it's enough to make you sick
and I'm telling you them pussy cats is quick ..."

Regnar Simonsen (26-10-2004)
Kommentar
Fra : Regnar Simonsen


Dato : 26-10-04 12:50


Preben Riis Sørensen
> Farten ved udsendelsestidspunktet.
>
> Det er forresten ikke så ligetil at beregne. Afstanden mellem to galakser
> stiger eksponentielt, for jo mere afstand, jo større hastighed fjerner de
sig fra hinanden med

Som Henning påpeger, kan du ikke argumentere således, da Hubblekonstanten
ændrer sig med tiden - specielt i det tidlige univers, aftog den hurtigt med
tiden.
Dvs. for en given afstand var universets ekspansion meget større dengang end
nu.
Hvis man ved, hvorledes H ændrer sig med tiden, er udregningerne ikke så
svære - i en simpel model gælder fx :
Hastigheden for en galakse med rødforskydning z :
Nu : v = 2·(1 - (1+z)^-½)·c
Dengang : v = 2·((1+z)^½-1)·c

> > Afstandensvurderingen passer meget godt, men hvad er farten det dobbelte
af??
>
PRS :
> Farten ved udsendelsestidspunktet.

I min udregning var hastigheden af galaksen 10 gange større ved
udesendelsestidspunktet end den er i dag (for z=100)

--
Hilsen
Regnar Simonsen



Preben Riis Sørensen (26-10-2004)
Kommentar
Fra : Preben Riis Sørensen


Dato : 26-10-04 17:58


"Regnar Simonsen" <regnar.simo@image.dk> skrev> I min udregning var
hastigheden af galaksen 10 gange større ved
> udesendelsestidspunktet end den er i dag (for z=100)

Hvor har du så det fra?
--
M.V.H.
Preben Riis Sørensen
preben@esenet.dk




Regnar Simonsen (26-10-2004)
Kommentar
Fra : Regnar Simonsen


Dato : 26-10-04 20:11

Preben Riis Sørensen
> Hvor har du så det fra?

Man er selvfølgelig nødt til at gøre nogle antagelser :
Man skal kende den nuværende Hubblekonstant (ca. 20 km/s/Mlysår)
Man skal have en model for udviklingen - her bruges standardmodellen
Universet antages at være tæt på at være fladt

Universets alder = 2/3·Th
Th= Hubbletiden
Da universets alder er ca. 13 milliarder år, bliver Hubbletiden : Th = 19,5
milliarder år
(Hubbletiden er den tid lyset er om at bevæge sig en Hubblelængde, som igen
er den afstand hvor galakserne bevæger sig med lysets hastighed)
Hubblelængde : Rh = 19,5 milliarder lysår

Vi kan nu antage, at vi ser en galakse med flg. rødforskydning : z = 100
(det betyder, at universet har udvidet sig med en faktor 101)

Med disse antagelser, kan man nu udregne flg. forhold

Afstanden til galaksen i dag : Rnu = 2·(1-(1+z)^-½)·Rh = 35 milliarder
lysår
Afstande til galaksen dengang lyset blev udsent : Ro =
2/(1+z)·(1-(1+z)^-½)·Rh = 0,35 milliarder lysår

Som et tjek kan man udregne : Rnu / Ro = 1+z = 101

Galaksens hastighed er i dag : v(nu) = 2·(1-(1+z)^-½)·c = 1,80·c
Dette passer med, at galaksen ligger i en afstand på 1,80·Rh

Galaksens hastighed dengang lyset blev udsendt : vo = 2·((1+z)^½ -1)·c =
18·c
Dvs. at hastigheden er sænket med en faktor 10 (men bevæger sig dog stadig
over lysets hastighed)

Kilde : "Cosmic Horizons" - jeg kan ikke lige huske forfatter og forlag.

--
Hilsen
Regnar Simonsen



Preben Riis Sørensen (26-10-2004)
Kommentar
Fra : Preben Riis Sørensen


Dato : 26-10-04 20:37


"Regnar Simonsen" <regnar.simo@image.dk> skrev i en meddelelse
news:417ea155$0$18207$14726298@news.sunsite.dk...
> Preben Riis Sørensen
> > Hvor har du så det fra?
>
> Man er selvfølgelig nødt til at gøre nogle antagelser :
> Man skal kende den nuværende Hubblekonstant (ca. 20 km/s/Mlysår)
> Man skal have en model for udviklingen - her bruges standardmodellen
> Universet antages at være tæt på at være fladt
>
> Universets alder = 2/3·Th
> Th= Hubbletiden
> Da universets alder er ca. 13 milliarder år, bliver Hubbletiden : Th =
19,5
> milliarder år
> (Hubbletiden er den tid lyset er om at bevæge sig en Hubblelængde, som
igen
> er den afstand hvor galakserne bevæger sig med lysets hastighed)
> Hubblelængde : Rh = 19,5 milliarder lysår
>
> Vi kan nu antage, at vi ser en galakse med flg. rødforskydning : z = 100
> (det betyder, at universet har udvidet sig med en faktor 101)
>
> Med disse antagelser, kan man nu udregne flg. forhold
>
> Afstanden til galaksen i dag : Rnu = 2·(1-(1+z)^-½)·Rh = 35 milliarder
> lysår
> Afstande til galaksen dengang lyset blev udsent : Ro =
> 2/(1+z)·(1-(1+z)^-½)·Rh = 0,35 milliarder lysår
>
> Som et tjek kan man udregne : Rnu / Ro = 1+z = 101
>
> Galaksens hastighed er i dag : v(nu) = 2·(1-(1+z)^-½)·c = 1,80·c
> Dette passer med, at galaksen ligger i en afstand på 1,80·Rh
>
> Galaksens hastighed dengang lyset blev udsendt : vo = 2·((1+z)^½ -1)·c =
> 18·c
> Dvs. at hastigheden er sænket med en faktor 10 (men bevæger sig dog stadig
> over lysets hastighed)

Det stemmer da ikke meget med at man antager at Universet udvider sig
hurtigere og hurtigere.
--
M.V.H.
Preben Riis Sørensen
preben@esenet.dk




Regnar Simonsen (27-10-2004)
Kommentar
Fra : Regnar Simonsen


Dato : 27-10-04 19:06

Preben Riis Sørensen
> Det stemmer da ikke meget med at man antager at Universet udvider sig
hurtigere og hurtigere

Nej - men det gør det heller ikke hele tiden!

Se fx. flg. grafer :
http://map.gsfc.nasa.gov/m_uni/101bb2_1.html
Her ser man, at et univers med "mørk energi" i starten decelererer, for
siden at accelerere.
Hvis der er "mørk energi" skal de angivne formler modificeres - som Jonas
også påpeger i sit indlæg. Men da densiteten af "mørk energi" endnu er noget
ubestemt, valgte jeg i første omgang at se bort fra dette.
Hvis der er "mørk energi", vil universets alder være lidt større, end man
hidtil har antaget (kan aflæses på grafen, eller udregnes vha. Jonas´
formel)

--
Hilsen
Regnar Simonsen



Jonas Møller Larsen (26-10-2004)
Kommentar
Fra : Jonas Møller Larsen


Dato : 26-10-04 23:36

Regnar Simonsen wrote:
> Afstanden til galaksen i dag : Rnu = 2·(1-(1+z)^-½)·Rh = 35 milliarder
> lysår

Så vidt jeg kan se, antager du også, at vakuumenergien er nul, og regner
dermed på et fladt, stofdomineret univers. (Hvilket udmærket
illustrerer, hvilke størrelser der er i spil, men antagelsen synes ikke
at holde for det virkelige univers.)

Ifølge min lærebog er den mest generelle sammenhæng mellem afstand (R)
og rødforskydning (z)

dR = Rh · [(1-Omega)(1+z)² + OmegaV + OmegaM(1+z)³ + OmegaR(1+z)^4]^(-½)
· dz,

hvor
Rh: Hubble-afstanden (c/H0)
Omega: tæthedsparameteren
OmegaV: vakuumenergitæthed
OmegaM: stoftæthed
OmegaR: strålingstæthed

Det er jo grimt, men det observerede univers er tæt på fladt (Omega =
1), og strålingsenergien er lille (OmegaR = 0), så formelen reducerer til

dR = Rh · [OmegaV + OmegaM(1+z)³]^(-½)·dz. (*)

Sætter jeg her OmegaV = 0 (og OmegaM = 1) og integrerer fra 0 til z,
giver det faktisk formelen citeret øverst.

Men da OmegaV tilsyneladende ikke er nul i det aktuelle univers, kunne
det jo være praktisk, om man kunne integrere (*) analytisk. Det er nok
desværre umuligt.


Tilsvarende gælder universets alder = 2/3·Th (Th er Hubble-tiden, 1/H0)
også kun i et fladt univers med OmegaV = 0. For at beregne det aktuelle
univers' alder, skal vi integrere

dt = Th · [OmegaV(1+z)² + OmegaM(1+z)^5]^(-½)·dz,

fra 0 til uendelig, og det er jo også svært. Integralet her burde dog
give en værdi større end de 2/3·Th, for universets anslåede alder
voksede, da man tilsatte vakuumenergi til modellerne.

--
Jonas Møller Larsen

Regnar Simonsen (22-10-2004)
Kommentar
Fra : Regnar Simonsen


Dato : 22-10-04 07:23

Finn Guldmann
> Ved en BB-teori må der jo være et centrum hvor alting udvider sig fra

Nej, centrum er i alle punkter i universet

> Nu kender jeg ikke den hastighed hvormed universet udvider sig, nogen
> hævder at den er stigende, men forudsat at den er mindre end lysets
> hastighed vil der være lys vi aldrig vil kunne få at se.

I starten udvidede universet sig hurtigere end lysets hastighed i vacuum
(set fra en eller anden position); dermed bliver lysets hastighed også
forskellig afhængig af tidspunkt og afstand - lokalt er lyshastigheden dog
altid konstant (hvis man ser bort fra bevægelse gennem gravitationsfelter).

> Det har længe været mig til undren at man snakker om at man kan se så og
> så mange mia. år tilbage i tiden. Og at man håber at kunne se helt,
> eller næsten helt, tilbage til Big Bang (BB)

Med teleskoper kan man som Martin nævner, se tilbage til ca. 300000 år eBB.
Man håber dog engang at kunne bygge neutrinodetektorer, der vil kunne "se"
universet som det var ca. 1 sekund eBB. En 3. mulighed er måling af
gravitationsbølger.


--
Hilsen
Regnar Simonsen



Per A. Hansen (22-10-2004)
Kommentar
Fra : Per A. Hansen


Dato : 22-10-04 12:02


"Regnar Simonsen" <regnar.simo@image.dk> skrev i en meddelelse
news:4178a75f$0$77017$14726298@news.sunsite.dk...
> Finn Guldmann
>> Ved en BB-teori må der jo være et centrum hvor alting udvider sig fra
>
> Nej, centrum er i alle punkter i universet
>
>> Nu kender jeg ikke den hastighed hvormed universet udvider sig, nogen
>> hævder at den er stigende, men forudsat at den er mindre end lysets
>> hastighed vil der være lys vi aldrig vil kunne få at se.
>
> I starten udvidede universet sig hurtigere end lysets hastighed i vacuum
> (set fra en eller anden position); dermed bliver lysets hastighed også
> forskellig afhængig af tidspunkt og afstand - lokalt er lyshastigheden dog
> altid konstant (hvis man ser bort fra bevægelse gennem
> gravitationsfelter).

Ja, efter inflationsteorien.
Men den er noget omstridt og kan meget vel tænkes
at blive afløst, hvis man finder en bedre forklaring?

--
Med venlig hilsen
Per A. Hansen


Regnar Simonsen (22-10-2004)
Kommentar
Fra : Regnar Simonsen


Dato : 22-10-04 22:35

Per A. Hansen
> Ja, efter inflationsteorien.
> Men den er noget omstridt og kan meget vel tænkes
> at blive afløst, hvis man finder en bedre forklaring?

Også efter standard-big-bang; men det er selvfølgelig ekstra grelt, hvis der
også går inflation i sagerne

--
Hilsen
Regnar Simonsen



Finn Guldmann (22-10-2004)
Kommentar
Fra : Finn Guldmann


Dato : 22-10-04 16:42

Regnar Simonsen wrote:

>>Ved en BB-teori må der jo være et centrum hvor alting udvider sig fra
> Nej, centrum er i alle punkter i universet
>
Iflg. BB-teorien, som jeg har fundet den beskrevet, var universet samlet
i et punkt på størrelse med et atom lige inden BB.

Dette punkt kan ikke ses da det først er 300.000 år senere der findes
stof der kan afgive lys.

>>Nu kender jeg ikke den hastighed hvormed universet udvider sig, nogen
>>hævder at den er stigende, men forudsat at den er mindre end lysets
>>hastighed vil der være lys vi aldrig vil kunne få at se.
>
> I starten udvidede universet sig hurtigere end lysets hastighed i vacuum
> (set fra en eller anden position); dermed bliver lysets hastighed også
> forskellig afhængig af tidspunkt og afstand - lokalt er lyshastigheden dog
> altid konstant (hvis man ser bort fra bevægelse gennem gravitationsfelter).
>
Det tidsrum hvor udvidelseshastigheden var størrere end lysets varede jo
ikke længe.

>
>>Det har længe været mig til undren at man snakker om at man kan se så og
>>så mange mia. år tilbage i tiden. Og at man håber at kunne se helt,
>>eller næsten helt, tilbage til Big Bang (BB)
> Med teleskoper kan man som Martin nævner, se tilbage til ca. 300000 år eBB.
> Man håber dog engang at kunne bygge neutrinodetektorer, der vil kunne "se"
> universet som det var ca. 1 sekund eBB. En 3. mulighed er måling af
> gravitationsbølger.
>
Mit problem er hvor lyset har "gemt" sig i den tid der er gået fra det
blev udsendt fra et givent punkt i universet og til det når frem til os,
hvis det er udsendt inden et bestemt tidspunkt i universets udvikling.

Det er store tal, og med for mig for mange ubekendter. Men det må kunne
lade sig gøre at regne sig frem til et tidspunkt i universets udvikling
hvor al lys udsendt før vil have passeret os.

Logisk set må lys der er udsendt fra det, for os, fjerneste punkt i et
univers under stadig udvidelse havde passeret os hvis det er udsendt
inden et givent tidspunkt. Noget med at sammenregne universets alder,
udvidelseshastighed (afstand) med lysets hastighed.

--
MVH Finn
To, på hinnanden følgende, undertekster i en udsendelse om
operasangere; "Jeg laver selv min mad" "og beder ofte til Gud"

Carsten Svaneborg (22-10-2004)
Kommentar
Fra : Carsten Svaneborg


Dato : 22-10-04 19:07

Finn Guldmann wrote:
> Iflg. BB-teorien, som jeg har fundet den beskrevet, var universet samlet
> i et punkt på størrelse med et atom lige inden BB.

Så ville jeg foreslå at finde en anden beskrivelse! Det samme
gælder for enhver beskrivelse, der bruger "lige inden BB" et
sted i teksten.

Universet er uendeligt stort og har altid været det. Der findes
en karakteristisk skala mellem objekter i universet a, denne
udvikler sig med tiden, og "hastigheden" af skalaens vækst er
Hubble konstanten (der selv ændrer sig langsomt med tiden).

Hvis vi tager to vilkårlige punkter i rummet, og forestiller os at vi
skruer tiden tilbage, så vil disse to bevæge sig tættere og tættere på
hinanden fordi universet har ekspanderet siden Big Bang. Ved t~0 vil
begge punkter være uhyre tætte på hinanden. Da du kan gøre dette med
to vilkårlige punkter, følger det at Big Bang skete i ethvert punkt
i universet. Der findes altså ikke noget "centrum" for Big Bang.

> Dette punkt kan ikke ses da det først er 300.000 år senere der findes
> stof der kan afgive lys.

"punktet" er hele universet. Før rekombinationstiden var der massere af
fotoner, men universet var fyldt af et plasma, og derfor blev fotonerne
spredt, ligesom de bliver spredt i et glas mælk. Universet var derfor
uigennemsigtigt men fyldt med fotoner og elektroner og protoner.

> Mit problem er hvor lyset har "gemt" sig i den tid der er gået fra det
> blev udsendt fra et givent punkt i universet og til det når frem til os,
> hvis det er udsendt inden et bestemt tidspunkt i universets udvikling.

Hvor er brevet gemt, fra det blev lagt i postkassen, til at du modtager det?

> Det er store tal, og med for mig for mange ubekendter. Men det må kunne
> lade sig gøre at regne sig frem til et tidspunkt i universets udvikling
> hvor al lys udsendt før vil have passeret os.

Vi modtager idag den kosmiske baggrundstråling fra en kugleskal
med radius 13.7 G lysår borte - 300000 lysår. Fordi det har taget
lyset så lang tid at nå os. Hvis du som tid bruger tidspunktet for
hvornår vi MODTAGER lyset, og ikke hvornår det blev afsendt, så
sker Big Bang i en kugleskal, der bevæger sig væk fra os med lysets
hastighed.

--
Mvh. Carsten Svaneborg
http://www.softwarepatenter.dk

Finn Guldmann (22-10-2004)
Kommentar
Fra : Finn Guldmann


Dato : 22-10-04 22:39

Carsten Svaneborg wrote:

>>Iflg. BB-teorien, som jeg har fundet den beskrevet, var universet samlet
>>i et punkt på størrelse med et atom lige inden BB.
> Så ville jeg foreslå at finde en anden beskrivelse! Det samme
> gælder for enhver beskrivelse, der bruger "lige inden BB" et
> sted i teksten.
>
Jeg henviser blot til nogen jeg formoder ved mere om det end jeg. F.eks.
nogen der arbejder med det professionelt.

> Universet er uendeligt stort og har altid været det. Der findes
> en karakteristisk skala mellem objekter i universet a, denne
> udvikler sig med tiden, og "hastigheden" af skalaens vækst er
> Hubble konstanten (der selv ændrer sig langsomt med tiden).
>
> Hvis vi tager to vilkårlige punkter i rummet, og forestiller os at vi
> skruer tiden tilbage, så vil disse to bevæge sig tættere og tættere på
> hinanden fordi universet har ekspanderet siden Big Bang. Ved t~0 vil
> begge punkter være uhyre tætte på hinanden. Da du kan gøre dette med
> to vilkårlige punkter, følger det at Big Bang skete i ethvert punkt
> i universet. Der findes altså ikke noget "centrum" for Big Bang.
>
Men det der var "Overalt" dengang er jo forskellig fra det der er
"Overalt" nu. I rummelig udstrækning altså.

Og så er det jeg mener der må være en måde at beskrive hvor det ene
"Overalt" befinder sig i forhold til det andet.

>>Mit problem er hvor lyset har "gemt" sig i den tid der er gået fra det
>>blev udsendt fra et givent punkt i universet og til det når frem til os,
>>hvis det er udsendt inden et bestemt tidspunkt i universets udvikling.
> Hvor er brevet gemt, fra det blev lagt i postkassen, til at du modtager det?
>
Der kan der defineres en række af hændelser bestående af
postmedarbejdere og maskiner der fysisk flytter brevet.

Men kan der defineres en "vej" lyset har tilbagelagt?

>>Det er store tal, og med for mig for mange ubekendter. Men det må kunne
>>lade sig gøre at regne sig frem til et tidspunkt i universets udvikling
>>hvor al lys udsendt før vil have passeret os.
> Vi modtager idag den kosmiske baggrundstråling fra en kugleskal
> med radius 13.7 G lysår borte - 300000 lysår. Fordi det har taget
> lyset så lang tid at nå os. Hvis du som tid bruger tidspunktet for
> hvornår vi MODTAGER lyset, og ikke hvornår det blev afsendt, så
> sker Big Bang i en kugleskal, der bevæger sig væk fra os med lysets
> hastighed.
>
Fandt et sted (på dr.dk) hvor de viser et billede af en galakse hvor de
hævder at lyset har været 13 mia år undervejs (13 mia lysår gammelt).

Vi må vel selv befinde os et eller andet sted i kugleskallen? På vej væk
fra et eller andet sted hvor BB fandt sted for 13,7 mia år siden?

Men hvis universet er 13,7 mia år gammelt og man mener at der findes lys
der først når os nu, efter at det har været undervejs i 13 mia år, må
universet jo udvide sig med en hastighed der er mindst i omegnen af det
halve af lysets hastighed.

Og "kugleskallen" må have en radius på lidt over 6,5 mia lysår
(61.490.000.000.000.000.000.000 km)

--
MVH Finn
To, på hinnanden følgende, undertekster i en udsendelse om
operasangere; "Jeg laver selv min mad" "og beder ofte til Gud"

Regnar Simonsen (22-10-2004)
Kommentar
Fra : Regnar Simonsen


Dato : 22-10-04 23:15

Finn Guldmann
> Jeg henviser blot til nogen jeg formoder ved mere om det end jeg. F.eks.
> nogen der arbejder med det professionelt

Hvis du mener DR (eller Discovery, Illustreret Vidensskab), forsimpler de af
og til tingene for at gøre det mere forståeligt; derved kan noget gå tabt.

> Men det der var "Overalt" dengang er jo forskellig fra det der er
"Overalt" nu. I rummelig udstrækning altså.

Nej det er ikke korrekt.
Ved BB opstod rumtiden med indhold. Hvis universet er uendeligt nu, var det
også uendeligt en brøkdel af et sekund efter BB (i hvertfald inden for
"normale" modeller). Forskellen fra dengang til nu er at alle afstande er
blevet større (samtidig med at temperaturen er faldet).

> Men kan der defineres en "vej" lyset har tilbagelagt?
Ja det kan man godt, hvis man kender Hubble-konstanten (og hvordan den
varierer).

> Men hvis universet er 13,7 mia år gammelt og man mener at der findes lys
> der først når os nu, efter at det har været undervejs i 13 mia år, må
> universet jo udvide sig med en hastighed der er mindst i omegnen af det
> halve af lysets hastighed.

Man kan ikke sige at universet udvider sig med en bestemt hastighed. De nære
galakser bevæger sig næsten ikke i fht. os, mens de fjerneste man kan se
bevæger sig bort fra os med flere gange lysets hastighed; og bag denne
horisont findes der galakser, der bevæger sig bort fra os med flere hundrede
gange lysets hastighed. Lyset fra disse galakser vil ikke nå os før om flere
milliarder år, da det faktisk på nuværende tidspunkt er på vej væk fra os,
selv om det har retning mod os.

--
Hilsen
Regnar Simonsen



Finn Guldmann (23-10-2004)
Kommentar
Fra : Finn Guldmann


Dato : 23-10-04 00:30

Regnar Simonsen wrote:

>>Jeg henviser blot til nogen jeg formoder ved mere om det end jeg. F.eks.
>>nogen der arbejder med det professionelt
> Hvis du mener DR (eller Discovery, Illustreret Vidensskab), forsimpler de af
> og til tingene for at gøre det mere forståeligt; derved kan noget gå tabt.
>
Det afhænger vel af hvem målgruppen er?

For en som mig tror jeg nu nok det er bedre på den måde end på nogle af
de måder jeg ellers kan blive udsat for.

>>Men det der var "Overalt" dengang er jo forskellig fra det der er
> "Overalt" nu. I rummelig udstrækning altså.
>
> Nej det er ikke korrekt.
> Ved BB opstod rumtiden med indhold. Hvis universet er uendeligt nu, var det
> også uendeligt en brøkdel af et sekund efter BB (i hvertfald inden for
> "normale" modeller). Forskellen fra dengang til nu er at alle afstande er
> blevet større (samtidig med at temperaturen er faldet).
>
"Indholdet" har jo også ændret sig. De første 300.000 år var der end
ikke brintatomer. I dag er der eksistenser sammensat af så mange
forskellige stoffer (som ikke eksisterede dengang) at de, f.eks., udgør
levende væsner.

>>Men kan der defineres en "vej" lyset har tilbagelagt?
> Ja det kan man godt, hvis man kender Hubble-konstanten (og hvordan den
> varierer).
>
Det gør jeg så ikke. Og har heller ikke fundet den beskrevet på en måde
der ligger indenfor min fatteevne.

>>Men hvis universet er 13,7 mia år gammelt og man mener at der findes lys
>>der først når os nu, efter at det har været undervejs i 13 mia år, må
>>universet jo udvide sig med en hastighed der er mindst i omegnen af det
>>halve af lysets hastighed.
>
> Man kan ikke sige at universet udvider sig med en bestemt hastighed. De nære
> galakser bevæger sig næsten ikke i fht. os, mens de fjerneste man kan se
> bevæger sig bort fra os med flere gange lysets hastighed; og bag denne
> horisont findes der galakser, der bevæger sig bort fra os med flere hundrede
> gange lysets hastighed. Lyset fra disse galakser vil ikke nå os før om flere
> milliarder år, da det faktisk på nuværende tidspunkt er på vej væk fra os,
> selv om det har retning mod os.
>
Mener at vide at lysets hastighed er konstant. Hvilket betyder at lys
udsendt fra et objekt der bevæger sig med 98% af lysets hastighed stadig
kun vil bevæge sig med lysets hastighed. Hvilket igen betyder at en
beskuer der befinder sig på, og i samme hastighed som, objektet vil
opfatte lysets hastighed som kun værende 2% af hvad den er, for det lys
der bevæger sig i samme retning som objektet. Mens lys der bevæger sig i
den modsatte retning vil opfattes som havende en hastighed på 198% af
lysets.

Men det betyder så også at lys der udsendes fra en galakse der er på vej
væk fra os, med en hastighed mange gange lysets, stadig vil bevæge sig
mod os med lysets hastighed. Således at det ikke er galaksens hastighed
men dens placering i universet, på et givent tidspunkt, der har
betydning for hvornår lyset når frem til os.

Jeg kan så godt fatte at lys der er udsendt fra denne fjerne galakse med
et sekunds mellemrum vil ankomme til os med 100 sekunders mellemrum.

Men at jeg fatter det på den måde er jo ikke ensbetydende med at det
også er rigtig.

--
MVH Finn
To, på hinnanden følgende, undertekster i en udsendelse om
operasangere; "Jeg laver selv min mad" "og beder ofte til Gud"

Regnar Simonsen (23-10-2004)
Kommentar
Fra : Regnar Simonsen


Dato : 23-10-04 08:59

Finn Guldmann
> > Hvis du mener DR (eller Discovery, Illustreret Vidensskab), forsimpler
de af
> > og til tingene for at gøre det mere forståeligt; derved kan noget gå
tabt.
> >
> Det afhænger vel af hvem målgruppen er?

Njah - I medierne sammenligner man ofte med velkendte ting fra hverdagen;
derved kan man lettere forstå nogle af teoriens aspekter; men den simple
model er sjældent helt dækkende for de teorier, den skal illustrere.

> "Indholdet" har jo også ændret sig. De første 300.000 år var der end
> ikke brintatomer. I dag er der eksistenser sammensat af så mange
> forskellige stoffer (som ikke eksisterede dengang) at de, f.eks., udgør
> levende væsner

Korrekt - hydrogenatomer blev dannet ret sent, men komponenterne til
hydrogenatomerne (protoner og elektroner) har været til stede (næsten) siden
BB.

>> Hubblekonstanten
> Det gør jeg så ikke. Og har heller ikke fundet den beskrevet på en måde
> der ligger indenfor min fatteevne.

Hubblekonstanten fortæller hvor hurtigt universet udvider sig i en given
afstand. Galaksernes hastighed bort fra os (og fra hinanden) øges med
afstanden - hvis de er dobbelt så langt borte, bevæger de sig dobbelt så
hurtigt.
Hubblekonstanten kan fx. defineres ud fra den hastighed, galakser i en
afstand på 1000 lysår bevæger sig med. Ud fra denne konstant kan man så
udregne alle andre galaksers hastigheder.

v = H·R

v = hastighed
H = Hubblekonstanten
R = afstanden

> Mener at vide at lysets hastighed er konstant. Hvilket betyder at lys
> udsendt fra et objekt der bevæger sig med 98% af lysets hastighed stadig
> kun vil bevæge sig med lysets hastighed. Hvilket igen betyder at en
> beskuer der befinder sig på, og i samme hastighed som, objektet vil
> opfatte lysets hastighed som kun værende 2% af hvad den er, for det lys
> der bevæger sig i samme retning som objektet. Mens lys der bevæger sig i
> den modsatte retning vil opfattes som havende en hastighed på 198% af
> lysets.

Det er korrekt, at lyset lokalt har en konstant hastighed - og dermed vil
man heller aldrig opfatte hastigheder på 2% eller 198%.
Men hvis man taler om kosmologiske afstande, kan lysets hastighed godt være
anderledes, da lyset bevæger sig gennem et rum, der udvider sig.


--
Hilsen
Regnar Simonsen



Finn Guldmann (23-10-2004)
Kommentar
Fra : Finn Guldmann


Dato : 23-10-04 11:35

Regnar Simonsen wrote:

>>Det afhænger vel af hvem målgruppen er?
> Njah - I medierne sammenligner man ofte med velkendte ting fra hverdagen;
> derved kan man lettere forstå nogle af teoriens aspekter; men den simple
> model er sjældent helt dækkende for de teorier, den skal illustrere.
>
Men den simple form gør det lidt lettere at forstå for "almindelige"
mennesker.

>>>Hubblekonstanten
>>Det gør jeg så ikke. Og har heller ikke fundet den beskrevet på en måde
>>der ligger indenfor min fatteevne.
> Hubblekonstanten fortæller hvor hurtigt universet udvider sig i en given
> afstand. Galaksernes hastighed bort fra os (og fra hinanden) øges med
> afstanden - hvis de er dobbelt så langt borte, bevæger de sig dobbelt så
> hurtigt.
> Hubblekonstanten kan fx. defineres ud fra den hastighed, galakser i en
> afstand på 1000 lysår bevæger sig med. Ud fra denne konstant kan man så
> udregne alle andre galaksers hastigheder.
> v = H·R
> v = hastighed
> H = Hubblekonstanten
> R = afstanden
>
Jeg kan følge formlen. Men holder den så vand?

Men jeg undrer mig over at den skal kunne passe. For jeg får det til at
den beskriver en udvidelse der går fra der hvor vi befinder os i
universet. Men i og med at vi selv er en del af udvidelsen burde der jo
forekomme afvigelser.

>>Mener at vide at lysets hastighed er konstant. Hvilket betyder at lys
>>udsendt fra et objekt der bevæger sig med 98% af lysets hastighed stadig
>>kun vil bevæge sig med lysets hastighed. Hvilket igen betyder at en
>>beskuer der befinder sig på, og i samme hastighed som, objektet vil
>>opfatte lysets hastighed som kun værende 2% af hvad den er, for det lys
>>der bevæger sig i samme retning som objektet. Mens lys der bevæger sig i
>> den modsatte retning vil opfattes som havende en hastighed på 198% af
>>lysets.
> Det er korrekt, at lyset lokalt har en konstant hastighed - og dermed vil
> man heller aldrig opfatte hastigheder på 2% eller 198%.
> Men hvis man taler om kosmologiske afstande, kan lysets hastighed godt være
> anderledes, da lyset bevæger sig gennem et rum, der udvider sig.
>
Logisk set må det vel være sådan at lyset, hvis man bevæger sig med 98%
af lysets hastighed, "kun" vil forsvinde med 2% af lysets hastighed i
bevægelsesretningen? Mens man vil indhente tidligere udsendt lys hvis
man øger hastigheden til 102%?

--
MVH Finn
To, på hinnanden følgende, undertekster i en udsendelse om
operasangere; "Jeg laver selv min mad" "og beder ofte til Gud"

Martin Larsen (23-10-2004)
Kommentar
Fra : Martin Larsen


Dato : 23-10-04 13:13

"Finn Guldmann" <n@a.invalid> skrev i en meddelelse news:cldc7l$2geu$1@news.cybercity.dk...
> Regnar Simonsen wrote:
>
> >>Det afhænger vel af hvem målgruppen er?
> > Njah - I medierne sammenligner man ofte med velkendte ting fra hverdagen;
> > derved kan man lettere forstå nogle af teoriens aspekter; men den simple
> > model er sjældent helt dækkende for de teorier, den skal illustrere.
> >
> Men den simple form gør det lidt lettere at forstå for "almindelige"
> mennesker.
>
Det er vrøvl - men det kan give en svag ide om hvad sagen
drejer sig om for dem som slet ikke ved noget (hvem de så
end er). Og det er da fuldkommen idiotisk at tro at man kan
zoome ind på detaljerne i en populær fremstilling og mene
at der kunne ligge noget "dybt" gemt der.

Mvh
Martin



Finn Guldmann (25-10-2004)
Kommentar
Fra : Finn Guldmann


Dato : 25-10-04 06:23

Martin Larsen wrote:

>>Men den simple form gør det lidt lettere at forstå for "almindelige"
>>mennesker.
> Det er vrøvl - men det kan give en svag ide om hvad sagen
> drejer sig om for dem som slet ikke ved noget (hvem de så
> end er). Og det er da fuldkommen idiotisk at tro at man kan
> zoome ind på detaljerne i en populær fremstilling og mene
> at der kunne ligge noget "dybt" gemt der.
>
Det der tolker jeg som om at man skal "holde sin kæft og blande sig uden
om" hvis man ikke har samme dybe viden som dem der arbejder med det til
daglig, og evner at forstå fagjagonnen. Sådan håber jeg da ikke det er
ment.

--
MVH Finn
To, på hinnanden følgende, undertekster i en udsendelse om
operasangere; "Jeg laver selv min mad" "og beder ofte til Gud"

Martin Larsen (25-10-2004)
Kommentar
Fra : Martin Larsen


Dato : 25-10-04 12:15

"Finn Guldmann" <n@a.invalid> skrev i en meddelelse news:cli2mr$12o8$2@news.cybercity.dk...
> Martin Larsen wrote:
>
> >>Men den simple form gør det lidt lettere at forstå for "almindelige"
> >>mennesker.
> > Det er vrøvl - men det kan give en svag ide om hvad sagen
> > drejer sig om for dem som slet ikke ved noget (hvem de så
> > end er). Og det er da fuldkommen idiotisk at tro at man kan
> > zoome ind på detaljerne i en populær fremstilling og mene
> > at der kunne ligge noget "dybt" gemt der.
> >
> Det der tolker jeg som om at man skal "holde sin kæft og blande sig uden
> om" hvis man ikke har samme dybe viden som dem der arbejder med det til
> daglig, og evner at forstå fagjagonnen. Sådan håber jeg da ikke det er
> ment.
>
Det var en pudsig tolkning. Men ikke helt ved siden af.

Mvh
Martin



Regnar Simonsen (23-10-2004)
Kommentar
Fra : Regnar Simonsen


Dato : 23-10-04 23:04

Finn Guldmann
> Men den simple form gør det lidt lettere at forstå for "almindelige"
mennesker

OK - blot man er klar over, at analogier har sine begrænsninger.

> Jeg kan følge formlen. Men holder den så vand?
Ja, det mener man. Bemærk dog at Hubblekonstanten varierer med tiden.

> Men jeg undrer mig over at den skal kunne passe. For jeg får det til at
> den beskriver en udvidelse der går fra der hvor vi befinder os i
> universet. Men i og med at vi selv er en del af udvidelsen burde der jo
> forekomme afvigelser.

Nej; en iagttager i en vilkårlig anden galakse, vil se det samme - nemlig at
være centrum for universets udvidelse. Også her gælder Hubbles lov.

> Logisk set må det vel være sådan at lyset, hvis man bevæger sig med 98%
> af lysets hastighed, "kun" vil forsvinde med 2% af lysets hastighed i
> bevægelsesretningen? Mens man vil indhente tidligere udsendt lys hvis
> man øger hastigheden til 102%?

Iflg. den specielle relativitetsteori (bekræftet utallige gange), er lysets
hastighed altid konstant for en lokal iagttager. Dvs. man altid vil måle en
bestemt hastighed af lyset, uanset hvordan det blev udsendt, eller hvordan
man bevæger sig i forhold til kilden. Selv om kilden bevæger sig med 98% af
lysets hastighed mod en, vil man måle, at lyset bevæger sig med 299792,458
km/s.


--
Hilsen
Regnar Simonsen



Carsten Svaneborg (23-10-2004)
Kommentar
Fra : Carsten Svaneborg


Dato : 23-10-04 23:56

Finn Guldmann wrote:
> Og så er det jeg mener der må være en måde at beskrive hvor det ene
> "Overalt" befinder sig i forhold til det andet.

Yeps. Den information er gemt i den måde man inden for kosmologi
beskriver koordinatsystemet. Man tænker sig et koordinat system
hvor alle galakse hobe sidder fast på faste koordinater, mens
skalen a(t) af koordinatsystemet selv vokser som tiden går.
Dermed øges de kosmologiske afstande løbende.

Tidsudviklingen af skala faktoren for de kosmologiske afstande a(t)
er hvad der beskrives af Friedmann ligningen, der igen er afledt
af Einstein generelle relativitetsteori samt Robinson-Walker
metrikken der antager at rummet er homogent og isotropisk.

(det skulle give dig nogle keywords at google efter)

http://scienceworld.wolfram.com/physics/EinsteinFieldEquations.html
http://scienceworld.wolfram.com/physics/FriedmannsEquation.html
(a(t) kaldes R i ovenstående)

Big Bang teorien er en teori for a(t) under antagelsen af forskellige
tilstandsligninger i strålings og stof dominerede æraer.

Hvad man f.eks. gør i dag er at lave målinger af en bestemt type
supernovaer for at estimere Hubble konstanten, man kan så fra a(t_nu)
og da/dt(t_nu) løse ligningerne og derfor udregne a(T_0)=0 og T_0 er
Big Bang øjeblikket. Så kender man universets alder.

> Der kan der defineres en række af hændelser bestående af
> postmedarbejdere og maskiner der fysisk flytter brevet.
> Men kan der defineres en "vej" lyset har tilbagelagt?

Yeps. Lyset følger en geodæt i rummet.

> Fandt et sted (på dr.dk) hvor de viser et billede af en galakse hvor de
> hævder at lyset har været 13 mia år undervejs (13 mia lysår gammelt).
Yeps. Og kigger du i den modsatte retning, kan du også tage et billed
af galakser der er 13 Glysår "borte".

> Vi må vel selv befinde os et eller andet sted i kugleskallen?

Kugleskallen er en fiktiv størrelse, der fortæller os hvor i rummet
vi idag modtager information fra. Vi befinder os i centrum af dem.

Præcist ligesom hvis du idag modtager en masse post, der blev sendt for
2 dage siden, og på kortet sætter mærker hvor brevene blev afsendt fra.
Hvis postbudet altid bevægede sig lige fra afsender til modtager med
en konstant hastighed, så ville det være en cirkel på jordoverfladen.

> På vej væk fra et eller andet sted hvor BB fandt sted for 13,7
> mia år siden?

Big Bang fandt sted alle steder i universet. Men den kosmiske stråling
i en "afstand" af 5G lysår er allerede passeret jorden (i alle retninger),
derfor ser vi ikke denne.

--
Mvh. Carsten Svaneborg
http://www.softwarepatenter.dk

Jens Olsen (22-10-2004)
Kommentar
Fra : Jens Olsen


Dato : 22-10-04 10:53

Finn Guldmann <n@a.invalid> wrote in message news:<cl9ruk$1ti4$1@news.cybercity.dk>...
> Under forudsætningerne at universet er skabt ved et BB og universet
> udvider sig lige meget i alle retninger, vil det jo aldrig kunne lade
> sig gøre at se længere tilbage i tiden end et vist tidspunkt.

Man kan se tilbage til det tidspunkt, hvor universet blev
gennemsigtigt for stråling af den bølgelængde man kigger efter.

> Ved en BB-teori må der jo være et centrum hvor alting udvider sig fra.

Nej. Tænk igen på anlogien med overfladen på ballonen der pustes op.

> Fra dette punkt vil alting, både lys og masse komme fra.

Nej, for sådan et punkt findes som sagt ikke.

> Teorierne siger at universet er omkring 13 mia år gammel. Hvis vi viste
> i hvilken retning "BB-punktet" var, og vi kunne se mere end 13 mia.
> lysår tilbage, ville det vi fik øje på jo være masse (sole, planeter,
> m.v.) der var på vej væk fra "BB-punktet" i modsat retning.

Igen nej, da punktet ikke eksisterer.

J.O.

Finn Guldmann (22-10-2004)
Kommentar
Fra : Finn Guldmann


Dato : 22-10-04 16:51

Jens Olsen wrote:

>>Ved en BB-teori må der jo være et centrum hvor alting udvider sig fra.
> Nej. Tænk igen på anlogien med overfladen på ballonen der pustes op.
>
Det gør jeg også. Men da teorien hævder at "alt" har været samlet i et
punkt der er mindre end et atom må det "sted" hvor dette punkt var vel
kunne betragtes som centrum for udvidelsen. For det er vel BB der har
leveret kraften til udvidelsen. Hvis vi holder os til "Ballon-teorien"?

>>Fra dette punkt vil alting, både lys og masse komme fra.
> Nej, for sådan et punkt findes som sagt ikke.
>
Men det må vel havde været et sted ved sekund 0 i universets udvikling?

>>Teorierne siger at universet er omkring 13 mia år gammel. Hvis vi viste
>>i hvilken retning "BB-punktet" var, og vi kunne se mere end 13 mia.
>>lysår tilbage, ville det vi fik øje på jo være masse (sole, planeter,
>>m.v.) der var på vej væk fra "BB-punktet" i modsat retning.
> Igen nej, da punktet ikke eksisterer.
>
Teoretisk må det vel være muligt at se fra det punkt på ballonen hvor vi
befinder os og over til den modsatte side. Et punkt der ligger omkring
27,4 mia (år*udvidelseshastighed) væk.

Men da lysets hastighed jo er størrere end udvidelseshastigheden, må lys
derfra, udsendt før et givent tidspunkt, jo forlængst være passeret her.

--
MVH Finn
To, på hinnanden følgende, undertekster i en udsendelse om
operasangere; "Jeg laver selv min mad" "og beder ofte til Gud"

Claus Christiansen (22-10-2004)
Kommentar
Fra : Claus Christiansen


Dato : 22-10-04 17:23

Finn Guldmann <n@a.invalid> wrote in news:clbad6$kem$1@news.cybercity.dk:

> Det gør jeg også. Men da teorien hævder at "alt" har været samlet i et
> punkt der er mindre end et atom må det "sted" hvor dette punkt var vel
> kunne betragtes som centrum for udvidelsen. For det er vel BB der har
> leveret kraften til udvidelsen. Hvis vi holder os til "Ballon-teorien"?
>
Du skal ikke tænke på BB som en eksplosion; tænk på BB som en ekspansion
*af* rummet og ikke som en eksplosion af stof *i* rummet. Al 'rum' var
samlet i BB, derfor er det 'sted' du søger overalt.


> Men det må vel havde været et sted ved sekund 0 i universets udvikling?
>
Se ovenfor.

> Men da lysets hastighed jo er størrere end udvidelseshastigheden
>
Nej, det er ikke rigtigt. Galakser der er meget langt væk fjerner sig,
pga udvidelse *af* rummet, hurtigere fra os end de galakser der er tæt på
os. Der er SVJV intet i vejen for, at meget fjerne objekter fjerner sig
fra os med over lysets hastighed. Det strider ikke mod 'tophastigheden'
c; som udtaler sig om bevægelse af lys *i* rummet.

Håber forskellen mellem *af* og *i* ovenfor hjælper :)

/Claus


Preben Riis Sørensen (22-10-2004)
Kommentar
Fra : Preben Riis Sørensen


Dato : 22-10-04 19:08


"Claus Christiansen" <claus_christiansen_nospam@hotmail.com> skrev
> Nej, det er ikke rigtigt. Galakser der er meget langt væk fjerner sig,
> pga udvidelse *af* rummet, hurtigere fra os end de galakser der er tæt på
> os. Der er SVJV intet i vejen for, at meget fjerne objekter fjerner sig
> fra os med over lysets hastighed. Det strider ikke mod 'tophastigheden'
> c; som udtaler sig om bevægelse af lys *i* rummet.

Han kan jo prøve med rosiner i en (uendelig stor) gærdej

Grænsen for det, der kan iagttages, ligger hvor galakserne 'fjerner sig' fra
hinanden hurtigere end lyset. Hvis man ser på en galakse der sendte sit lys
én mia. år efter big bang, ses nu, hvor vi modtager lyset, som en lille rød
prik i de bedste Hubblebilleder, 13 mia. lysår væk. Men der er den jo ikke
nu, den har siden været med i rummets udvidelse og er altså 26 mia. lysår
væk. Og så kan man vende kikkerten og finde en tilsvarende. De to galakser
er så 52 mia.lysår fra hinanden nu. Hvis altså teorien om rummets udvidelse
er korrekt. Det tror jeg dog ikke den er. For hvorhen skulle noget, der er
uendeligt stort, udvide sig hen?
--
M.V.H.
Preben Riis Sørensen
preben@esenet.dk




Finn Guldmann (22-10-2004)
Kommentar
Fra : Finn Guldmann


Dato : 22-10-04 20:08

Claus Christiansen wrote:

>>Det gør jeg også. Men da teorien hævder at "alt" har været samlet i et
>>punkt der er mindre end et atom må det "sted" hvor dette punkt var vel
>>kunne betragtes som centrum for udvidelsen. For det er vel BB der har
>>leveret kraften til udvidelsen. Hvis vi holder os til "Ballon-teorien"?
> Du skal ikke tænke på BB som en eksplosion; tænk på BB som en ekspansion
> *af* rummet og ikke som en eksplosion af stof *i* rummet. Al 'rum' var
> samlet i BB, derfor er det 'sted' du søger overalt.
>
Sakset fra en af DR's sider:
"Dette er Big Bang-teorien:
Alting udspringer fra en enkelt gigantisk eksplosion. Fra en plet, der
er mindre end et atom, opstår der materie, rum og tid. Det er glohedt.

I det første mikrosekund udvider universet sig til en galakses
størrelse. Det er en suppe af partikler, der bliver større og større.
Det er så varmt, at energi spontant udvikler materie og anti-materie. De
tilintetgør hinanden, og starter forfra. Materien vinder en kneben sejr
og kommer til at dominere vores univers - protoner, neutroner og
elektroner."

De skriver "en plet, der er mindre end et atom". Det tolker jeg som et
bestemt sted. Et sted der har et geografisk forhold til det univers vi
har i dag. F.eks. som centrum i den "ballonudvidelse" universet også
beskrives som.

>>Men da lysets hastighed jo er størrere end udvidelseshastigheden
> Nej, det er ikke rigtigt. Galakser der er meget langt væk fjerner sig,
> pga udvidelse *af* rummet, hurtigere fra os end de galakser der er tæt på
> os. Der er SVJV intet i vejen for, at meget fjerne objekter fjerner sig
> fra os med over lysets hastighed. Det strider ikke mod 'tophastigheden'
> c; som udtaler sig om bevægelse af lys *i* rummet.
> Håber forskellen mellem *af* og *i* ovenfor hjælper :)
>
Et sted så jeg udvidelseshastigheden som værene mellem "26-29
miles/sec.". Det får jeg til noget med 150-165.000 km/t.
Derfra er der langt op til lysets ca. 1.000.000.000 km/t.

Jeg har så været ved at lege lidt med tallene.
Med en udvidelseshastighed på 165.000 km/t vil et univers efter
"Ballon-teorien" kunne nå at blive 41.100.000.000.000.000.000 km i
diameter.
Lyset vil være 4.344.608 år om at nå fra et yderpunkt til et andet.

Og så er det jeg spørger om hvor det lys der hævdes at være ældre end
4,3 mill år "gemt" sig i de mellemliggende år?

--
MVH Finn
To, på hinnanden følgende, undertekster i en udsendelse om
operasangere; "Jeg laver selv min mad" "og beder ofte til Gud"

Claus Christiansen (22-10-2004)
Kommentar
Fra : Claus Christiansen


Dato : 22-10-04 22:15

Finn Guldmann <n@a.invalid> wrote in
news:clblun$10mi$1@news.cybercity.dk:

> Sakset fra en af DR's sider:
Bemærk flg fra dit 'saks':

> "Dette er Big Bang-teorien:
> Fra en plet [...] opstår der materie, rum og tid.
>
Selve *rummet* (og tiden) skabtes ved BB; så det giver ikke rigtig mening
at spørge om 'hvor' BB-pletten lå; en placering kræver et rum at være i
og selve rummet opstod først med BB.


> Et sted så jeg udvidelseshastigheden som værene mellem "26-29
> miles/sec.".
>
Universets udvidelse menes i dag at være accelererende. Desuden fjerner
vi os som sagt hurtigere fra i forvejen fjerne objekter end nærmere
objekter; derfor giver en fast hastighed som angivet kun meget ringe
mening: "26-29 miles/sec" i forhold til hvad??


> Og så er det jeg spørger om hvor det lys der hævdes at være ældre end
> 4,3 mill år "gemt" sig i de mellemliggende år?
>
Dit problem ligger i din forventning om at "BB's centrum" er et sted i
vores univers. Baggrundsstrålingen som (med lidt god vilje) kan betragtes
som 'glimtet' fra BB, kan stadig måles netop fordi det ikke er et
kugleformet glimt fra et punkt; men derimod stammer fra ethvert punkt
(derfor der osse er flere andre i tråden der skriver at BB skete
overalt).




Finn Guldmann (22-10-2004)
Kommentar
Fra : Finn Guldmann


Dato : 22-10-04 22:57

Claus Christiansen wrote:

>>Sakset fra en af DR's sider:
> Bemærk flg fra dit 'saks':
>>"Dette er Big Bang-teorien:
>>Fra en plet [...] opstår der materie, rum og tid.
> Selve *rummet* (og tiden) skabtes ved BB; så det giver ikke rigtig mening
> at spørge om 'hvor' BB-pletten lå; en placering kræver et rum at være i
> og selve rummet opstod først med BB.
>
Men i og med at vi bevæger os væk fra et eller andet, og resten af
universet gør det samme (i andre retninger), må det vel være muligt at
definere hvor dette "et eller andet" befinder sig (har befundet sig).
Hvor det er vi bevæger os væk fra.

>>Et sted så jeg udvidelseshastigheden som værene mellem "26-29
>>miles/sec.".
> Universets udvidelse menes i dag at være accelererende. Desuden fjerner
> vi os som sagt hurtigere fra i forvejen fjerne objekter end nærmere
> objekter; derfor giver en fast hastighed som angivet kun meget ringe
> mening: "26-29 miles/sec" i forhold til hvad??
>
I og med at det hævdes at der forekommer lys der har været undervejs i
13 mia år (13 mia lysår) i et univers der "kun" har bestået i 13,7 mia
år, holder den udvidelseshastighed ikke.

>>Og så er det jeg spørger om hvor det lys der hævdes at være ældre end
>>4,3 mill år "gemt" sig i de mellemliggende år?
> Dit problem ligger i din forventning om at "BB's centrum" er et sted i
> vores univers. Baggrundsstrålingen som (med lidt god vilje) kan betragtes
> som 'glimtet' fra BB, kan stadig måles netop fordi det ikke er et
> kugleformet glimt fra et punkt; men derimod stammer fra ethvert punkt
> (derfor der osse er flere andre i tråden der skriver at BB skete
> overalt).
>
Hvis universet udvider sig som en ballon (kugleskal) må der jo være et
centrum af denne kugle. Et centrum som alting har bevæget sig væk fra i
13,7 mia år. Mens det har ændret form og fysisk tilstand.

--
MVH Finn
To, på hinnanden følgende, undertekster i en udsendelse om
operasangere; "Jeg laver selv min mad" "og beder ofte til Gud"

Regnar Simonsen (22-10-2004)
Kommentar
Fra : Regnar Simonsen


Dato : 22-10-04 23:27

Finn Guldmann
> Men i og med at vi bevæger os væk fra et eller andet, og resten af
> universet gør det samme (i andre retninger), må det vel være muligt at
> definere hvor dette "et eller andet" befinder sig (har befundet sig).
> Hvor det er vi bevæger os væk fra.

Nej, vi bevæger os ikke væk fra noget fast punkt - men fra alle andre
galakser. Husk at universets udvidelse ikke er stof, der bevæger sig ud i et
allerede eksisterende rum.
Universets udvidelse skal heller ikke forstås på den måde, at universet
bliver større (i hvert fald ikke, hvis det er uendeligt); udvidelsen
manifesterer sig ved, at stoffet fjerner sig fra hinanden.

> Hvis universet udvider sig som en ballon (kugleskal) må der jo være et
> centrum af denne kugle. Et centrum som alting har bevæget sig væk fra i
> 13,7 mia år. Mens det har ændret form og fysisk tilstand.

Det er svagheden ved ballonmodellen. Ballonfladen omgiver et centrum, men
det er ikke gældende for universet.
Det er måske nemmere at forestille sig en elastikmodel. Man kan tegne en
række prikker (galakser) på et elastik, og derefter strække elastikken.
Ligemeget fra hvilken prik, man måler, vil man se, at de andre prikker
fjerner sig - og jo fjernere prikken er, jo hurtigere går det (man kan endda
finde en Hubble-konstant for elastikker). Ved BB har elastikken en længde på
0, og alle prikker er samlet i et punkt sammen med selve elastikken
(svarende til rumtiden). Svagheden ved elastikmodellen er, at nogle prikker
ligger tæt ved en rand, hvilket ikke gælder for galakser i rummet.


--
Hilsen
Regnar Simonsen



Finn Guldmann (23-10-2004)
Kommentar
Fra : Finn Guldmann


Dato : 23-10-04 00:48

Regnar Simonsen wrote:

>>Men i og med at vi bevæger os væk fra et eller andet, og resten af
>>universet gør det samme (i andre retninger), må det vel være muligt at
>>definere hvor dette "et eller andet" befinder sig (har befundet sig).
>>Hvor det er vi bevæger os væk fra.
> Nej, vi bevæger os ikke væk fra noget fast punkt - men fra alle andre
> galakser. Husk at universets udvidelse ikke er stof, der bevæger sig ud i et
> allerede eksisterende rum.
> Universets udvidelse skal heller ikke forstås på den måde, at universet
> bliver større (i hvert fald ikke, hvis det er uendeligt); udvidelsen
> manifesterer sig ved, at stoffet fjerner sig fra hinanden.
>
Men det må vel stadig bevæge sit en en bestemt retning? Eller er retning
vilkårlig, evt. skiftende?

>>Hvis universet udvider sig som en ballon (kugleskal) må der jo være et
>>centrum af denne kugle. Et centrum som alting har bevæget sig væk fra i
>>13,7 mia år. Mens det har ændret form og fysisk tilstand.
> Det er svagheden ved ballonmodellen. Ballonfladen omgiver et centrum, men
> det er ikke gældende for universet.
> Det er måske nemmere at forestille sig en elastikmodel. Man kan tegne en
> række prikker (galakser) på et elastik, og derefter strække elastikken.
> Ligemeget fra hvilken prik, man måler, vil man se, at de andre prikker
> fjerner sig - og jo fjernere prikken er, jo hurtigere går det (man kan endda
> finde en Hubble-konstant for elastikker). Ved BB har elastikken en længde på
> 0, og alle prikker er samlet i et punkt sammen med selve elastikken
> (svarende til rumtiden). Svagheden ved elastikmodellen er, at nogle prikker
> ligger tæt ved en rand, hvilket ikke gælder for galakser i rummet.
>
Jeg har lidt svært ved at kapere at noget bevæger sig uden at det
bevæger sig fra et sted og i retning af et andet sted.

Men har universet ikke udvidet sig lige meget i alle retninger fra BB?

--
MVH Finn
To, på hinnanden følgende, undertekster i en udsendelse om
operasangere; "Jeg laver selv min mad" "og beder ofte til Gud"

Carsten Svaneborg (24-10-2004)
Kommentar
Fra : Carsten Svaneborg


Dato : 24-10-04 00:26

Finn Guldmann wrote:
>> Universets udvidelse skal heller ikke forstås på den måde, at universet
>> bliver større (i hvert fald ikke, hvis det er uendeligt); udvidelsen
>> manifesterer sig ved, at stoffet fjerner sig fra hinanden.
> Men det må vel stadig bevæge sit en en bestemt retning?
> Eller er retning vilkårlig, evt. skiftende?

Sidder vi på jorden og kigger på galakserne vil de alle bevæge sig VÆK
fra os, ligegyldigt hvilken retning du kigger i. Sidder du nu i en nabo
galakse, så vil alle de galakser du kan se også bevæge sig VÆK. Alt
bevæger sig væk fra alt muligt andet.

Det giver ikke mening at tale om at vores galakse bevæger sig i en
bestemt retning i forhold til Big Bang, fordi Big Bang skete alle
steder i rummet. Enhver retning og ethvert punkt i rummet er derfor
lige god.

Du kan dog sige at solsystemet bevæger sig ifht. den kosmiske
baggrundstråling vi observerer. Ser du på det midterste billed på
nedenstående side, så er den kosmiske baggrundstråling "varmere"
i en retning end i den modsatte retning, fordi solsystemet bevæger
sig. http://map.gsfc.nasa.gov/m_uni/uni_101Flucts.html
Ligeledes bevæger solsystemet sig ifht. galaksen.

--
Mvh. Carsten Svaneborg
http://www.softwarepatenter.dk

Per Rønne (24-10-2004)
Kommentar
Fra : Per Rønne


Dato : 24-10-04 05:50

Carsten Svaneborg <zqex@sted.i.tyskland.de> wrote:

> Sidder vi på jorden og kigger på galakserne vil de alle bevæge sig VÆK
> fra os, ligegyldigt hvilken retning du kigger i.

Ikke helt korrekt. Der er faktisk galakser der bevæger sig mod os, eller
som vi bevæger os mod. Det gælder ikke mindst i vores lokale gruppe af
galakser, som bl.a. også indeholder Andromeda-galaksen og de to
Magellanske Skyer. Det gælder også superhobe af galakser, hvor bl.a. vi
bevæger os mod Virgo.

Kun på det helt store plan er det korrekt, at galakserne alle bevæger
sig bort fra os. Men der er det jo altså også rummet der udvider sig. I
noget tættere afstande er det ikke /nødvendigvis/ tilfældet.

Encyclopædia Britannica 2003:

Cosmos

Large-scale structure and expansion of the universe

Clustering of galaxies

The Local Group

The Local Group contains seven reasonably prominent galaxies and perhaps
another two dozen less conspicuous members. The dominant pair in the
group is the Milky Way andAndromeda, both giant spirals of Hubble type
Sb and luminosity class II. The distance to the Andromeda system was
first measured by Hubble, but his estimate was too low bya factor of two
because astronomers at that time did not recognize the distinction
between variable stars belonging to Population II (like those studied by
Shapley) and Population I (those studied by Hubble). Another spiral in
the Local Group - M33, Hubble type Sc and luminosity class III - is
notable, but the rest are intermediate to dwarf systems, either
irregulars or ellipticals. Most of the mass of the Local Group is
associated with the Milky Way and Andromeda, and with a few exceptions
the smaller systems tend to congregate about one or the other of these
galaxies. The size of the Local Group is therefore larger only by about
50 percent than the 2 * 10^6 light-years separating the Milky Way system
and the Andromeda galaxy, and the centre of mass lies roughly halfway
between these two giants.

The Andromeda galaxy is one of the few galaxies in the universe that
actually has a velocity of approach with respect to the centre of the
Galaxy. If this approach results from the reversal by the mutual
gravitational attraction of a former recession, then the total mass of
the Local Group probably amounts to a few times 1012 solar masses. This
is greater than the mass inferred for the optically visible parts of the
galaxies and is another manifestation of the dark matter problem.


Neighbouring groups and clusters

Beyond the fringes of the Local Group lie many similar small groups. The
best studied ofthese is the M81 group, whose dominant galaxy is the
spiral galaxy M81. Much like the Andromeda and Milky Way systems, M81 is
of Hubble type Sb and luminosity class II. The distance to M81, as well
as to the outlying galaxy NGC 2403, can be determined from various
stellar calibrators to be at a distance of 107 light-years. It is not
known whether NGC 2403 and its companion NGC 2366 are truly bound to M81
or whether they are an independent pair seen by chance to lie near the
M81 group. If they are bound to M81, then a measurement of their
velocity along the line of sight relative to that of M81 yields, by an
argument similar to that used for the Andromeda and Milky Way galaxies,
an estimate of the gravitating mass of M81. This estimate equals 2 ?
1012 solar masses and exceeds by an order of magnitude what is deduced
from measurements of the rotation curve of M81 inside its optically
visible disk.

The M81 group also has a few normal galaxies with classifications
similar to those of galaxies in the Local Group, and it was noticed by
some astronomers that the linear sizes of the largest H II regions
(which are illuminated by many OB stars) in these galaxies had about the
same intrinsic sizes as their counterparts in the Local Group. This led
Allan Sandage and the German chemist and physicist Gustav Tammann to the
(controversial) technique of using the sizes of H II regions as a
distance indicator, because a measurement of their angular sizes,
coupled with knowledge of their linear sizes, allows an inference of
distance.

This method can be used, for example, to obtain the distance to the M101
group, whose dominant galaxy M101 is a supergiant spiral - the closest
system of Hubble type Sc and luminosity class I. Since Sc I galaxies are
the most luminous spiral galaxies, with very large H II regions strung
out along their spiral arms, determining the distance to M101 is a
crucial step in obtaining the absolute sizes of the giant H II regions
of these important systems. The sizes of the H II regions in the
companion galaxies of M101 compared with the calibrated values for
nearby galaxies of the same class yield a distance to the M101 group of
approximately 2 ? 107 light-years.

Having calibrated the sizes of the giant H II regions in M101, Sandage
and Tammann could then obtain the distances to 50 field Sc I galaxies.
Once this had been done, it became possible to measure the absolute
brightnesses of Sc I galaxies, and it was ascertained that all such
systems have nearly the same luminosity. Since Sc I galaxies like M101
or M51 can be recognized on purely morphological grounds (well-organized
spiral structure with massive arms dominated by giant H II regions),
they can now be used as "standard candles" to help measure the distances
to irregular clusters that contain such galaxies (e.g., the Virgo
cluster containing the Sc I galaxy M100).

The Virgo cluster is the closest large cluster and is located at a
distance of about 5 * 10^7 light-years in the direction of the
constellation Virgo. About 200 bright galaxies reside in the Virgo
cluster, scattered in various subclusters whose largest concentration
(near the famous system M87) is about 5 * 10^6 light-years in diameter.
Of the galaxies in the Virgo cluster, 68 percent are spirals, 19 percent
are ellipticals, and the rest are irregulars or unclassified. Although
spirals are more numerous, the four brightest galaxies are giant
ellipticals, among them M87. Calibration of the absolute brightnesses of
these giant ellipticals allows a leap to the distant regular clusters.

The nearest rich cluster containing thousands of systems, the Coma
cluster, lies about seven times farther than the Virgo cluster in the
direction of the constellation Coma Berenices. The main body of the Coma
cluster has a diameter of about 2.5 * 10^7 light-years, but enhancements
above the background can be traced out to a supercluster of a diameter
of about 2 * 10^8 light-years. Ellipticals or S0s constitute 85 percent
of the bright galaxies in the Coma cluster; the two brightest
ellipticals in Coma are located near the centre of the system and are
individually more than 10 times as luminous as the Andromeda galaxy.
These galaxies have a swarm of smaller companions orbiting them and may
have grown to their bloated sizes by a process of "galactic cannibalism"
like that hypothesized to explain the supergiant elliptical cD systems
(see above).

The spatial distribution of galaxies in rich clusters such as the Coma
cluster closely resembles what one would expect theoretically for a
bound set of bodies moving in the collective gravitational field of the
system. Yet, if one measures the dispersion of random velocities of the
Coma galaxies about the mean, one finds that it amounts to almost 900
km/sec. For a galaxy possessing this random velocity along a typical
line of sight to be gravitationally bound within the known dimensions of
the cluster requires Coma to have a total mass of about 5 * 10^15 solar
masses. The total luminosity of the Coma cluster is measured to be about
3 * 10^13 solar luminosities; therefore, the mass-to-light ratio in
solar units required to explain Coma as a bound system exceeds by an
order of magnitude what can be reasonably ascribed to the known stellar
populations. A similar situation exists for every rich cluster that has
been examined in detail. This dark matter problem for rich clusters was
known to the Swiss astronomer Fritz Zwicky as early as 1933. The
discovery of X-ray-emitting gas in rich clusters has alleviated the
dynamic problem by a factor of about two, but a substantial discrepancy
remains.

Superclusters

In 1932 Harlow Shapley and Adelaide Ames introduced a catalog that
showed the distributions of galaxies brighter than 13th magnitude to be
quite different north and south of the plane of the Galaxy. Their study
was the first to indicate that the universe might contain substantial
regions that departed from the assumption of homogeneity and isotropy.
The most prominent feature in the maps they produced in 1938 was the
Virgo cluster, though already apparent at that time were elongated
appendages that stretched on both sides of Virgo to a total length
exceeding 5 * 10^7 light-years. This configuration is the kernel of what
came to be known later - through the work of Erik Holmberg, Gérard de
Vaucouleurs, and George O. Abell - as the Local Supercluster, a
flattened collection of about 100 groups and clusters of galaxies
including the Local Group. The Local Supercluster is centred
approximately on the Virgo cluster and has a total extent of roughly 2 *
10^8 light-years. Its precise boundaries, however, are difficult to
define inasmuch as the local enhancement in numbers of galaxies above
the cosmological average in all likelihood just blends smoothly into the
background.

Also apparent in the Shapley-Ames maps were three independent
concentrations of galaxies, separate superclusters viewed from a
distance. Astronomers now believe superclusters fill perhaps 10 percent
of the volume of the universe. Most galaxies, groups, and clusters
belong to superclusters, the space between superclusters being
relatively empty. The dimensions of superclusters range up to a few
times 10^8 light-years. For larger scales the distribution of galaxies
is essentially homogeneous and isotropic - that is, there is no evidence
for the clustering of superclusters. This fact can be understood by
recognizing that the time it takes a randomly moving galaxy to traverse
the long axis of a supercluster is typically comparable to the age of
the universe. Thus, if the universe started out homogeneous and
isotropic on small scales, there simply has not been enough time for it
to become inhomogeneous on scales much larger than superclusters. This
interpretation is consistent with the observation that superclusters
themselves look dynamically unrelaxed - that is, they lack the regular
equilibrium shapes and central concentrations that typify systems well
mixed by several crossings.

--
Per Erik Rønne

Carsten Svaneborg (25-10-2004)
Kommentar
Fra : Carsten Svaneborg


Dato : 25-10-04 12:53

Per Rønne wrote:
>> Sidder vi på jorden og kigger på galakserne vil de alle bevæge sig VÆK
>> fra os, ligegyldigt hvilken retning du kigger i.
> Ikke helt korrekt. Der er faktisk galakser der bevæger sig mod os, eller
> som vi bevæger os mod.

Enig. Jeg (over)simplificerede.

--
Mvh. Carsten Svaneborg
http://www.softwarepatenter.dk

Finn Guldmann (25-10-2004)
Kommentar
Fra : Finn Guldmann


Dato : 25-10-04 07:28

Carsten Svaneborg wrote:

> Sidder vi på jorden og kigger på galakserne vil de alle bevæge sig VÆK
> fra os, ligegyldigt hvilken retning du kigger i. Sidder du nu i en nabo
> galakse, så vil alle de galakser du kan se også bevæge sig VÆK. Alt
> bevæger sig væk fra alt muligt andet.
>
Betyder det at de andre galakser bevæger sig væk fra os i en ret linie
der er vinkelret på vores grundplan. Eller er der galakser der er på vej
lidt mere nord, syd, øst og/eller vest på end andre?

> Det giver ikke mening at tale om at vores galakse bevæger sig i en
> bestemt retning i forhold til Big Bang, fordi Big Bang skete alle
> steder i rummet. Enhver retning og ethvert punkt i rummet er derfor
> lige god.
>
Den der har jeg altså stadig svært ved at fatte.

For at man kan have en bevægelse må der jo være en kraft der skaber
denne bevægelse. Og hvis den kraft der gør at universet udvides komme
fra BB, må bevægelsen jo ske væk fra BB. Ergo må der kunne defineres et
sted for hvor BB skete ved at udregne hvilken retning de forskellige
galakser bevæger sig i.

> Du kan dog sige at solsystemet bevæger sig ifht. den kosmiske
> baggrundstråling vi observerer. Ser du på det midterste billed på
> nedenstående side, så er den kosmiske baggrundstråling "varmere"
> i en retning end i den modsatte retning, fordi solsystemet bevæger
> sig. http://map.gsfc.nasa.gov/m_uni/uni_101Flucts.html
> Ligeledes bevæger solsystemet sig ifht. galaksen.
>
Den bevægelse er vist den der kommer af at vi sidder på en planet der
roterer rundt om en sol, der igen roterer rundt i en galakse, der igen
roterer rundt i en galaksehob.

Men alene det at en roterende bevægelse lader sig konstatere må vel
betyde de retninger de øvrige galakser bevæger sig i ikke "bare" er væk
fra os?

--
MVH Finn
To, på hinnanden følgende, undertekster i en udsendelse om
operasangere; "Jeg laver selv min mad" "og beder ofte til Gud"

Henning Makholm (25-10-2004)
Kommentar
Fra : Henning Makholm


Dato : 25-10-04 15:21

Scripsit Finn Guldmann <n@a.invalid>

> For at man kan have en bevægelse må der jo være en kraft der skaber
> denne bevægelse.

Nej. Siden Newton har man kunne have bevægelse uden kræfter. Når der
ikke er nogen kraft på et legeme, beholder det den hastighed det nu
engang har. For at *ændre* bevægelsen kræves kraft.

Big bang i sig selv forklarer ikke hvorfor universets og dets stof
blev skabt med et bevægelsesmønster væk fra hinanden. Vi kan blot
konstatere at sådan er det.

--
Henning Makholm "*Vi vil ha wienerbrød!*"

Finn Guldmann (25-10-2004)
Kommentar
Fra : Finn Guldmann


Dato : 25-10-04 19:39

Henning Makholm wrote:

>>For at man kan have en bevægelse må der jo være en kraft der skaber
>>denne bevægelse.
> Nej. Siden Newton har man kunne have bevægelse uden kræfter. Når der
> ikke er nogen kraft på et legeme, beholder det den hastighed det nu
> engang har. For at *ændre* bevægelsen kræves kraft.
> Big bang i sig selv forklarer ikke hvorfor universets og dets stof
> blev skabt med et bevægelsesmønster væk fra hinanden. Vi kan blot
> konstatere at sådan er det.
>
Netop. Men den kraft der har startet "indholdet" i universet i dets
"rejse" må jo være kommet et sted fra.

En masse der ikke, og aldrig har været, påvirket af en kraft vil jo
ligge stille.

Men da det, tilsyneladende, kan konstateres at massen i universet, i
dens forskellige former, er på vej væk fra hinnanden, må der jo havde
været en kraft der har startet denne bevægelse.

Hvorfra denne kraft så er kommet fra er åbenbart det store spørgsmål.

Når jeg opfatter det som at BB er kilden til denne kraft, og at
bevægelserne i universet derfor må have en retning væk fra BB, får jeg
at vide at det er forkert. Og får at vide at BB har været "over alt".

Når jeg så opfatter _det_ som at kraften til bevægelsen kommer alle
steder fra, og at det så må betyde at div. objekterne i universet er på
vej i alle mulige mærkelige retninger, får jeg at vide at det også er
forkert. Fordi jeg fortælles at alle objekterne bevæger sig væk fra os.

Men jeg mangler endnu at finde sammenhængen i det.

--
MVH Finn
To, på hinnanden følgende, undertekster i en udsendelse om
operasangere; "Jeg laver selv min mad" "og beder ofte til Gud"

Per Rønne (25-10-2004)
Kommentar
Fra : Per Rønne


Dato : 25-10-04 20:53

Finn Guldmann <n@a.invalid> wrote:

> Men den kraft der har startet "indholdet" i universet i dets "rejse" må jo
> være kommet et sted fra.

Men intet tyder på at vi bliver i stand til, inden for naturvidenskaben,
at finde ud af det. Den må du finde inden for teologien i stedet for.

Personligt tror jeg ikke at det nogensinde bliver muligt at »se« tilbage
til et tidspunk før Planck tid.
--
Per Erik Rønne

Finn Guldmann (25-10-2004)
Kommentar
Fra : Finn Guldmann


Dato : 25-10-04 21:56

Per Rønne wrote:

>>Men den kraft der har startet "indholdet" i universet i dets "rejse" må jo
>>være kommet et sted fra.
> Men intet tyder på at vi bliver i stand til, inden for naturvidenskaben,
> at finde ud af det. Den må du finde inden for teologien i stedet for.
>
Nu er det et område jeg ikke tror på kan vække den store interesse hos
mig.

> Personligt tror jeg ikke at det nogensinde bliver muligt at »se« tilbage
> til et tidspunk før Planck tid.
>
Det kan jeg heller ikke lige se hvordan skulle kunne lade dig gøre.

--
MVH Finn
To, på hinnanden følgende, undertekster i en udsendelse om
operasangere; "Jeg laver selv min mad" "og beder ofte til Gud"

Per Rønne (26-10-2004)
Kommentar
Fra : Per Rønne


Dato : 26-10-04 05:41

Finn Guldmann <n@a.invalid> wrote:

> Per Rønne wrote:
>
> >>Men den kraft der har startet "indholdet" i universet i dets "rejse" må jo
> >>være kommet et sted fra.
> > Men intet tyder på at vi bliver i stand til, inden for naturvidenskaben,
> > at finde ud af det. Den må du finde inden for teologien i stedet for.
> >
> Nu er det et område jeg ikke tror på kan vække den store interesse hos
> mig.

Ja, det jeg siger her er blot at her må vi forlade os på tro, ikke på
videnskab.
--
Per Erik Rønne

Henning Makholm (25-10-2004)
Kommentar
Fra : Henning Makholm


Dato : 25-10-04 21:02

Scripsit Finn Guldmann <n@a.invalid>
> Henning Makholm wrote:

> > Big bang i sig selv forklarer ikke hvorfor universets og dets stof
> > blev skabt med et bevægelsesmønster væk fra hinanden. Vi kan blot
> > konstatere at sådan er det.

> Netop. Men den kraft der har startet "indholdet" i universet i dets
> "rejse" må jo være kommet et sted fra.

En kraft kræves for at skabe en *ændring* i hastighed. Hvis universet
er skabt sådan at tingene allerede bevægede sig fra hinanden fra
begyndelsen, er der ingen *ændring* i hastighederne og derfor heller
ikke noget behov for en kraft.

> En masse der ikke, og aldrig har været, påvirket af en kraft vil jo
> ligge stille.

"Ligge stille" i forhold til hvad? Der er ikke noget absolut rum man
kan definere "ligge stille" ud fra.

--
Henning Makholm "Kurt er den eneste jeg kender der er
*dum* nok til at gå i *ring* på et jernbanespor."

Finn Guldmann (25-10-2004)
Kommentar
Fra : Finn Guldmann


Dato : 25-10-04 22:03

Henning Makholm wrote:

>>>Big bang i sig selv forklarer ikke hvorfor universets og dets stof
>>>blev skabt med et bevægelsesmønster væk fra hinanden. Vi kan blot
>>>konstatere at sådan er det.
>>Netop. Men den kraft der har startet "indholdet" i universet i dets
>>"rejse" må jo være kommet et sted fra.
> En kraft kræves for at skabe en *ændring* i hastighed. Hvis universet
> er skabt sådan at tingene allerede bevægede sig fra hinanden fra
> begyndelsen, er der ingen *ændring* i hastighederne og derfor heller
> ikke noget behov for en kraft.
>
Det eneste der undrer mig i den forbindelse er at det siges at
universets udvidelseshastighed øges.
Men ikke så meget om hvor den kraft der skulle få det til at ske kommer fra.

>>En masse der ikke, og aldrig har været, påvirket af en kraft vil jo
>>ligge stille.
> "Ligge stille" i forhold til hvad? Der er ikke noget absolut rum man
> kan definere "ligge stille" ud fra.
>
F.eks. ligge stille i forhold til sine omgivelser.

--
MVH Finn
To, på hinnanden følgende, undertekster i en udsendelse om
operasangere; "Jeg laver selv min mad" "og beder ofte til Gud"

Henning Makholm (25-10-2004)
Kommentar
Fra : Henning Makholm


Dato : 25-10-04 22:14

Scripsit Finn Guldmann <n@a.invalid>

> Det eneste der undrer mig i den forbindelse er at det siges at
> universets udvidelseshastighed øges.
> Men ikke så meget om hvor den kraft der skulle få det til at ske kommer fra.

Her er du i din ret til at undre dig; det undrer også de astronomer
og kosmologer der for nyligt har fået præcise nok observationer til
at opdage at udvigelseshastigeden tilsyneladende stiger. Hvis du
spørger intrængende nok hvorfor, ender de med at mumle noget om "mørk
energi"; det er den p.t. herskende videnskabelige jargon for "vi
forstår endnu ikke hvad det skyldes".

> >>En masse der ikke, og aldrig har været, påvirket af en kraft vil jo
> >>ligge stille.

> > "Ligge stille" i forhold til hvad?

> F.eks. ligge stille i forhold til sine omgivelser.

Men i et (uniformt) ekspanderende univers ligger enhver galakse også
så stille i forhold til sine nærmeste nabogalakser som det er muligt.

--
Henning Makholm "The great secret, known to internists and
learned early in marriage by internists' wives, but
still hidden from the general public, is that most things get
better by themselves. Most things, in fact, are better by morning."

Finn Guldmann (25-10-2004)
Kommentar
Fra : Finn Guldmann


Dato : 25-10-04 22:42

Henning Makholm wrote:

> Her er du i din ret til at undre dig; det undrer også de astronomer
> og kosmologer der for nyligt har fået præcise nok observationer til
> at opdage at udvigelseshastigeden tilsyneladende stiger. Hvis du
> spørger intrængende nok hvorfor, ender de med at mumle noget om "mørk
> energi"; det er den p.t. herskende videnskabelige jargon for "vi
> forstår endnu ikke hvad det skyldes".
>
Det er da altid noget at det ikke er fordi jeg er helt ude i hampen.

> Men i et (uniformt) ekspanderende univers ligger enhver galakse også
> så stille i forhold til sine nærmeste nabogalakser som det er muligt.
>
Ja, til de nærmeste, men ikke til helheden.

--
MVH Finn
To, på hinnanden følgende, undertekster i en udsendelse om
operasangere; "Jeg laver selv min mad" "og beder ofte til Gud"

Henning Makholm (26-10-2004)
Kommentar
Fra : Henning Makholm


Dato : 26-10-04 00:29

Scripsit Finn Guldmann <n@a.invalid>
> Henning Makholm wrote:

> > Men i et (uniformt) ekspanderende univers ligger enhver galakse også
> > så stille i forhold til sine nærmeste nabogalakser som det er muligt.

> Ja, til de nærmeste, men ikke til helheden.

Helheden har ikke noget veldefineret hvilesystem man kan udtale sig om
en hastighed i forhold til.

--
Henning Makholm "The compile-time type checker for this
language has proved to be a valuable filter which
traps a significant proportion of programming errors."

Carsten Svaneborg (25-10-2004)
Kommentar
Fra : Carsten Svaneborg


Dato : 25-10-04 17:35

Finn Guldmann wrote:
> Betyder det at de andre galakser bevæger sig væk fra os i en ret linie
> der er vinkelret på vores grundplan. Eller er der galakser der er på vej
> lidt mere nord, syd, øst og/eller vest på end andre?

Fjernere galakser vil bevæge sig mere lige væk fra os end
nærmere galakser der kan have en egen bevægelse ifht. den
bevægelse ekspansionen er skyld i af rent geometriske årsager.


>> Enhver retning og ethvert punkt i rummet er derfor lige god.
> Den der har jeg altså stadig svært ved at fatte.

Det er en af de fundamentale antagelser i kosmologi.


> For at man kan have en bevægelse må der jo være en kraft der skaber
> denne bevægelse.

Krafter accelerere objekter uden kraft vil objekter fortsætte med at
bevæge sig langs en lige linie. Galakserne er stort set i hvile ifht.
rummets ekspansion.

Bemærk at retningen som et objekt bevæger sig i afhænger ikke kun
af objektet, men også din egen bevægelse ifht. objektet. Da der ikke
er noget koordinatsystem, der er specielt godt, er der ikke nogen
retning for objektets bevægelse der er specielt god eller rigtig.


> Og hvis den kraft der gør at universet udvides komme
> fra BB, må bevægelsen jo ske væk fra BB.

Energien der driver universets udvidelse idag (den kosmologiske konstant)
kommer fra universet idag og ikke fra Big Bang, men som sagt er det en
konstant og den variere derfor ikke i rummet. Derfor giver det ikke mening
at tale om at der er et sted hvor den kosmologiske konstant er størst, og
kalde dette for "centrum".

Præcist hvilke fysiske årsag der er til at vi har en kosmologisk konstant
er dog uvist, men det kunne være en konsekvens af nulpunkts
fluktuationerne af kvantefelter.


> Ergo må der kunne defineres et sted for hvor BB skete ved at
> udregne hvilken retning de forskellige galakser bevæger sig i.

Enig, men

Kigger du på galakserne så bevæger de sig alle væk fra os her på
jorden, ergo er jorden stedet hvor Big Bang skete. Sidder du på
en anden galakse, så vil du se at alle andre galakser bevæger
sig bort, ergo skete Big Bang også der.

Sidder du et vilkårligt sted i rummet vil du se at alle galakser
bevæger sig bort fra dig, ergo skete Big Bang også der.

Big Bang skete altså i alle punkter i universet, og alt bevæger
sig væk fra alt (på store længde skalaer).


> Men alene det at en roterende bevægelse lader sig konstatere må vel
> betyde de retninger de øvrige galakser bevæger sig i ikke "bare" er væk
> fra os?

Nu er væk fra mælkevejen nu et mere fornuftigt koordinatsystem
end væk fra jorden. Men disse hastigheder er små ifht. galaksernes
relative hastigheder.

--
Mvh. Carsten Svaneborg
http://www.softwarepatenter.dk

Finn Guldmann (25-10-2004)
Kommentar
Fra : Finn Guldmann


Dato : 25-10-04 20:42

Carsten Svaneborg wrote:

>>Betyder det at de andre galakser bevæger sig væk fra os i en ret linie
>>der er vinkelret på vores grundplan. Eller er der galakser der er på vej
>>lidt mere nord, syd, øst og/eller vest på end andre?
> Fjernere galakser vil bevæge sig mere lige væk fra os end
> nærmere galakser der kan have en egen bevægelse ifht. den
> bevægelse ekspansionen er skyld i af rent geometriske årsager.
>
Er det med de fjernere galakser ikke kun et spørgsmål om at den størrere
afstand gør det sværere at afgøre retningen mere nøjagtig?

>>>Enhver retning og ethvert punkt i rummet er derfor lige god.
>>Den der har jeg altså stadig svært ved at fatte.
> Det er en af de fundamentale antagelser i kosmologi.
>
Den må jeg se om jeg kan fatte en dag.

>>For at man kan have en bevægelse må der jo være en kraft der skaber
>>denne bevægelse.
> Krafter accelerere objekter uden kraft vil objekter fortsætte med at
> bevæge sig langs en lige linie. Galakserne er stort set i hvile ifht.
> rummets ekspansion.
>
I forhold til ekspasionen, ja. Men er det ikke netop fordi de selv er en
del af ekspansionen? Fordi de indeholder en enerti fra tidligere, som
ingen anden kraft har forsøgt at fravriste dem?

> Bemærk at retningen som et objekt bevæger sig i afhænger ikke kun
> af objektet, men også din egen bevægelse ifht. objektet. Da der ikke
> er noget koordinatsystem, der er specielt godt, er der ikke nogen
> retning for objektets bevægelse der er specielt god eller rigtig.
>
Jeg kan forstå så meget at et objekt der bevæger sig i samme retning som
os ("Jagter os") kan se ud som om det fjerner sig fra os fordi det har
en lavere bevægelseshastighed end os.

Men det med retningen var noget med om det kan konstateres om objekter
bevæger sig væk fra os i retninger der er lig med vores synsretning til
det enkelte objekt? Eller om det kan konstateres om det enkelte objekt
kan være nogle grader "ude af kurs"? Og i det sidste tilfælde, om der så
kan ses et mønster i disse retninger?

>>Og hvis den kraft der gør at universet udvides komme
>>fra BB, må bevægelsen jo ske væk fra BB.
> Energien der driver universets udvidelse idag (den kosmologiske konstant)
> kommer fra universet idag og ikke fra Big Bang, men som sagt er det en
> konstant og den variere derfor ikke i rummet. Derfor giver det ikke mening
> at tale om at der er et sted hvor den kosmologiske konstant er størst, og
> kalde dette for "centrum".
>
Energi, herunder bevægelsesenergi, er jo ikke noget der bare opstår. Den
må jo nødvendigvis komme et eller andet sted fra.

Så et objekt der bevæger sig i universet må jo havde fået tilført
bevægelsesenergi på et tidspunkt. En energi der så ligger oplagret i
objektet som enerti så længe objektet ikke påvirkes af andre energier.

Ergo må den udvidelse der sker af universet også kunne ses om resultatet
af at BB har tilført, stort set, al den masse der er i universet en
bevægelsesenergi der betyder at al massen er på vej væk fra det punkt
hvor BB fandt sted. Og da der ikke har været noget til at stoppe denne
bevægelsesenergi foregår udvidelsen stadig nu 13,7 mia år senere.

Der har så, lokalt, været andre energier, f.eks. tiltrækningskraft, der
har fået universets masse til at samle sig i de objekter vi kan se i
dag. Men uden at det er gået ud over de invorlverde objekters samlede
enerti fra BB.

>>Ergo må der kunne defineres et sted for hvor BB skete ved at
>>udregne hvilken retning de forskellige galakser bevæger sig i.
> Enig, men
> Kigger du på galakserne så bevæger de sig alle væk fra os her på
> jorden, ergo er jorden stedet hvor Big Bang skete. Sidder du på
> en anden galakse, så vil du se at alle andre galakser bevæger
> sig bort, ergo skete Big Bang også der.
>
> Sidder du et vilkårligt sted i rummet vil du se at alle galakser
> bevæger sig bort fra dig, ergo skete Big Bang også der.
>
> Big Bang skete altså i alle punkter i universet, og alt bevæger
> sig væk fra alt (på store længde skalaer).
>
Den "har jeg fået" en gang, men kan det konstateres om de forskellige
objekter holder en kurs der er lig med vores synsretning, eller er der
(ganske små) afvigelser?

>>Men alene det at en roterende bevægelse lader sig konstatere må vel
>>betyde de retninger de øvrige galakser bevæger sig i ikke "bare" er væk
>>fra os?
>
>
> Nu er væk fra mælkevejen nu et mere fornuftigt koordinatsystem
> end væk fra jorden. Men disse hastigheder er små ifht. galaksernes
> relative hastigheder.
>
Svjv kan det konstateres at andre objekter i Mælkevejen bevæger sig i
andre retninger end væk fra os. Nogle enda vistnok i retning af os.
Det er det forhold jeg spørger efter om det kan overføres til resten af
universet.

At det vil være utroligt meget sværere at konstatere, kvag de størrere
afstande er jeg godt klar over.

--
MVH Finn
To, på hinnanden følgende, undertekster i en udsendelse om
operasangere; "Jeg laver selv min mad" "og beder ofte til Gud"

Jens Olsen (23-10-2004)
Kommentar
Fra : Jens Olsen


Dato : 23-10-04 09:51

"Regnar Simonsen" <regnar.simo@image.dk> wrote in message news:<41798927$0$77008$14726298@news.sunsite.dk>...
> > Hvis universet udvider sig som en ballon (kugleskal) må der jo være et
> > centrum af denne kugle. Et centrum som alting har bevæget sig væk fra i
> > 13,7 mia år. Mens det har ændret form og fysisk tilstand.

Ok, det er fordi du ikke helt har forstået ballonanalogien så. Det er
jo netop kun en analogi.

Altså universet er ballonens overflade, og KUN overfladen. Altså et
todimensionelt univers. Ballonens indre er slet ikke en del af
universet eller i det hele taget relevant i sammenhængen.

Lige gyldigt hvad punkt på ballonens overflade du står på, når
ballonens pustes op, så vil du se alle andre punkter bevæge sig væk
fra dig. Du kan ikke sige, at et bestemt punkt på ballonens overflade
er centrum for ekspansionen.

Hjælper det dig til at forstår det.

J.O.

Finn Guldmann (23-10-2004)
Kommentar
Fra : Finn Guldmann


Dato : 23-10-04 11:50

Jens Olsen wrote:

>>>Hvis universet udvider sig som en ballon (kugleskal) må der jo være et
>>>centrum af denne kugle. Et centrum som alting har bevæget sig væk fra i
>>>13,7 mia år. Mens det har ændret form og fysisk tilstand.
> Ok, det er fordi du ikke helt har forstået ballonanalogien så. Det er
> jo netop kun en analogi.
> Altså universet er ballonens overflade, og KUN overfladen. Altså et
> todimensionelt univers. Ballonens indre er slet ikke en del af
> universet eller i det hele taget relevant i sammenhængen.
> Lige gyldigt hvad punkt på ballonens overflade du står på, når
> ballonens pustes op, så vil du se alle andre punkter bevæge sig væk
> fra dig. Du kan ikke sige, at et bestemt punkt på ballonens overflade
> er centrum for ekspansionen.
> Hjælper det dig til at forstår det.
>
Det giver mig to problemer.

For det første; Hvad så med det jeg kan opfatte i det tredie plan (3D)?
Og hvad forhold har det plan (2D) du beskriver i forhold til
udvidelsesretningen?

For det andet får den beskrivelse mig til at tænke på om det er et
forsøg på at gøre jorden til centrum for udvidelsen.

Men hvis alting fjerner sig fra ethvert punkt i universet, hvordan
harmonerer det så med BB?

For med BB må vi jo være på vej væk fra det sted hvor alting var samlet
lige efter BB. Det må jo så betyde, det der ikke er kommet så langt i
udvidelsen som vi er ikke rejser med samme hastighed som vi gør. For at
vores afstand til det kan øges.

Og så kommer det store spørgsmål; Hvilken kraft får hastigheden til at
stige?

--
MVH Finn
To, på hinnanden følgende, undertekster i en udsendelse om
operasangere; "Jeg laver selv min mad" "og beder ofte til Gud"

ML-78 (23-10-2004)
Kommentar
Fra : ML-78


Dato : 23-10-04 13:24

Finn Guldmann skrev:

> Det giver mig to problemer.
>
> For det første; Hvad så med det jeg kan opfatte i det tredie plan
(3D)?

Det kan du ikke. Der findes ikke noget tredje plan for dem der bor i det
hypotetiske 2-dimensionelle univers på ballonen.

> For det andet får den beskrivelse mig til at tænke på om det er et
> forsøg på at gøre jorden til centrum for udvidelsen.

Tværtimod. Beskrivelsen er et forsøg på at illustrere, at der ikke er
noget centrum for udvidelsen. Eller at det punkt udvidelsen begyndte fra
*er* hele universet.

> Men hvis alting fjerner sig fra ethvert punkt i universet, hvordan
> harmonerer det så med BB?

Fint. Det er netop det teorien går ud på.

> For med BB må vi jo være på vej væk fra det sted hvor alting var
samlet
> lige efter BB.

Nej, vi er på vej væk fra alle aldre punkter i universet. Ligesom
ethvert andet punkt er på vej væk fra alle punkter.


ML-78


Finn Guldmann (25-10-2004)
Kommentar
Fra : Finn Guldmann


Dato : 25-10-04 06:34

ML-78 wrote:

>>Det giver mig to problemer.
>>For det første; Hvad så med det jeg kan opfatte i det tredie plan
>>(3D)?
> Det kan du ikke. Der findes ikke noget tredje plan for dem der bor i det
> hypotetiske 2-dimensionelle univers på ballonen.
>
Så er det da godt at det kun er hypotetisk. For nu kan jeg jo se
stjerner m.v. i alle tre plan.

>>For det andet får den beskrivelse mig til at tænke på om det er et
>>forsøg på at gøre jorden til centrum for udvidelsen.
> Tværtimod. Beskrivelsen er et forsøg på at illustrere, at der ikke er
> noget centrum for udvidelsen. Eller at det punkt udvidelsen begyndte fra
> *er* hele universet.
>>Men hvis alting fjerner sig fra ethvert punkt i universet, hvordan
>>harmonerer det så med BB?
> Fint. Det er netop det teorien går ud på.
>>For med BB må vi jo være på vej væk fra det sted hvor alting var
>> samlet lige efter BB.
> Nej, vi er på vej væk fra alle aldre punkter i universet. Ligesom
> ethvert andet punkt er på vej væk fra alle punkter.
>
Så er det da vist godt at det kun er hypotetisk. For ellers skal vi jo
til at udtænke et helt nyt sæt naturlove.

For hvis den teori holder vand, som jeg forstår den, kan vi jo ikke
stole den mindste smugle på at det vi kan se også er rigtig. Og at,
f.eks., opgivelserne af universets alder har blot det mindste med
realiteterne at gøre.

--
MVH Finn
To, på hinnanden følgende, undertekster i en udsendelse om
operasangere; "Jeg laver selv min mad" "og beder ofte til Gud"

ML-78 (25-10-2004)
Kommentar
Fra : ML-78


Dato : 25-10-04 08:55

Finn Guldmann skrev:

> > Det kan du ikke. Der findes ikke noget tredje plan for dem der bor i
det
> > hypotetiske 2-dimensionelle univers på ballonen.
> >
> Så er det da godt at det kun er hypotetisk. For nu kan jeg jo se
> stjerner m.v. i alle tre plan.

Ja, vores univers har tre rumlige dimensioner, mens universet i
ballon-analogien kun har to.

> > Nej, vi er på vej væk fra alle aldre punkter i universet. Ligesom
> > ethvert andet punkt er på vej væk fra alle punkter.
> >
> Så er det da vist godt at det kun er hypotetisk. For ellers skal vi jo
> til at udtænke et helt nyt sæt naturlove.

Det er ikke hypotetisk. At galakserne fjerner sig fra hinanden er
velunderbygget, og det er ikke i strid med de kendte naturlove. Det
hypotetiske er ballon-analogiens 2D-univers.

> For hvis den teori holder vand, som jeg forstår den, kan vi jo ikke
> stole den mindste smugle på at det vi kan se også er rigtig. Og at,
> f.eks., opgivelserne af universets alder har blot det mindste med
> realiteterne at gøre.

Universets alder er jo netop bestemt ud fra antagelsen om at den teori
er korrekt, så jeg forstår ikke helt hvad du mener problemet er.


ML-78


Finn Guldmann (25-10-2004)
Kommentar
Fra : Finn Guldmann


Dato : 25-10-04 14:05

ML-78 wrote:

>>Så er det da godt at det kun er hypotetisk. For nu kan jeg jo se
>>stjerner m.v. i alle tre plan.
> Ja, vores univers har tre rumlige dimensioner, mens universet i
> ballon-analogien kun har to.
>
Gør man så ligesom med lovgiverne; Griner af dem når man finder ud af at
det man får fortalt af "eksperterne" ikke "lever op til" det man kan se
og opfatte?

>>>Nej, vi er på vej væk fra alle aldre punkter i universet. Ligesom
>>>ethvert andet punkt er på vej væk fra alle punkter.
>>Så er det da vist godt at det kun er hypotetisk. For ellers skal vi jo
>>til at udtænke et helt nyt sæt naturlove.
> Det er ikke hypotetisk. At galakserne fjerner sig fra hinanden er
> velunderbygget, og det er ikke i strid med de kendte naturlove. Det
> hypotetiske er ballon-analogiens 2D-univers.
>
Det der konflikter er det at et 3D univers skal forklares i 2D.
Det der, efter min mening, bliver problemet er at der bliver for mange
ubekendter. Og det derfor kun er teoretiske forklaringer der bliver
brugt. Og teorier det vil tage tusinder, mill., eller mia. år at bevise.
Mao. at de ikke kan bevises/afvises i vores levetid.

>>For hvis den teori holder vand, som jeg forstår den, kan vi jo ikke
>>stole den mindste smugle på at det vi kan se også er rigtig. Og at,
>>f.eks., opgivelserne af universets alder har blot det mindste med
>>realiteterne at gøre.
> Universets alder er jo netop bestemt ud fra antagelsen om at den teori
> er korrekt, så jeg forstår ikke helt hvad du mener problemet er.
>
At det er teorier der fortæller alderen (og udvidelsen og andet). Men
teorier der først kan bevises/afvises når vi ikke er her mere. (Evt.
endda som menneskehed)

--
MVH Finn
To, på hinnanden følgende, undertekster i en udsendelse om
operasangere; "Jeg laver selv min mad" "og beder ofte til Gud"

ML-78 (25-10-2004)
Kommentar
Fra : ML-78


Dato : 25-10-04 15:51

Finn Guldmann skrev:

> > Ja, vores univers har tre rumlige dimensioner, mens universet i
> > ballon-analogien kun har to.
> >
> Gør man så ligesom med lovgiverne; Griner af dem når man finder ud af
at
> det man får fortalt af "eksperterne" ikke "lever op til" det man kan
se
> og opfatte?

Det lader til, at du ikke er helt med på forskellen mellem den
videnskabelige teori og så en populær fremstilling, der skal illustrere
teorien for lægfolk. Den ballon-analogi vi taler om, er ikke en
videnskabelig model, som eksperterne har fremsat som forklaring.

> > Det er ikke hypotetisk. At galakserne fjerner sig fra hinanden er
> > velunderbygget, og det er ikke i strid med de kendte naturlove. Det
> > hypotetiske er ballon-analogiens 2D-univers.
> >
> Det der konflikter er det at et 3D univers skal forklares i 2D.

Ja, det er en svaghed ved ballon-analogien.

> Det der, efter min mening, bliver problemet er at der bliver for mange
> ubekendter. Og det derfor kun er teoretiske forklaringer der bliver
> brugt. Og teorier det vil tage tusinder, mill., eller mia. år at
bevise.
> Mao. at de ikke kan bevises/afvises i vores levetid.

Ballon-analogien er ikke en teori. Det er big bang derimod, og der er
indsamlet masser af målinger og observationer, der understøtter den (se
mange tidligere diskussioner i gruppen her).

> At det er teorier der fortæller alderen (og udvidelsen og andet). Men
> teorier der først kan bevises/afvises når vi ikke er her mere. (Evt.
> endda som menneskehed)

Se ovenstående bemærkning.


ML-78


Finn Guldmann (25-10-2004)
Kommentar
Fra : Finn Guldmann


Dato : 25-10-04 19:49

ML-78 wrote:

> Det lader til, at du ikke er helt med på forskellen mellem den
> videnskabelige teori og så en populær fremstilling, der skal illustrere
> teorien for lægfolk. Den ballon-analogi vi taler om, er ikke en
> videnskabelig model, som eksperterne har fremsat som forklaring.
>
Jamen betyder det at "ballonnen" er "populærfortolkningen"?

I såfald er det en meget dårlig fortolkning, da den imho så kun gør
tingene værrere.

>>Det der konflikter er det at et 3D univers skal forklares i 2D.
> Ja, det er en svaghed ved ballon-analogien.
>
Men hvis den så ikke er teoretikernes egen måde at forklare det på,
hvordan forklarer de det så?

(I mit eget erhverv holder jeg mig til den faktiske lovtekst i stedet
for det virvar af populærfortolkninger der gives om samme. Måske jeg vil
have det på samme måde med det her)

> Ballon-analogien er ikke en teori. Det er big bang derimod, og der er
> indsamlet masser af målinger og observationer, der understøtter den (se
> mange tidligere diskussioner i gruppen her).
>
Det er endnu ikke lykkedes for mig at finde BB-teorien som tekst der er
skrevet af den eller de der først har fremsat teorien.

Jeg kan finde noget der siger at Hubble regnes for "faderen" til
BB-teorien. Men kan ikke finde en tekst med den der angivelig skulle
komme fra ham. Måske er det endda kun på baggrund af hans observationer
at andre har formuleret teorien. Men i såfald hvem er de så?

--
MVH Finn
To, på hinnanden følgende, undertekster i en udsendelse om
operasangere; "Jeg laver selv min mad" "og beder ofte til Gud"

Henning Makholm (25-10-2004)
Kommentar
Fra : Henning Makholm


Dato : 25-10-04 21:14

Scripsit Finn Guldmann <n@a.invalid>

> Jamen betyder det at "ballonnen" er "populærfortolkningen"?

Ja.

> I såfald er det en meget dårlig fortolkning, da den imho så kun gør
> tingene værrere.

Ja.

> Men hvis den så ikke er teoretikernes egen måde at forklare det på,
> hvordan forklarer de det så?

Ved hjælp af differentialgeometri, firedimensionale mangfoldigheder og
en masse matematik som det kræver mindst et par måneders koncentreret
studium at sætte sig ind i.

Hvis du vil bruge de par måneder (eller mere, alt efter hvor
omfattende dine matematiske forudsætninger er), er det helt fint. Det
kan bare ikke rigtig gøres over usenet; der er for megen information
at absorbere. Derimod kan jeg kan personligt anbefale bogen

Wolfang Rindler: _Relativity - Special, General, and Cosmological_

Den brugte jeg selv til at arbejde mig ud af den situation du nu er i
- jeg fandt det ikke tilfredsstillende at nøjes med de upræcise
populære fremstillinger og ville hellere have den rå model.

Hvis du er villig til at bruge den indsats et selvstudium kræver, er
jeg sikker på at du kan få gode svar på på konkrete spørgsmål i dit
arbejde med en passende lærebog her i gruppen.

> (I mit eget erhverv holder jeg mig til den faktiske lovtekst i stedet
> for det virvar af populærfortolkninger der gives om samme. Måske jeg
> vil have det på samme måde med det her)

Muligt.

> Det er endnu ikke lykkedes for mig at finde BB-teorien som tekst der
> er skrevet af den eller de der først har fremsat teorien.

Hvis du går til primærkilderne lige med det samme, ender du med at
skulle rede dig ud af de historiske misforståelser og tilbageviste
hypoteser i samme takt som teoretikerne selv. Brug hellere en god
lærebog der forklarer teorien ordentligt og systematisk.

--
Henning Makholm "Larry wants to replicate all the time ... ah, no,
all I meant was that he likes to have a bang everywhere."

Per Rønne (25-10-2004)
Kommentar
Fra : Per Rønne


Dato : 25-10-04 21:36

Henning Makholm <henning@makholm.net> wrote:

> Hvis du vil bruge de par måneder (eller mere, alt efter hvor
> omfattende dine matematiske forudsætninger er), er det helt fint.

Jeg tror at det vil tage længere tid for ham. Af det han skriver så kan
man udlede at han er jurist af uddannelse. Og når han ikke kender til
Newtons mekanik, så har han hverken gået i 3. real eller i matematisk
linie. Så mit gæt er: sproglig student, direkte i 1g efter 2. real eller
9. klasse.

I 80ernes begyndelse tog et gymnasialt suppleringskursus til
matematisk-fysisk niveau, matematik, fysik og kemi, et fuldt skoleår på
en gymnasieskole. Og det er så nok til at have forudsætningerne til at
bruge de to måneder på ekstra matematik, som du foreslår.

Til gengæld er han så naturligvis bedre til latin, tysk og fransk end vi
andre er ...
--
Per Erik Rønne

Finn Guldmann (25-10-2004)
Kommentar
Fra : Finn Guldmann


Dato : 25-10-04 22:38

Per Rønne wrote:

>>Hvis du vil bruge de par måneder (eller mere, alt efter hvor
>>omfattende dine matematiske forudsætninger er), er det helt fint.
> Jeg tror at det vil tage længere tid for ham. Af det han skriver så kan
> man udlede at han er jurist af uddannelse. Og når han ikke kender til
> Newtons mekanik, så har han hverken gået i 3. real eller i matematisk
> linie. Så mit gæt er: sproglig student, direkte i 1g efter 2. real eller
> 9. klasse.
>
Takker for tilliden.

Men jeg må skuffe dig.

Det blev "kun" til en 9. klasse, efterfulgt af en håndværksmæssig
uddannelse, der hurtigst muligt blev afløst af indtagelse af pladsen bag
et lastbilrat. Hvilket jeg så har levet af de sidste ca. 25 år.

> I 80ernes begyndelse tog et gymnasialt suppleringskursus til
> matematisk-fysisk niveau, matematik, fysik og kemi, et fuldt skoleår på
> en gymnasieskole. Og det er så nok til at have forudsætningerne til at
> bruge de to måneder på ekstra matematik, som du foreslår.
>
> Til gengæld er han så naturligvis bedre til latin, tysk og fransk end vi
> andre er ...
>
Kun hvis dit latin er lig nul, dit tyske kun rækker til hvad der er
nødvendig for at kunne få læsset og aflæsset en lastbil og dit franske
kun omfatter nogle få ord.

--
MVH Finn
To, på hinnanden følgende, undertekster i en udsendelse om
operasangere; "Jeg laver selv min mad" "og beder ofte til Gud"

Finn Guldmann (25-10-2004)
Kommentar
Fra : Finn Guldmann


Dato : 25-10-04 22:13

Henning Makholm wrote:


>>Men hvis den så ikke er teoretikernes egen måde at forklare det på,
>>hvordan forklarer de det så?
> Ved hjælp af differentialgeometri, firedimensionale mangfoldigheder og
> en masse matematik som det kræver mindst et par måneders koncentreret
> studium at sætte sig ind i.
>
Dem er jeg stødt på.

Mon de virkelig talte sammen på den måde den gang?

> Hvis du vil bruge de par måneder (eller mere, alt efter hvor
> omfattende dine matematiske forudsætninger er), er det helt fint. Det
> kan bare ikke rigtig gøres over usenet; der er for megen information
> at absorbere. Derimod kan jeg kan personligt anbefale bogen
>
> Wolfang Rindler: _Relativity - Special, General, and Cosmological_
>
> Den brugte jeg selv til at arbejde mig ud af den situation du nu er i
> - jeg fandt det ikke tilfredsstillende at nøjes med de upræcise
> populære fremstillinger og ville hellere have den rå model.
>
> Hvis du er villig til at bruge den indsats et selvstudium kræver, er
> jeg sikker på at du kan få gode svar på på konkrete spørgsmål i dit
> arbejde med en passende lærebog her i gruppen.
>
Jeg tror jeg vender tilbage til at søge svar på mit oprindelige
spørgsmål. Det syntes lidt mere overskuelig.

>>Det er endnu ikke lykkedes for mig at finde BB-teorien som tekst der
>>er skrevet af den eller de der først har fremsat teorien.
> Hvis du går til primærkilderne lige med det samme, ender du med at
> skulle rede dig ud af de historiske misforståelser og tilbageviste
> hypoteser i samme takt som teoretikerne selv. Brug hellere en god
> lærebog der forklarer teorien ordentligt og systematisk.
>
Findes en sådan bog uden al matematikken?

--
MVH Finn
To, på hinnanden følgende, undertekster i en udsendelse om
operasangere; "Jeg laver selv min mad" "og beder ofte til Gud"

Henning Makholm (25-10-2004)
Kommentar
Fra : Henning Makholm


Dato : 25-10-04 23:37

Scripsit Henning Makholm <henning@makholm.net>
> Scripsit Finn Guldmann <n@a.invalid>

> > Det er endnu ikke lykkedes for mig at finde BB-teorien som tekst der
> > er skrevet af den eller de der først har fremsat teorien.

> Hvis du går til primærkilderne lige med det samme, ender du med at
> skulle rede dig ud af de historiske misforståelser og tilbageviste
> hypoteser i samme takt som teoretikerne selv.

Hm, måske er det du i virkeligheden efterlyser mindre langhåret end
jeg er gået ud fra. Her er et forsøg på at svare på: Hvad er egentlig
indholdet af "big bang-teorien"?

Først lidt baggrund. Giordano Bruno var den første der seriøst
foreslog at de synlige stjerner er sole ganske som vor egen, som blot
er så utrolig langt væk at de ser meget små ud. Bruno blev brændt som
kætter i 1600 (han havde kontroversielle ideer om mange andre ting end
astronomi), men hans hypotese om stjernernes natur var accepteret af
toneangivende naturvidenskabsmænd blot et århundrede senere (og har
været det lige siden).

Isaac Newton var en af dem, og han formulerede en model for hvordan
universet er indrettet i stor skala: Det er uendelig stort i alle
retninger, og jævnt fordelt i det findes stjerner som ikke bevæger sig
i forhold til hinanden. Ifølge Newton har universet enten eksisteret
uendelig længe eller (hvis man foretrækker en guddommelig skaberakt på
et bestemt tidspunkt) det er blevet skabt med stjernerne i samme
indbyrdes positioner som de vil have indtil dommedag.

Newtons model var ikke ganske uden interne paradokser (når man
inddrager Newtons egen tyngdelov i den), men i flere hundrede år det
var den eneste forhåndenværende teori, så det var den folk arbejdede
efter. I 1917 indførte Einstein sin berømte "kosmologiske konstant"
netop for at få sin almindelige relativitetsteori til at stemme med
Newtons universmodel.

Takket være bl.a. Hubbles observationer blev det i løbet af 1920'erne
klart at universets stjerner er samlet i galakser som er endnu
utroligere langt fra hinanden end stjernerne selv. Men Newtons model
holdt stadig, blot man erstattede "stjerner" med "galakser".

Så i 1929 annoncerede Hubble resultaterne af en række observationer
af spektre fra galakser som han (med andre metoder) havde vurderet
afstanden til. Jo fjernere en galakse er, desto mere rødforskudt er
dens lysspektrum. Mere præcist er det lys vi ser fra en galakse
rødforskudt netop så meget som det ville være hvis afstanden mellem os
og galaksen voksede med en hastighed proportionalt med afstanden til
galaksen. Eftersom der ikke rigtig er andre mulige kilder til sådan en
generel rødforskydning, konkluderede Hubble selv at galakserne
*faktisk* bevæger sig bort fra os - eller i det mindste gjorde det på
det tidspunkt det lys vi ser, blev udsendt.

Hubbles opdagelse tog livet af Newtons universmodel - galaksernes
afstande fra hinanden er *ikke* konstant. Spørgsmålet var hvad man
skulle sætte i stedet, som stemte med Hubbles afstandslov. Big bang er
en af 3-4 mulige teorier der er blevet fremsat:

1. STEADY STATE-teorien siger at universet har eksisteret uendelig
længe, og at det altid har udvidet sig. Nærmere bestemt har
forholdet mellem to galaksers indbyrdes afstand og den hastighed
afstanden vokser med, altid været den samme som nu (med tiden
*vokser* de to galaksers indbyrdes hastighed altså). Derudover har
det gennemsnitlige antal galakser pr. rumenhed også været den samme
altid - efterhånden som afstanden mellem galakserne vokser, opstår
der nye galakser mellem dem (ud af intet!) med netop den rette
hastighed.

2. DET OSCILLERENDE UNIVERS. Galakser kan ikke opstå af intet. For
lang tid siden sådan at galakserne bevægede sig *tættere* på
hinanden. Men de havde ikke præcis retning mod hinanden, så de
ramte lidt forbi. Den gennemsnitlige afstand mellem nabogalakser
faldt til et meget lille minimum, og galakserne begyndte at bevæge
sig væk fra hinanden igen; det er den fase vi er i nu. Også det
oscillerende univers har eksisteret uendelig lang tid; muligvis er
det en med jævne mellemrum forekommende hændelse at alle galakser
på grund af deres indbyrdes tiltrækning begynder at falde mod
hinanden, men rammer ved siden af og giver anledning til en ny
udvidelse.

3. BIG BANG. Galakser kan ikke opstå ud af intet midt mens universet
udfolder sig omkring dem. Tidligere i universets historie har der
derfor været kortere mellem galakserne end der er nu, og hvis vi
går langt nok tilbage bliver den gennemsnitlige afstand mellem to
vilkårlige galakser til 0. På det tidspunkt har universets tæthed
været uendelig høj, og afstanden mellem ethvert par af punkter i
universet 0. Under de betingelser bryder alle kendte naturlove
sammen; vi kan derfor ikke sige noget om hvad der skete netop i
0-øjeblikket, eller hvad der gik forud for det, i det omfang det
overhovedet giver mening at noget er gået forud for 0-øjeblikket.

(Navnet "big bang", som i dag er en neutral betegnelse for
0-øjeblikket, er i øvrigt oprindelig et forsøg fra teoriens
modstandere på at give den et latterliggørende navn).

4. GUD GJORDE DET. Af årsager som Han ikke har indviet os i, er
universet skabt i en tilstand hvor galakserne allerede var langt
fra hinanden *og* allerede havde en jævn hastighed væk fra
hinanden. Før skabelsesøjeblikket var hverken galakser eller tid
til, og der har derfor hverken været noget big bang eller et "near
miss" som i det oscillerende univers.

Bemærk at *alle* disse teorier - borset, måske, fra (4) - forudsætter
at universet i øjeblikket udvider sig jævnt, men alligevel uden at
have noget centrum. Denne egenskab (som er hvad ballonmodellen
forsøger at anskueliggøre, med større eller mindre held) er altså ikke
specifik for Big Bang-teorien, men gælder også for dens alternativer.

Model (4) kan i princippet ikke afvises af nogen observationer; det
gør den videnskabeligt uinteressant, og vi kan derfor se bort fra
den. For at kunne vælge mellem de 3 første modeller er det nødvendigt
at gå ud fra en underliggende teori for hvordan tyngdekraften mellem
galakserne indvirker på deres indbyrdes bevægelse. Denne teori er
Einsteins almindelige relativitetsteori (GR), med et lille antal
mulige varianter. [1]

Relativitetsteorien gjorde det med tiden af med Steady State-teorien.
Kombinationen af GR og Steady State gav anledning til forudsigelser om
sammenhængen mellem bl.a. Hubblekonstanten, tyngdekraftens styrke og
den gennemsnitlige tæthed af galakser i universet, som ikke passede
med observationerne. Det var en god teori; den var filosofisk
tiltrækkende og producerede præcise forudsigelser. De passede bare
ikke.

Det viste sig også (Penrose og Hawking) at relativitetsteorien ikke
tillader et oscillerende univers - det kan ifølge GR ikke lade sig
gøre for tunge galakser at ramme forbi hinanden på den måde den
oscillerende teori påstår.

Tilbage er så Big Bang-teorien. Det eneste *den* siger er at universet
er opstået på et konkret tidspunkt, og at afstandene i det har udvidet
sig siden. Den præcise udviklingshistorie for hvor hurtigt afstandene
er vokset hvornår er ikke en del af det blotte begreb om big bang. Man
kan derfor nærmere sige at big bang er en fælles ramme for en hel
række af forskellige mulige teorier som har til fælles at afstandene
er vokset fra 0 i big bang-øjeblikket til afstandende i dag.

[1] Man kan naturligvis postulere at tyngdekraften opfører sig på en
helt speciel måde på meget store afstande, som ikke behøver at
være konsistent med den måde vi ser tyngdekraften virke i
solsystemet og vores lokale galakse. Så kan man få sin
kosmologiske model til at gøre præcis hvad man har lyst til; men
det gør i almindelighed den tilgang uinteressant af samme grund
som "Gud gjorde det" ikke er en videnskabeligt relevant teori,
hvor rigtig den end kan tænkes at være på et metafysisk plan.

--
Henning Makholm "The practical reason for continuing our
system is the same as the practical reason
for continuing anything: It works satisfactorily."

Finn Guldmann (26-10-2004)
Kommentar
Fra : Finn Guldmann


Dato : 26-10-04 03:53

Henning Makholm wrote:

> Hm, måske er det du i virkeligheden efterlyser mindre langhåret end
> jeg er gået ud fra. Her er et forsøg på at svare på: Hvad er egentlig
> indholdet af "big bang-teorien"?
>
<cut>
Manner tak for en forklaring på et niveau hvor jeg også kunne være med.

Det meste af det du skriver er jeg allerede stødt på. Men som fragmenter
af noget andet. Ikke pænt sammenhængende og fyldestgørende som her.

Og som det nok mest herlige lukker det ikke af for at nogle af de
opfattelser jeg har af tingene godt kan have noget på sig.

--
MVH Finn
To, på hinnanden følgende, undertekster i en udsendelse om
operasangere; "Jeg laver selv min mad" "og beder ofte til Gud"

Claus Christiansen (25-10-2004)
Kommentar
Fra : Claus Christiansen


Dato : 25-10-04 16:01

Finn Guldmann <n@a.invalid> wrote in news:clitpc$1tdn$1
@news.cybercity.dk:

> Det der konflikter er det at et 3D univers skal forklares i 2D.
>
Det skal det jo heller ikke! Der er tale om en *forsimpling*. Du kunne
ligeså godt kværulere over at vores univers ikke er lavet af gummi...


> Det der, efter min mening, bliver problemet er at der bliver for mange
> ubekendter.
>
Såsom?


> Og det derfor kun er teoretiske forklaringer der bliver
> brugt. Og teorier det vil tage tusinder, mill., eller mia. år
> at bevise. Mao. at de ikke kan bevises/afvises i vores levetid.
>
Forkert. Det kræver 'kun' en ny model med et eksperiment der kan
eftervises og som strider mod en nuværende model; i så fald vil BB kunne
afvises.

Iøvrigt 'beviser' man kun ting i logik og matematik. I fysik er det
relevant om en model (og dens forudsigelser) stemmer overens med
eksperimentielle forsøg.


> At det er teorier der fortæller alderen (og udvidelsen og andet).
>
Tja, hvad andet forestiller du dig?? Et skilt sat op af en venlig guddom.


Finn Guldmann (25-10-2004)
Kommentar
Fra : Finn Guldmann


Dato : 25-10-04 20:13

Claus Christiansen wrote:

>>Det der konflikter er det at et 3D univers skal forklares i 2D.
> Det skal det jo heller ikke! Der er tale om en *forsimpling*. Du kunne
> ligeså godt kværulere over at vores univers ikke er lavet af gummi...
>
Men hvorfor denne forsimpling hvis den blot gør tingene sværere at fatte?

>>Det der, efter min mening, bliver problemet er at der bliver for mange
>>ubekendter.
> Såsom?
>
At teorier begynder at bygge på andre teorier, der endnu ikke er bevist.

>>Og det derfor kun er teoretiske forklaringer der bliver
>>brugt. Og teorier det vil tage tusinder, mill., eller mia. år
>>at bevise. Mao. at de ikke kan bevises/afvises i vores levetid.
> Forkert. Det kræver 'kun' en ny model med et eksperiment der kan
> eftervises og som strider mod en nuværende model; i så fald vil BB kunne
> afvises.
>
Og hvilke eksperimenter skulle det så være?

Det de har gang i nede på CERN vil, såvidt jeg har forstået det, jo ikke
kunne bruges til at be- eller afkræfte BB.

Opnår de det resultat de søger vil de kunne påvise at den tilstand
teorien siger har været indenfor det første sekund efter BB kan lade sig
gøre. Men ikke at den faktisk har været der.

Og modsat; Hvis de ikke opnår at skabe "ursuppen" kan de jo ikke bruge
det til at afgøre at BB ikke har fundet sted. For så kan det jo være
fordi deres forsøg ikke er istand til at skabe de rigtige forhold



> Iøvrigt 'beviser' man kun ting i logik og matematik. I fysik er det
> relevant om en model (og dens forudsigelser) stemmer overens med
> eksperimentielle forsøg.
>
Ville det ikke være mere relevant om modellen blev undersøgt for om den
stemte overens med "det virkelige liv"?

>>At det er teorier der fortæller alderen (og udvidelsen og andet).
> Tja, hvad andet forestiller du dig?? Et skilt sat op af en venlig guddom.
>
Egentlig forventer jeg ikke noget.

Men det ville da være rart om det, i det mindste, ville være muligt at
skelne mellem det der udelukkende hviler på en teori og det der hviler
på fysiske fakta.

F.eks. ved at opdele astronomien i to dele; Den teoretiske og den
praktiske. Og så lade det fremstå klart hvor grænsen gik på et givent
tidspunkt.

Lige nu henstår jeg i en situation hvor jeg er forvirret over hvad der
er teorier og hvad der er fysiske fakta. Men også over hvad der er
teoretikerne og praktikernes konkrete teorier og resultater, og hvad der
er "udenforståendes" populærfortolkninger.

Og jeg kan ikke lige opdage hvordan jeg ser mig ud af det. (Det er
derfor jeg spørger efter links f.eks. til hvor jeg finder den
oprindelige formulering af BB-teorien. (Og gerne uden for mange
matematiske ligninger (Dem fatter jeg aligevel ikke en lyd af)))

--
MVH Finn
To, på hinnanden følgende, undertekster i en udsendelse om
operasangere; "Jeg laver selv min mad" "og beder ofte til Gud"

Henning Makholm (25-10-2004)
Kommentar
Fra : Henning Makholm


Dato : 25-10-04 21:18

Scripsit Finn Guldmann <n@a.invalid>

> > Det skal det jo heller ikke! Der er tale om en *forsimpling*. Du
> > kunne ligeså godt kværulere over at vores univers ikke er lavet af
> > gummi...

> Men hvorfor denne forsimpling hvis den blot gør tingene sværere at fatte?

Nogen mener at den gør tingene *lettere* at fatte. Det er helt tilladt
at have forskellige vurderinger af hvor godt den slags hjælpemidler
virker.

> > Forkert. Det kræver 'kun' en ny model med et eksperiment der kan
> > eftervises og som strider mod en nuværende model; i så fald vil BB
> > kunne afvises.

> Og hvilke eksperimenter skulle det så være?

Det kommer helt an på hvordan den nye model afviger fra den gældende.
Men der vil sandsynligvis være tale om astronomiske observationer.

> > Iøvrigt 'beviser' man kun ting i logik og matematik. I fysik er det
> > relevant om en model (og dens forudsigelser) stemmer overens med
> > eksperimentielle forsøg.

> Ville det ikke være mere relevant om modellen blev undersøgt for om
> den stemte overens med "det virkelige liv"?

Det er jo det man gør ved eksperimenter.

> Men det ville da være rart om det, i det mindste, ville være muligt at
> skelne mellem det der udelukkende hviler på en teori og det der hviler
> på fysiske fakta.

Hvilken slags "fysiske fakta" der ikke "udelukkende hviler på en
teori" forestiller du dig?

--
Henning Makholm "Er det det med pingvinen?"

Finn Guldmann (25-10-2004)
Kommentar
Fra : Finn Guldmann


Dato : 25-10-04 22:17

Henning Makholm wrote:

>>Men det ville da være rart om det, i det mindste, ville være muligt at
>>skelne mellem det der udelukkende hviler på en teori og det der hviler
>>på fysiske fakta.
> Hvilken slags "fysiske fakta" der ikke "udelukkende hviler på en
> teori" forestiller du dig?
>
F.eks. i stedet for at teoretisere om hvordan universet udvider sig, så
foretage faktiske målinger af hvor en galakse bevæger sig hen og med
hvilken hastighed.

Dobblereffekten er vist så veldokumenteret at den ikke kan kaldes teori
mere. Og det er vist den man bruger til målingerne.

--
MVH Finn
To, på hinnanden følgende, undertekster i en udsendelse om
operasangere; "Jeg laver selv min mad" "og beder ofte til Gud"

Henning Makholm (26-10-2004)
Kommentar
Fra : Henning Makholm


Dato : 26-10-04 00:28

Scripsit Finn Guldmann <n@a.invalid>
> Henning Makholm wrote:

> >>Men det ville da være rart om det, i det mindste, ville være muligt at
> >>skelne mellem det der udelukkende hviler på en teori og det der hviler
> >>på fysiske fakta.

> > Hvilken slags "fysiske fakta" der ikke "udelukkende hviler på en
> > teori" forestiller du dig?

> F.eks. i stedet for at teoretisere om hvordan universet udvider sig,
> så foretage faktiske målinger af hvor en galakse bevæger sig hen og
> med hvilken hastighed.

Det er jo det man har gjort!

--
Henning Makholm "Det nytter ikke at flygte
der er henna overalt"

Finn Guldmann (26-10-2004)
Kommentar
Fra : Finn Guldmann


Dato : 26-10-04 03:57

Henning Makholm wrote:

>>>Hvilken slags "fysiske fakta" der ikke "udelukkende hviler på en
>>>teori" forestiller du dig?
>>F.eks. i stedet for at teoretisere om hvordan universet udvider sig,
>>så foretage faktiske målinger af hvor en galakse bevæger sig hen og
>>med hvilken hastighed.
> Det er jo det man har gjort!
>
Og hvad kom man så til? At de alle bevæger sig ret væk fra vores del af
universet? Eller at de er "ude af kurs" for vores synsvinkel?

--
MVH Finn
To, på hinnanden følgende, undertekster i en udsendelse om
operasangere; "Jeg laver selv min mad" "og beder ofte til Gud"

ML-78 (26-10-2004)
Kommentar
Fra : ML-78


Dato : 26-10-04 11:47

Finn Guldmann skrev:

> Og hvad kom man så til? At de alle bevæger sig ret væk fra vores del
af
> universet? Eller at de er "ude af kurs" for vores synsvinkel?

At de alle bevæger sig ret væk fra vores del af universet. Som nævnt
tidligere betyder det ikke, at vi befinder sig i centrum, da det samme
vil gøre sig gældende for enhver iagttager.


ML-78


Finn Guldmann (26-10-2004)
Kommentar
Fra : Finn Guldmann


Dato : 26-10-04 14:37

ML-78 wrote:

>>Og hvad kom man så til? At de alle bevæger sig ret væk fra vores del
>>af universet? Eller at de er "ude af kurs" for vores synsvinkel?
> At de alle bevæger sig ret væk fra vores del af universet. Som nævnt
> tidligere betyder det ikke, at vi befinder sig i centrum, da det samme
> vil gøre sig gældende for enhver iagttager.
>
Så begynder det at blive virkelig sjovt at forklare hvis vi blander
flere galakser og flere observatørere ind i det.

F.eks. at observatør a står i galakse A og b i B. De betragter begge to
galakse C, som ligge mellem dem.
For a er synsvinklen mellem B og C 45 grd. For b er vinklen mellem A og
C også 45 grd.

Som jeg forstår det du skriver vil C bevæge sig i en ret linie væk fra
A, samtidig med at den bevæger sig væk fra B i en ret linie. Hvilket må
blive noget med 90 grd. på tværs af vores synsretning.

Jeg kan ikke lige se hvordan det lader sig praktisere.

--
MVH Finn
To, på hinnanden følgende, undertekster i en udsendelse om
operasangere; "Jeg laver selv min mad" "og beder ofte til Gud"

Henning Makholm (26-10-2004)
Kommentar
Fra : Henning Makholm


Dato : 26-10-04 16:19

Scripsit Finn Guldmann <n@a.invalid>

> F.eks. at observatør a står i galakse A og b i B. De betragter begge
> to galakse C, som ligge mellem dem.
> For a er synsvinklen mellem B og C 45 grd. For b er vinklen mellem A
> og C også 45 grd.

> Som jeg forstår det du skriver vil C bevæge sig i en ret linie væk fra
> A, samtidig med at den bevæger sig væk fra B i en ret linie.

Ja. Alle afstande i trekanten ABC vokser med tiden i samme forhold.
Derfor er vinklerne i trekanten også bevaret.

> Hvilket må blive noget med 90 grd. på tværs af vores synsretning.

Nej for "væk fra B i en ret linje" er målt i et koordinatsystem der
følger med B's egen bevægelse. Det er ikke ensbetydende med at
bevægelsen peger direkte væk fra B i et andet koordinatsystem (fx As).

--
Henning Makholm "What has it got in its pocketses?"

Finn Guldmann (27-10-2004)
Kommentar
Fra : Finn Guldmann


Dato : 27-10-04 09:30

Henning Makholm wrote:

>>F.eks. at observatør a står i galakse A og b i B. De betragter begge
>>to galakse C, som ligge mellem dem.
>>For a er synsvinklen mellem B og C 45 grd. For b er vinklen mellem A
>>og C også 45 grd.
>>Som jeg forstår det du skriver vil C bevæge sig i en ret linie væk fra
>>A, samtidig med at den bevæger sig væk fra B i en ret linie.
> Ja. Alle afstande i trekanten ABC vokser med tiden i samme forhold.
> Derfor er vinklerne i trekanten også bevaret.
>>Hvilket må blive noget med 90 grd. på tværs af vores synsretning.
> Nej for "væk fra B i en ret linje" er målt i et koordinatsystem der
> følger med B's egen bevægelse. Det er ikke ensbetydende med at
> bevægelsen peger direkte væk fra B i et andet koordinatsystem (fx As).
>
Jeg får så et problem med hvilken kraft der kan få det til at ske. Jeg
kan ikke få det til at harmonere med BB.

--
MVH Finn
To, på hinnanden følgende, undertekster i en udsendelse om
operasangere; "Jeg laver selv min mad" "og beder ofte til Gud"

Henning Makholm (27-10-2004)
Kommentar
Fra : Henning Makholm


Dato : 27-10-04 15:06

Scripsit Finn Guldmann <n@a.invalid>
> Henning Makholm wrote:

> >>Hvilket må blive noget med 90 grd. på tværs af vores synsretning.
> > Nej for "væk fra B i en ret linje" er målt i et koordinatsystem der
> > følger med B's egen bevægelse. Det er ikke ensbetydende med at
> > bevægelsen peger direkte væk fra B i et andet koordinatsystem (fx As).

> Jeg får så et problem med hvilken kraft der kan få det til at ske.

Der skal ikke nogen kraft til. Ingen hastigheder ændrer sig.

--
Henning Makholm "Manden med det store pindsvin er
kommet vel ombord i den grønne dobbeltdækker."

ML-78 (26-10-2004)
Kommentar
Fra : ML-78


Dato : 26-10-04 16:32

Finn Guldmann skrev:

> Som jeg forstår det du skriver vil C bevæge sig i en ret linie væk fra
> A, samtidig med at den bevæger sig væk fra B i en ret linie. Hvilket

> blive noget med 90 grd. på tværs af vores synsretning.
>
> Jeg kan ikke lige se hvordan det lader sig praktisere.

Her kommer ballon-analogien ind i billedet. Prøv at tegne den trekant du
beskriver på overfladen og pust så ballonen yderligere op.


ML-78


Finn Guldmann (27-10-2004)
Kommentar
Fra : Finn Guldmann


Dato : 27-10-04 09:35

ML-78 wrote:

>>Som jeg forstår det du skriver vil C bevæge sig i en ret linie væk fra
>>A, samtidig med at den bevæger sig væk fra B i en ret linie. Hvilket
>>må
>>blive noget med 90 grd. på tværs af vores synsretning.
>>Jeg kan ikke lige se hvordan det lader sig praktisere.
> Her kommer ballon-analogien ind i billedet. Prøv at tegne den trekant du
> beskriver på overfladen og pust så ballonen yderligere op.
>
Jeg kan godt følge hvad du mener, men jeg får et problem når jeg vil
"overføre" det til det univers jeg kan se.

--
MVH Finn
To, på hinnanden følgende, undertekster i en udsendelse om
operasangere; "Jeg laver selv min mad" "og beder ofte til Gud"

Carsten Svaneborg (26-10-2004)
Kommentar
Fra : Carsten Svaneborg


Dato : 26-10-04 16:10

Finn Guldmann wrote:
> For a er synsvinklen mellem B og C 45 grd. For b er vinklen mellem A og
> C også 45 grd.
> Som jeg forstår det du skriver vil C bevæge sig i en ret linie væk fra
> A, samtidig med at den bevæger sig væk fra B i en ret linie. Hvilket må
> blive noget med 90 grd. på tværs af vores synsretning.

Husk at afstanden fra A og B også vokser.

Hvis vi ser det i 2D fra A's perspektiv: A = (0,0) B=(2,0) og C=(1,1)

Som tiden går vil ekspansionen betyde at B=(2s,0) mens C=(s,s) hvor
s er skalaen der vokser.

Hvis vi ser det i 2D fra B's perspektiv: A = (-2,0) B=(0,0) C=(-1,1)
Som tiden går vil ekspansionen betyder at A=(-2s,0) mens C=(-s,s).

Der er således ikke noget problem, alt bevæger sig væk fra alt, og
vinklerne forbliver de samme. Prøv at tegne det på et stykke papir,
f.eks. med s=2.

--
Mvh. Carsten Svaneborg
http://www.softwarepatenter.dk

Finn Guldmann (27-10-2004)
Kommentar
Fra : Finn Guldmann


Dato : 27-10-04 09:25

Carsten Svaneborg wrote:

>>For a er synsvinklen mellem B og C 45 grd. For b er vinklen mellem A og
>>C også 45 grd.
>>Som jeg forstår det du skriver vil C bevæge sig i en ret linie væk fra
>>A, samtidig med at den bevæger sig væk fra B i en ret linie. Hvilket må
>>blive noget med 90 grd. på tværs af vores synsretning.
> Husk at afstanden fra A og B også vokser.
> Hvis vi ser det i 2D fra A's perspektiv: A = (0,0) B=(2,0) og C=(1,1)
> Som tiden går vil ekspansionen betyde at B=(2s,0) mens C=(s,s) hvor
> s er skalaen der vokser.
> Hvis vi ser det i 2D fra B's perspektiv: A = (-2,0) B=(0,0) C=(-1,1)
> Som tiden går vil ekspansionen betyder at A=(-2s,0) mens C=(-s,s).
> Der er således ikke noget problem, alt bevæger sig væk fra alt, og
> vinklerne forbliver de samme. Prøv at tegne det på et stykke papir,
> f.eks. med s=2.
>
Til mig at se må et eller flere elementer i beregningen lave
"krumspring" for at få regnestykket til at gå op. Og det bliver ikke
bedre af at der kommer flere elementer ind i beregningen.

Men for at det holder vand bliver man, til mig at se, også nødt til at
droppe BB og "opfinde" en kraft der der opstår over alt og som presser
elementerne fra hinnanden, lige meget i alle retninger. F.eks. ved at
udfylde det ekstra rum der opstår mellem elementerne.

--
MVH Finn
To, på hinnanden følgende, undertekster i en udsendelse om
operasangere; "Jeg laver selv min mad" "og beder ofte til Gud"

Carsten Svaneborg (27-10-2004)
Kommentar
Fra : Carsten Svaneborg


Dato : 27-10-04 12:11

Finn Guldmann wrote:
> Men for at det holder vand bliver man, til mig at se, også nødt til at
> droppe BB og "opfinde" en kraft der der opstår over alt og som presser
> elementerne fra hinnanden, lige meget i alle retninger.

Nej. Vi /observerer/ at alt bevæger sig fra hinanden og denne effekt
er accelerende. Formålet med kosmologi er at beskrive observationerne,
og man bliver derfor nødt til at introducerer en kosmologisk konstant
for at kunne beskrive denne acceleration.

Jeg tror du forveksler teori med observation.

--
Mvh. Carsten Svaneborg
http://www.softwarepatenter.dk

Finn Guldmann (27-10-2004)
Kommentar
Fra : Finn Guldmann


Dato : 27-10-04 22:24

Carsten Svaneborg wrote:

>>Men for at det holder vand bliver man, til mig at se, også nødt til at
>>droppe BB og "opfinde" en kraft der der opstår over alt og som presser
>>elementerne fra hinnanden, lige meget i alle retninger.
> Nej. Vi /observerer/ at alt bevæger sig fra hinanden og denne effekt
> er accelerende. Formålet med kosmologi er at beskrive observationerne,
> og man bliver derfor nødt til at introducerer en kosmologisk konstant
> for at kunne beskrive denne acceleration.
>
> Jeg tror du forveksler teori med observation.
>
Jeg tænker nu lidt mere praktisk på det.

For at du kan få noget til at accelerere skal du jo have tilført en
kraft der kan skabe denne acceleration.

Den kraft der blev tilført for 13,7 mia. år siden, ved BB, kan jo ikke
skabe acceleration nu.

--
MVH Finn
To, på hinnanden følgende, undertekster i en udsendelse om
operasangere; "Jeg laver selv min mad" "og beder ofte til Gud"

Carsten Svaneborg (27-10-2004)
Kommentar
Fra : Carsten Svaneborg


Dato : 27-10-04 22:49

Finn Guldmann wrote:
> Den kraft der blev tilført for 13,7 mia. år siden, ved BB,
> kan jo ikke skabe acceleration nu.

Hvoraf det følger af det er en egenskab ved universet idag,
og ikke af Big Bang, at vi observere acceleration.

--
Mvh. Carsten Svaneborg
http://www.softwarepatenter.dk

Finn Guldmann (28-10-2004)
Kommentar
Fra : Finn Guldmann


Dato : 28-10-04 00:29

Carsten Svaneborg wrote:

>>Den kraft der blev tilført for 13,7 mia. år siden, ved BB,
>>kan jo ikke skabe acceleration nu.
> Hvoraf det følger af det er en egenskab ved universet idag,
> og ikke af Big Bang, at vi observere acceleration.
>
Det store spørgsmål er så hvor denne kraft kommer fra.

--
MVH Finn
To, på hinnanden følgende, undertekster i en udsendelse om
operasangere; "Jeg laver selv min mad" "og beder ofte til Gud"

Carsten Svaneborg (28-10-2004)
Kommentar
Fra : Carsten Svaneborg


Dato : 28-10-04 12:35

Finn Guldmann wrote:
>> Hvoraf det følger af det er en egenskab ved universet idag,
>> og ikke af Big Bang, at vi observere acceleration.
> Det store spørgsmål er så hvor denne kraft kommer fra.

Enig.

--
Mvh. Carsten Svaneborg
http://www.softwarepatenter.dk

Sven Nielsen (26-10-2004)
Kommentar
Fra : Sven Nielsen


Dato : 26-10-04 19:40

In article <cllk1f$1bio$1@news.cybercity.dk>, n@a.invalid says...

> Som jeg forstår det du skriver vil C bevæge sig i en ret linie væk fra
> A, samtidig med at den bevæger sig væk fra B i en ret linie. Hvilket må
> blive noget med 90 grd. på tværs af vores synsretning.
>
> Jeg kan ikke lige se hvordan det lader sig praktisere.

Det lader ikke til at være big bang teorien, du har et problem med. Det
er såmænd almindelig geometri, du er uenig med. Prøv med passer og
lineal.

Anyways, det vi snakker om, er det, der kaldes det kosmologiske princip.

Det siger, at intet sted i rummet er særlig forskellig fra de andre, samt
at ingen retning i rummet er forskellig fra de andre. Dette er en
antagelse, som passer med det man observerer, når man kigger på
Universet. Det kosmologiske princip er ikke en del af selve big bang
teorien. Det er en grundliggende antagelse bag ved teorien. Samtidig gør
den antagelse teorien meget simpel, idet Universets udvidelse kan
beskrives med et eneste tal, som kaldes skalafaktoren R(t). Det
kosmologiske princip er man nødt til at acceptere, før man overhovedet
kan gå i gang med at studere big bang teorien.

Hvordan er den kosmologiske model som kaldes "standard big bang teorien"
så opbygget? Teorien kan såmænd udledes ved almindelig regnearbejde med
blyant og papir, hvis man bruger flg. ingredienser:

1) Almen relativitetsteori (som Einstein opfandt omkring 1915)
2) Det kosmologiske princip (som nævnt ovenfor)

Ud fra dette kommer hele big bang teorien ved stort set mekanisk
regnearbejde. Selvfølgelig skulle nogen få ideen til det. En af de
første, som fik denne ide og gjorde noget ved det, var den belgiske
astrofysiker (og præst) Lemaître, som arbejdede med dette i årene omkring
1930. Senere har andre astrofysikere puttet mere fysik ind i modellen:

3) Kernefysik (1940'erne)
4) Standardmodellen indenfor partikelfysikken (1970'erne)
5) Mørkt stof og senest mørkt energi.

Med disse tilføjelser har standard big bang teorien fået den form, den
har i dag. Dernæst kan man spørge hvad der er kommet ud af teorien - hvad
er der af beviser for den:

A) Hubbles lov (1929) - det faktum, at fjerne galakser bevæger sig væk
fra os med en hastighed, der er propotional med afstanden til galaksen.

B) Mængden af grundstoffer i det tidlige Univers forudsiges af big bang
teorien (1940'erne). Observationerne passer med teorien.

C) Den kosmiske mikrobølgestråling (opdaget af Penzias og Wilson i 1965,
men forudsagt af Gamow i 1940'erne). Dette betragter langt de fleste
astrofysikere som det endelige bevis for, at der har været et big bang.

Senere er der kommet nye observationer og ny fysik ind (4-5 ovenfor), og
ingen af observationerne er i modstrid i standard big bang. Faktisk har
observationer af især små variationer i den kosmiske mikrøbølgestråling
(også kaldes baggrundstrålingen) og bedre afstandbestemmelser til fjerne
galakser gjort teorien langt mere præcis ved at udelukke mange teoretiske
muligheder i den oprindelige teori. Det har så gjort, at man har kunne
udlede nye forudsigelser, som kan bekræftes. Derfor har man i dag mere
tiltro til teorien, end man nogensinde før har haft.

Alligevel er der lavet en modificeret teori, som kaldes big bang med
inflation. Den giver på nogle områder en bedre forklaring på, hvorfor
Univserset ser ud som det gør. Den oprindelige version af denne teori
tilskrives Alan Guth, som i 1981 fik ideen. I dag er denne teori dog
forældet, og afløst af den nye inflationsteori, som bl.a. den russisk-
amerikanske fysiker Andrei Linde står for. Inflationsteorien går kort
fortalt ud på, at det meget tidlige Univers i løbet af meget kort tid er
blevet "pustet op" til en størrelse, der er mange gange større, end det
ville være ifølge standard big bang teorien. Det synlige Univers har
ifølge standard big bang ikke været mindre end ca. 1/100 millimeter (ved
Planck epoken, som er det tidligste tidpunkt, man kan regne tilbage til
indenfor teorien). Det er jo ikke specielt småt - 100.000 gange større
end små atomer. Men med inflation har det synlige Univers været meget,
meget mindre ved Planck epoken.

Med venlig hilsen Sven.

Per Rønne (26-10-2004)
Kommentar
Fra : Per Rønne


Dato : 26-10-04 20:41

Sven Nielsen <sven@DONT.SPAM.ME.SCIENTIST.COM> wrote:

> En af de første, som fik denne ide og gjorde noget ved det, var den
> belgiske astrofysiker (og præst) Lemaître, som arbejdede med dette i årene
> omkring 1930.

Ja, så er det jo at man spekulerer på, hvad den katolske kirke sagde til
angrebet mod 1. mosebog. Et opslag i Encyclopædia Britannica 2003 giver
dog intet svar herpå:

Lemaître, Georges
born July 17, 1894, Charleroi, Belg.
died June 20, 1966, Louvain


Belgian astronomer and cosmologist who formulated the modern big-bang
theory, which holds that the universe began in a cataclysmic explosion
of a small, primeval "super-atom."

A civil engineer, Lemaître served as an artillery officer in the Belgian
Army during World War I. After the war he entered a seminary and in 1923
was ordained a priest. He studied at the University of Cambridge's solar
physics laboratory (1923-24) and then at the Massachusetts Institute of
Technology, Cambridge (1925-27), where he became acquainted with the
findings of the American astronomers Edwin P. Hubble and Harlow Shapley
on the expanding universe. In 1927, the year he became professor of
astrophysics at the University of Louvain, he proposed his big-bang
theory, which explained the recession of the galaxies within the
framework of Albert Einstein's theory of general relativity. Although
expanding models of the universe had been considered earlier, notably by
the Dutch astronomer Willem de Sitter, Lemaître's theory, as modified
byGeorge Gamow, has become the leading theory of cosmology.

Lemaître also did research on cosmic rays and on the three-body problem,
which concerns the mathematical description of the motion of three
mutually attracting bodies in space. His works include Discussion sur
l'évolution de l'univers (1933; "Discussion on the Evolution of the
Universe") and L'Hypothèse de l'atome primitif (1946; "Hypothesis of the
Primeval Atom").
--
Per Erik Rønne

Finn Guldmann (27-10-2004)
Kommentar
Fra : Finn Guldmann


Dato : 27-10-04 10:28

Sven Nielsen wrote:

> Det lader ikke til at være big bang teorien, du har et problem med. Det
> er såmænd almindelig geometri, du er uenig med. Prøv med passer og
> lineal.
>
<cut (En af de forklaringer det sætter tingene i et andet lys)>

Den med geometrien skal nok passe. Om ikke andet så når den holdes
sammen med fysikken i det.

Der er bl.a. skrevet om at udvidelseshastigheden ændres. Det giver mig
et problem når det handler om at den stiger. Bl.a. med hvilken kraft der
får den til det. (Hvis det altså er tilfældet)

Men så for at vende tilbage til mit oprindelige spørgsmål.

Så udvidelseshastigheden indenfor det første sekund beskrevet som at
universet udvidede sig fra at være på størrelse med et atom til at være
på størrelse med en galakse. Det giver jo en sådan pænt stor
begyndelseshastighed. (Min regnestok er ikke lang nok til at udregne
den, men 50-100.000 lysår på under et sekund)

Så medmindre udvidelseshastigheden altid har været størrere end lysets
hastighed, hvordan kan lys der er udsendt fra et objekt for 13 mia. år
siden så først komme frem til os nu?

--
MVH Finn
To, på hinnanden følgende, undertekster i en udsendelse om
operasangere; "Jeg laver selv min mad" "og beder ofte til Gud"

Sven Nielsen (27-10-2004)
Kommentar
Fra : Sven Nielsen


Dato : 27-10-04 22:40

In article <clnpq1$m0o$1@news.cybercity.dk>, n@a.invalid says...

> Der er bl.a. skrevet om at udvidelseshastigheden ændres. Det giver mig
> et problem når det handler om at den stiger. Bl.a. med hvilken kraft der
> får den til det. (Hvis det altså er tilfældet)

Det er observeret i sidst i 1990'erne, at udvidelsen af Universet går
hurtigere og hurtigere. Dette er altså et faktum, forudsat der ikke er
fejl i observationerne. Så undrer du dig over, hvilken kraft, der
forårsager dette. Jeg kan fortælle dig, at det undrer astronomerne sig
også over. Det vides endnu ikke, men der er forskellige gæt. Det
førende gæt hedder vakuumenergi. Yderligere undersøgelser vil vise, hvor
gode gættene er. Alt dette har dog ikke nævneværdig betydning for
Universets historie. Det er derimod afgørende for, hvordan det vil
udvikle sig i fremtiden.

> Så udvidelseshastigheden indenfor det første sekund beskrevet som at
> universet udvidede sig fra at være på størrelse med et atom til at være
> på størrelse med en galakse. Det giver jo en sådan pænt stor
> begyndelseshastighed. (Min regnestok er ikke lang nok til at udregne
> den, men 50-100.000 lysår på under et sekund)

Man kan ikke som sådan tale om en udvidelseshastighed uden at angive den
afstand, den er målt over. Hastigheden er jo proportional med afstanden.
Dette er Hubbles lov.

> Så medmindre udvidelseshastigheden altid har været størrere end lysets
> hastighed, hvordan kan lys der er udsendt fra et objekt for 13 mia. år
> siden så først komme frem til os nu?

Fordi det har taget 13 mia år at nå frem. Jeg tror godt, at jeg forstår
dit problem med det. Lad os kaste et blik på tallene:

En galakse, der udsendte lys mod os for 13,0 mia år siden (det er en
meget tidlig galakse) har et rødskift på z = 8,0. Da den udsendte lyset,
har den været 3,3 mia lysår væk fra os. Nu er den 30 mia lysår væk.
Afstanden mellem os og galaksen har altså udvidet sig med en faktor 9
siden lyset blev udsendt. Hvis Universet ikke udvidede sig, ville lyset
selvfølgelig kun have været 3,3 mia år om at nå os. Nu tog det så 13,0
mia år, fordi Universet udvidede sig imens.

Har du nogensinde prøvet at gå på et rullende fortov, der ruller imod den
retning, du går i? Så tror jeg at du forstår, at det tager længere tid at
nå frem, end hvis båndet stod stille - selv om du går med samme
hastighed. Det er lidt det samme, der er sket med lyset fra galakserne.
Hvor ser du nu problemet i det?

ps: I ovenstående afstandsberegninger har jeg brugt de parametre, som man
tror mest på i dag: H_0 = 71, OmegaM = 0,27 Omega_vac = 0,73 Dvs. at
jeg har taget hensyn til, at der er sandsynligvis findes vakuumenergi i
Univserset. Fjerner man vakuumenergien, gør det Universet 2,4 mia år
yngre og gør, at galaksen nu kun er 24 mia lysår væk. Galaksen er så kun
10,7 mia år gammel.

Med venlig hilsen Sven.

Finn Guldmann (28-10-2004)
Kommentar
Fra : Finn Guldmann


Dato : 28-10-04 00:28

Sven Nielsen wrote:

>>Der er bl.a. skrevet om at udvidelseshastigheden ændres. Det giver mig
>>et problem når det handler om at den stiger. Bl.a. med hvilken kraft der
>>får den til det. (Hvis det altså er tilfældet)
> Det er observeret i sidst i 1990'erne, at udvidelsen af Universet går
> hurtigere og hurtigere. Dette er altså et faktum, forudsat der ikke er
> fejl i observationerne. Så undrer du dig over, hvilken kraft, der
> forårsager dette. Jeg kan fortælle dig, at det undrer astronomerne sig
> også over. Det vides endnu ikke, men der er forskellige gæt. Det
> førende gæt hedder vakuumenergi. Yderligere undersøgelser vil vise, hvor
> gode gættene er. Alt dette har dog ikke nævneværdig betydning for
> Universets historie. Det er derimod afgørende for, hvordan det vil
> udvikle sig i fremtiden.
>
Og dog. En stigende acceleration indeholder måske forklaringen på
hvorfor lys der blev udsendt for 13 mia. år siden først kommer her forbi nu.

> Man kan ikke som sådan tale om en udvidelseshastighed uden at angive den
> afstand, den er målt over. Hastigheden er jo proportional med afstanden.
> Dette er Hubbles lov.
>
Jeg så den blot som værende på størrelse med en galakse. Svjv sættes den
vi er en del af til at være ca. 100.000 lysår stor. Men de "fåes vistnok
både størrere og mindre. Men det er stadig en betragtelig hastighed.


>>Så medmindre udvidelseshastigheden altid har været størrere end lysets
>>hastighed, hvordan kan lys der er udsendt fra et objekt for 13 mia. år
>>siden så først komme frem til os nu?
> Fordi det har taget 13 mia år at nå frem. Jeg tror godt, at jeg forstår
> dit problem med det. Lad os kaste et blik på tallene:
> En galakse, der udsendte lys mod os for 13,0 mia år siden (det er en
> meget tidlig galakse) har et rødskift på z = 8,0. Da den udsendte lyset,
> har den været 3,3 mia lysår væk fra os. Nu er den 30 mia lysår væk.
> Afstanden mellem os og galaksen har altså udvidet sig med en faktor 9
> siden lyset blev udsendt. Hvis Universet ikke udvidede sig, ville lyset
> selvfølgelig kun have været 3,3 mia år om at nå os. Nu tog det så 13,0
> mia år, fordi Universet udvidede sig imens.
>
Udfra ovenstående må udvidelseshastigheden, i gennemsnit, havde været 3
x lysets. Er det korrekt antaget?

Men hvordan forklarer du hvordan lyset har brugt 13 mia. år om at
tilbagelægge 3,3 mia. lysår?

Jeg opfatter benævnelsen 'lysår' mere som en afstandsangivelse end som
en tidsangivelse. Velvidende at det er en sammnregning af dem begge.

Og 3,3 mia lysår får jeg til 31.218.000.000.000.000.000.000 km. På
13 mia. år vil lyset "rejse" 122.980.000.000.000.000.000.000 km.

Logisk set vil lys der er udsendt 3,3 mia. lysår væk bruge 3.3 mia. år
om at nå frem til os. Ikke 13 mia. år.

> Har du nogensinde prøvet at gå på et rullende fortov, der ruller imod den
> retning, du går i? Så tror jeg at du forstår, at det tager længere tid at
> nå frem, end hvis båndet stod stille - selv om du går med samme
> hastighed. Det er lidt det samme, der er sket med lyset fra galakserne.
> Hvor ser du nu problemet i det?
>
Svjv er en af de grundliggende ting i afgørelser om universet at lysets
hastighed er konstant.

Så lys udsendt fra en genstand der bevæger sig med 100 km/t bevæger sig
ikke med lysets hastighed + 100 km/t (1.080.000.000 km/t > 1.080.000.100
km/t)

> ps: I ovenstående afstandsberegninger har jeg brugt de parametre, som man
> tror mest på i dag: H_0 = 71, OmegaM = 0,27 Omega_vac = 0,73 Dvs. at
> jeg har taget hensyn til, at der er sandsynligvis findes vakuumenergi i
> Univserset. Fjerner man vakuumenergien, gør det Universet 2,4 mia år
> yngre og gør, at galaksen nu kun er 24 mia lysår væk. Galaksen er så kun
> 10,7 mia år gammel.
>
Ovenstående ligger udenfor mine evner udi beregninger. Jeg er mere til
km og år (og lysår).

--
MVH Finn
To, på hinnanden følgende, undertekster i en udsendelse om
operasangere; "Jeg laver selv min mad" "og beder ofte til Gud"

Henning Makholm (28-10-2004)
Kommentar
Fra : Henning Makholm


Dato : 28-10-04 01:20

Scripsit Finn Guldmann <n@a.invalid>

> Men hvordan forklarer du hvordan lyset har brugt 13 mia. år om at
> tilbagelægge 3,3 mia. lysår?

Da lyset startede var der kun 3,3 mia lysår - men før det var nået
særlig langt havde den afstand der var tilbage udvidet sig betydeligt.

Forestil dig en myre der kravler hen ad en elastik mens vi strækker
elastikken. Myren kravler 10 mm i sekundet (det er en turbomyre!), men
hvert sekund strækker vi elastikken så den bliver 8 mm længere.

Da myren begynder at kravle er elastikken 100 mm lang, så det burde
tage den 10 sekunder at nå frem - hvis altså ikke det var fordi vi
strækker elastikken undervejs. Hvad der i virkeligheden sker er noget
i retning af:

Efter 1 sekund skulle myren have været 90 mm fra målet, men elastikken
er i mellemtiden blevet strukket, så de 90 mm bliver til 90 mm *
108/100 - der er faktisk 97 mm igen.

Efter 2 sekunder tror myren den er 87 mm fra målet, men på grund
af den løbende strækning er de 87 mm nu blevet til 87 mm * 116/108
= 93 mm igen.

Efter 3 sekunder er 83 resterende mm blevet forlænget i forholdet
124/116, og der er stadig 89 mm tilbage.

Efter 4 sekunder er der ikke 79 men 79 * 132/124 = 84 mm tilbage
Efter 5 sekunder er der ikke 74 men 74 * 140/132 = 78 mm tilbage
Efter 6 sekunder er der ikke 68 men 68 * 148/140 = 72 mm tilbage
7 62 62 * 156/148 = 65
8 55 55 * 164/156 = 57
9 47 47 * 172/164 = 49
10 39 39 * 180/172 = 41
11 31 31 * 188/180 = 32
12 22 22 * 196/188 = 23
13 13 13 * 204/196 = 14
14 4 4 * 212/204 = 4

og efter cirka 14½ s er myren endelig nået til enden af elastikken,
som nu samlet er vokset til knap 22 cm.

> Logisk set vil lys der er udsendt 3,3 mia. lysår væk bruge 3.3 mia. år
> om at nå frem til os. Ikke 13 mia. år.

Ikke når universet udvider sig undervejs. Det er det elastikmodellen
illustrerer. (Præcis hvad det vil sige at udvide sig på denne måde
skal man bruge generel relativitetsteori for at forstå. Men når man
først forstår det, kan man se at en del af problemet er at den
oprindelige afstand på "3,3 mia lysår" faktisk er en fupværdi som er
målt langs en *krum* linje i rumtiden og derfor faktisk ikke fortjener
at kaldes "afstand" i virkeligheden.)

> Svjv er en af de grundliggende ting i afgørelser om universet at
> lysets hastighed er konstant.

Ja, men i almindelig relativitetsteori gælder det kun når man måler
lysets hastighed i forhold til en observatør som befinder sig lige i
nærheden af det. Det gælder *ikke* når man forsøger at måle
hastigheden for lys der er (i kosmisk skala) langt væk fra en selv ved
at så på hvor hurtigt dets afstand til mig ændrer sig. Og slet ikke
når man bruger et fup-afstandsmål til det!

--
Henning Makholm "That's okay. I'm hoping to convince the
millions of open-minded people like Hrunkner Unnerby."

Finn Guldmann (28-10-2004)
Kommentar
Fra : Finn Guldmann


Dato : 28-10-04 16:36

Henning Makholm wrote:

>>Men hvordan forklarer du hvordan lyset har brugt 13 mia. år om at
>>tilbagelægge 3,3 mia. lysår?
> Da lyset startede var der kun 3,3 mia lysår - men før det var nået
> særlig langt havde den afstand der var tilbage udvidet sig betydeligt.
>
> Forestil dig en myre der kravler hen ad en elastik mens vi strækker
> elastikken. Myren kravler 10 mm i sekundet (det er en turbomyre!), men
> hvert sekund strækker vi elastikken så den bliver 8 mm længere.
>
> Da myren begynder at kravle er elastikken 100 mm lang, så det burde
> tage den 10 sekunder at nå frem - hvis altså ikke det var fordi vi
> strækker elastikken undervejs. Hvad der i virkeligheden sker er noget
> i retning af:
>
> Efter 1 sekund skulle myren have været 90 mm fra målet, men elastikken
> er i mellemtiden blevet strukket, så de 90 mm bliver til 90 mm *
> 108/100 - der er faktisk 97 mm igen.
>
<cut>

>>Logisk set vil lys der er udsendt 3,3 mia. lysår væk bruge 3.3 mia. år
>>om at nå frem til os. Ikke 13 mia. år.
> Ikke når universet udvider sig undervejs. Det er det elastikmodellen
> illustrerer. (Præcis hvad det vil sige at udvide sig på denne måde
> skal man bruge generel relativitetsteori for at forstå. Men når man
> først forstår det, kan man se at en del af problemet er at den
> oprindelige afstand på "3,3 mia lysår" faktisk er en fupværdi som er
> målt langs en *krum* linje i rumtiden og derfor faktisk ikke fortjener
> at kaldes "afstand" i virkeligheden.)
>
Jeg kan udemærket følge din forklaring, men burde man så ikke bruge en
anden betegnelse end 'lysår'?

Et eller andet der udtrykker lysår+udvidelseskoficient?

Bare sådan for ikke at forvirrere begreberne?

>>Svjv er en af de grundliggende ting i afgørelser om universet at
>>lysets hastighed er konstant.
> Ja, men i almindelig relativitetsteori gælder det kun når man måler
> lysets hastighed i forhold til en observatør som befinder sig lige i
> nærheden af det. Det gælder *ikke* når man forsøger at måle
> hastigheden for lys der er (i kosmisk skala) langt væk fra en selv ved
> at så på hvor hurtigt dets afstand til mig ændrer sig. Og slet ikke
> når man bruger et fup-afstandsmål til det!
>
Men hvad kan så betragte som 'lysets sande hastighed'?

I øvrigt må forholdet vel være det samme uanset afstanden. Blot er det
så lille at det ikke er målebart for de nære objekter?

--
MVH Finn
To, på hinnanden følgende, undertekster i en udsendelse om
operasangere; "Jeg laver selv min mad" "og beder ofte til Gud"

Henning Makholm (28-10-2004)
Kommentar
Fra : Henning Makholm


Dato : 28-10-04 16:52

Scripsit Finn Guldmann <n@a.invalid>
> Henning Makholm wrote:

> > (Præcis hvad det vil sige at udvide sig på denne måde skal man
> > bruge generel relativitetsteori for at forstå. Men når man først
> > forstår det, kan man se at en del af problemet er at den
> > oprindelige afstand på "3,3 mia lysår" faktisk er en fupværdi som
> > er målt langs en *krum* linje i rumtiden og derfor faktisk ikke
> > fortjener at kaldes "afstand" i virkeligheden.)

> Jeg kan udemærket følge din forklaring, men burde man så ikke bruge en
> anden betegnelse end 'lysår'?

Tja, måske. Desværre står det ikke i min magt at ændre ved de rigtige
kosmologernes fagsprog. Jeg er blot en halvstuderet røver som forsøger
at følge med i hvad de foretager sig.

> > Ja, men i almindelig relativitetsteori gælder det kun når man måler
> > lysets hastighed i forhold til en observatør som befinder sig lige i
> > nærheden af det. Det gælder *ikke* når man forsøger at måle
> > hastigheden for lys der er (i kosmisk skala) langt væk fra en selv ved
> > at så på hvor hurtigt dets afstand til mig ændrer sig. Og slet ikke
> > når man bruger et fup-afstandsmål til det!

> Men hvad kan så betragte som 'lysets sande hastighed'?

Lysets sande hastighed er kun den man kan måle i et intertialsystem
der omfatter den begivenhed lyset bevæger sig igennem. Den er på
299792458 m/s.

--
Henning Makholm "Monarki, er ikke noget materielt ... Borger!"

Per Rønne (28-10-2004)
Kommentar
Fra : Per Rønne


Dato : 28-10-04 17:41

Finn Guldmann <n@a.invalid> wrote:

> Jeg kan udemærket følge din forklaring, men burde man så ikke bruge en
> anden betegnelse end 'lysår'?
>
> Et eller andet der udtrykker lysår+udvidelseskoficient?
>
> Bare sådan for ikke at forvirrere begreberne?

Der er skam intet at udsætte på betegnelsen »lysår«. Lysets hastighed er
299.792.458 m/s, og året består af cirka 365,25 dage. Dermed er et lysår
9460730472580800 meter langt. Tænk for letheds skyld på et lysår som
10^13 km [13-skalaen!]. Eller som 1 Pm, 1 petameter.
--
Per Erik Rønne

Martin Larsen (28-10-2004)
Kommentar
Fra : Martin Larsen


Dato : 28-10-04 20:07

"Per Rønne" <spam@husumtoften.invalid> skrev i en meddelelse news:1gmdoyi.dbd0db1aa1ed7N%spam@husumtoften.invalid...
>
> Der er skam intet at udsætte på betegnelsen »lysår«. Lysets hastighed er
> 299.792.458 m/s, og året består af cirka 365,25 dage. Dermed er et lysår
> 9460730472580800 meter langt. Tænk for letheds skyld på et lysår som
> 10^13 km [13-skalaen!]. Eller som 1 Pm, 1 petameter.

FYI: 1 lysår = 9.4605284 petameter

Mvh
Martin



Per Rønne (28-10-2004)
Kommentar
Fra : Per Rønne


Dato : 28-10-04 20:41

Martin Larsen <mlarsen@post7.tele.dk> wrote:

> "Per Rønne" <spam@husumtoften.invalid> skrev i en meddelelse
> news:1gmdoyi.dbd0db1aa1ed7N%spam@husumtoften.invalid...
> >
> > Der er skam intet at udsætte på betegnelsen »lysår«. Lysets hastighed er
> > 299.792.458 m/s, og året består af cirka 365,25 dage. Dermed er et lysår
> > 9460730472580800 meter langt. Tænk for letheds skyld på et lysår som
> > 10^13 km [13-skalaen!]. Eller som 1 Pm, 1 petameter.
>
> FYI: 1 lysår = 9.4605284 petameter

OK, et lysår ≈ 10 Pm. Det er da til at huske. Hvorfor er man i øvrigt så
tilbøjelig til at lave sådanne småfejl?
--
Per Erik Rønne

Martin Larsen (28-10-2004)
Kommentar
Fra : Martin Larsen


Dato : 28-10-04 22:34

"Per Rønne" <spam@husumtoften.invalid> skrev i en meddelelse news:1gmdyrn.hx561617jud3kN%spam@husumtoften.invalid...
> Martin Larsen <mlarsen@post7.tele.dk> wrote:
>
> > FYI: 1 lysår = 9.4605284 petameter
>
> OK, et lysår ? 10 Pm. Det er da til at huske. Hvorfor er man i øvrigt så
> tilbøjelig til at lave sådanne småfejl?

Det er fordi man ikke er grundig nok med at læse det
man har skrevet.
Man er måske lettere euforisk over det gode indlæg man
har lavet og glemmer den kritiske sans.
Desuden tror jeg at vi er nogle der desværre kigger mere
på tastaturet end på skærmen når vi skriver.

Mvh
Martin



Henning Makholm (28-10-2004)
Kommentar
Fra : Henning Makholm


Dato : 28-10-04 16:40

Scripsit Henning Makholm <henning@makholm.net>

> Forestil dig en myre der kravler hen ad en elastik mens vi strækker
> elastikken. Myren kravler 10 mm i sekundet (det er en turbomyre!), men
> hvert sekund strækker vi elastikken så den bliver 8 mm længere.

Se bort fra denne forklaring - ved nærmere eftertanke giver den først
mening når man forstår Friedmann-Robertson-Walker-modellen og dens
underliggende matematik.

Et bedre svar er (jeg kan ikke huske præcis hvor gammelt det er
universet er i den model Sven udtaler sig om, men lad os sætte det til
13,5 mia år):

Det lys vi taler om, blev udsendt fra den fjerne galakse på et
tidspunkt hvor der *med dens tidsmål* var gået 0,5 mia år siden big
bang. Eftersom der *i vores tid* er gået 13,5 mia år siden big bang,
trækker kosmologerne de to tal fra hinanden og finder at det var 13
mia år siden lyset blev udsendt.

Det kan kosmologerne tillade sig, fordi de ved hvad det betyder og
derfor ikke lader sig forvirre af at det strengt taget er noget
vrøvl. Faktisk svarer fremgangsmåden til at sige at der er 35 km fra
Roskilde til København og 29 km fra Roskilde til Frederikssund - altså
må Frederikssund ligge 6 km fra København!

(Dette eksempel giver en afvigelse i den modsatte retning af
kosmologernes. Det skyldes at deres beregning sker delvist i tid,
ikke i rum, og i relativitetsteori kommer man tit ud for at tid
opfører sig modsat af rum).

For at få et tidsrum og afstand der giver bedre mening, må man huske
at den fjerne galakse bevæger sig med tæt på lysets hastighed i
forhold til os. Derfor ser vi dens ure gå meget langsommere end vores
egne, og når de er nået til "0,5 mia år efter big bang", er vores nået
meget længere - fx til "11 mia år efter big bang". Derfor vil det være
rigtigere at sige at det lys vi snakker om blev udsendt "for 2,5 mia
år siden".

Tilsvarende er kosmologernes afstandsangivelse på 3,3 mia lysår
vildledende - den skal man nemlig forestille sig målt med et ensemble
af tommestokke som alle befinder sig 0,5 mia år fra big bang, og som
hver for sig er i hvile i forhold til den galakse der er nærmest hver
enkelt tommestok. Eftersom disse galakser bevæger sig væk fra os, vil
de fjerneste af tommestokkene blive relativistisk forkortet, og de vil
derfor måle en større afstand (der skal flere tommestokke til at dække
strækningen) end den hvor lyshastigheden som afstand pr tid giver
mening i *vores* koordinatsystem. Målt med tommestokke som er i hvile
i forhold til *os* ligger udsendelsesbegivenheden måske kun 2,5 mia
lysår væk. Så har lyset været 2,5 mia år om at bevæge sig 2,5 mia
lysår, og alting stemmer.

(Før folk overfalder mig, er ovenstående forklaring naturligvis
voldsomt forsimplet. Tallene er grebet ud af luften, og jeg er ikke
sikker på om alting faktisk vil ende med at stemme når man regner på
den måde. De *vil* stemme i det Milneske grænsetilfælde hvor der
hverken er tyngdekraft eller kosmologisk konstant, og umiddelbart tror
jeg ikke de vil blive vanvittig meget forskudt af at bruge en mere
realistisk model. Jeg er også klar over at det strengt taget ikke
giver mening at forestille sig et inertialsystem der strækker sig helt
fra udsendelsestidspunktet til vi ser lyset. Men jeg mener det er
*tættere* på at give mening end at begynde at trække kosmologiske
tidspunkter på forskellige steder fra hinanden...)

--
Henning Makholm "However, the fact that the utterance by
Epimenides of that false sentence could imply the
existence of some Cretan who is not a liar is rather unsettling."

Henning Makholm (28-10-2004)
Kommentar
Fra : Henning Makholm


Dato : 28-10-04 18:11

Scripsit Henning Makholm <henning@makholm.net>

> Tilsvarende er kosmologernes afstandsangivelse på 3,3 mia lysår
> vildledende - den skal man nemlig forestille sig målt med et ensemble
> af tommestokke som alle befinder sig 0,5 mia år fra big bang, og som
> hver for sig er i hvile i forhold til den galakse der er nærmest hver
> enkelt tommestok. Eftersom disse galakser bevæger sig væk fra os, vil
> de fjerneste af tommestokkene blive relativistisk forkortet, og de vil
> derfor måle en større afstand

Suk. Faktisk er det omvent - idet længdemålingen lokalt sker momentant
i et system der bevæger sig sammen med den forkortede tommestok, vil
målebegivenheden i den ene ende af stokken set i vores koordinatsystem
ske tidligere end målebegivenheden i den anden ende. I løbet af det
tidspunkt har tommestokken bevæget sig meget længere væk fra os end
det er nødvendigt for at ophæve effekten af længdeforkortelsen.
Faktisk vil det kosmologiske afstandsmål give en *mindre* afstand end
vi får når vi forsøger at måle hele afstanden i et inertialsystem hvor
vi står stille - og forholdet er netop gamma.

(Dette bliver klart når man faktisk foretager Lorentz-transformationen
i stedet for bare at huske et færdigfordøjet resultat om
længdeforkortning).

> Så har lyset været 2,5 mia år om at bevæge sig 2,5 mia
> lysår, og alting stemmer.

Det er stadig rigtigt at alting stemmer, men mine tal er
skrupforkerte. Jeg konstruerede dem på slumpt baglæns ud fra en
overordnet viden om at det hele stemmer i Milne-modellen, men i
forhold til Svens beregning burde jeg snarere have konkluderet noget i
retning af

Så har lyset været 5 mia år om at bevæge sig 5 mia lysår, og alting
stemmer.

--
Henning Makholm "I ... I have to return some videos."

Per Rønne (28-10-2004)
Kommentar
Fra : Per Rønne


Dato : 28-10-04 14:33

Finn Guldmann <n@a.invalid> wrote:

> Men hvordan forklarer du hvordan lyset har brugt 13 mia. år om at
> tilbagelægge 3,3 mia. lysår?

Det har det jo heller ikke været; da lyset blev udsendt var afstanden
3,3 milliarder lysår, men når det har bevæget sig 1 milliard lysår, vil
rummets samtidige udvidelse betyde at der nu vil være måske fem
milliarder lysår tilbage.
--
Per Erik Rønne

Sven Nielsen (28-10-2004)
Kommentar
Fra : Sven Nielsen


Dato : 28-10-04 18:49

In article <clpb0k$2927$1@news.cybercity.dk>, n@a.invalid says...

> Og dog. En stigende acceleration indeholder måske forklaringen på
> hvorfor lys der blev udsendt for 13 mia. år siden først kommer her forbi nu.

Nej, det gør det, som jeg allerede har skrevet, ikke. Den accelererende
udvidelse er først kommet til for nylig, og spiller derfor en begrænset
rolle for Universets fortid.

> Jeg så den blot som værende på størrelse med en galakse. Svjv sættes den
> vi er en del af til at være ca. 100.000 lysår stor. Men de "fåes vistnok
> både størrere og mindre. Men det er stadig en betragtelig hastighed.

Jeg kan ikke få dette til at give mening. Hvad er det lige, du mener?
Hvad er på størrelse med en galakse? Man kan ikke sammenligne en
udstrækning med en hastighed.

> Udfra ovenstående må udvidelseshastigheden, i gennemsnit, havde været 3
> x lysets. Er det korrekt antaget?

Nej. Universet udvider sig ikke med en bestemt hastighed, man kan
sammenligne med c. Universets udvidelse beskrives af hubble-konstanten,
som er det H, der står i Hubbles lov: v = H * d. Her er v en galakses
hastighed, og d er afstanden til galaksen.

> Men hvordan forklarer du hvordan lyset har brugt 13 mia. år om at
> tilbagelægge 3,3 mia. lysår?

Lyset har jo også tilbagelagt 13 mia lysår. Man skal vide, at hastigheder
og afstande er lidt skumle indenfor relativitetsteori. Men når det nu er
sagt, synes jeg at det giver god mening.

> Jeg opfatter benævnelsen 'lysår' mere som en afstandsangivelse end som
> en tidsangivelse. Velvidende at det er en sammnregning af dem begge.

Nej, lysår er altid en enhed for afstand, ligesom meter eller parsec
Parsec er astronomernes foretrukne enhed, når man beregner afstande i
Universet.

> Og 3,3 mia lysår får jeg til 31.218.000.000.000.000.000.000 km. På
> 13 mia. år vil lyset "rejse" 122.980.000.000.000.000.000.000 km.

Det er korrekt.

> Logisk set vil lys der er udsendt 3,3 mia. lysår væk bruge 3.3 mia. år
> om at nå frem til os. Ikke 13 mia. år.

Nu er det dig, der er ulogisk. Og jeg skrev jo også helt præcist, at 3,3
mia år er korrekt i et univers, der ikke udvider sig. Men naturligvis
ikke, når man har en udvidelse. Der er man nødt til at bruge
relativitetsteori, for at regne det rigtigt ud.

Jeg kan vise dig en lille tilnærmet beregning, som kan antyde princippet.
Lad os starte med, at der er ca. 3 mia lysår mellem os og galaksen, når
vi starter. Fra start til slut udvider Universet sig med en faktor 9.
Altså er det som det eksempel, vi før snakkede om. Vi deler strækningen
mellem galaksen og os op i tre (fra starten) lige store dele: A, B og C.
Vi lader som om, at Univset ikke udvider sig jævnt, men i to "ryk" for at
gøre det nemt at regne på. OK, here goes:

Lyset bevæger sig gennem strækning A på en mia år.
Nu udvider Univsetset sig en faktor 3.
B er derfor 3 gange så lang som A. Det tager 3 mia år for lyset af løbe
gennem B. Universet udvider sig igen faktor 3. I alt 3*3=9 siden vi
startede. Strækning C er nu 9 gange så lang som A oprindelig var. Det
tager lyset 9 mia år at løbe C igennem. Sammenlagt har det taget lyset
1+3+9 = 13 mia år at nå os. Udregningen gav ovenikøbet det rigtige
resultat!

> Svjv er en af de grundliggende ting i afgørelser om universet at lysets
> hastighed er konstant.

Det er både rigtigt og forkert. Det er altid rigtigt, hvis man overholder
de spilleregler, der er i den specielle relativitetsteori. Men den kan
desværre ikke bruges, når man taler om hele Universet. Her bruger man den
almene relativitetsteori. En diskussion om dette kan blive meget teknisk.

> Så lys udsendt fra en genstand der bevæger sig med 100 km/t bevæger sig
> ikke med lysets hastighed + 100 km/t (1.080.000.000 km/t > 1.080.000.100
> km/t)

Det gælder kun "lokalt." I kosmologi vælger man af praktiske hensyn at
omdefinere tid og sted lidt i forhold til normal klassisk fysik. Det er
tilladt, fordi at relativitetsteorien garanterer, at målbare størrelser
ikke forvanskes. Igen er dette meget teknisk. Men det betyder, at du ikke
kan bruge dit udsagn til at argumentere for, at big bang teorien er
regnemæssigt forkert. Det er den ikke, når man bruger de rigtige
regneregler, som man får fra almen relativitetsteori.

> > ps: I ovenstående afstandsberegninger har jeg brugt de parametre, som man
> > tror mest på i dag: H_0 = 71, OmegaM = 0,27 Omega_vac = 0,73 Dvs. at
> > jeg har taget hensyn til, at der er sandsynligvis findes vakuumenergi i
> > Univserset. Fjerner man vakuumenergien, gør det Universet 2,4 mia år
> > yngre og gør, at galaksen nu kun er 24 mia lysår væk. Galaksen er så kun
> > 10,7 mia år gammel.
> >
> Ovenstående ligger udenfor mine evner udi beregninger. Jeg er mere til
> km og år (og lysår).

Det var også mest ment som en serviceoplysning til dem, der evt. måtte
ønske at eftergøre mine beregninger. Ikke desto mindre er disse tre tal
helt centrale, hvis man vil vide noget om Universet efter big bang. Det
er de tal, man sætter ind i den model, der gør, at man kan regne ud hvor
lang tid det tager lys at bevæge sig rundt i Universet. Det er de tal,
der gør, at vi ved hvor gammelt Universet er. Derfor går megen forskning
ud på at måle disse tre størrelser så præcist som muligt. H_0 er hubble-
konstanten, der fortæller, hvor meget Universet udvider sig. OmegaM
angiver hvor stor den gennemsnitlige massetæthed er i Universet.
Omega_vac angiver, hvor meget vakuumenergi, der er.

Med venlig hilsen Sven.

Finn Guldmann (29-10-2004)
Kommentar
Fra : Finn Guldmann


Dato : 29-10-04 04:50

Sven Nielsen wrote:

>>Og dog. En stigende acceleration indeholder måske forklaringen på
>>hvorfor lys der blev udsendt for 13 mia. år siden først kommer her forbi nu.
> Nej, det gør det, som jeg allerede har skrevet, ikke. Den accelererende
> udvidelse er først kommet til for nylig, og spiller derfor en begrænset
> rolle for Universets fortid.
>
Det gør udvidelsen endnu mere mystisk.

Hvilken hændelse skulle det være der har igangsat denne øgede
acceleration fornylig?
Den burde være til at finde spor efter.


>>Jeg så den blot som værende på størrelse med en galakse. Svjv sættes den
>>vi er en del af til at være ca. 100.000 lysår stor. Men de "fåes vistnok
>>både størrere og mindre. Men det er stadig en betragtelig hastighed.
> Jeg kan ikke få dette til at give mening. Hvad er det lige, du mener?
> Hvad er på størrelse med en galakse? Man kan ikke sammenligne en
> udstrækning med en hastighed.
>
Det bliver et stort tal. Men det burde kunne lade sig gøre at udregne en
udvidelseshastighed udfra oplysningerne om at udvidelsen fra en
størrelse lig et atom til en størrelse lig en galakse på (f.eks.) 50.000
lysår i udstrækning, sker indenfor et sekund.

>>Men hvordan forklarer du hvordan lyset har brugt 13 mia. år om at
>>tilbagelægge 3,3 mia. lysår?
> Lyset har jo også tilbagelagt 13 mia lysår. Man skal vide, at hastigheder
> og afstande er lidt skumle indenfor relativitetsteori. Men når det nu er
> sagt, synes jeg at det giver god mening.
>
Jamen, jamen. Hvordan kan det så være kun 3,3 mia. lysår gammelt.

Du nævner godtnok at tingene kan være skumle, men alligevel.

> Nu er det dig, der er ulogisk. Og jeg skrev jo også helt præcist, at 3,3
> mia år er korrekt i et univers, der ikke udvider sig. Men naturligvis
> ikke, når man har en udvidelse. Der er man nødt til at bruge
> relativitetsteori, for at regne det rigtigt ud.
> Jeg kan vise dig en lille tilnærmet beregning, som kan antyde princippet.
> Lad os starte med, at der er ca. 3 mia lysår mellem os og galaksen, når
> vi starter. Fra start til slut udvider Universet sig med en faktor 9.
> Altså er det som det eksempel, vi før snakkede om. Vi deler strækningen
> mellem galaksen og os op i tre (fra starten) lige store dele: A, B og C.
> Vi lader som om, at Univset ikke udvider sig jævnt, men i to "ryk" for at
> gøre det nemt at regne på. OK, here goes:
> Lyset bevæger sig gennem strækning A på en mia år.
> Nu udvider Univsetset sig en faktor 3.
> B er derfor 3 gange så lang som A. Det tager 3 mia år for lyset af løbe
> gennem B. Universet udvider sig igen faktor 3. I alt 3*3=9 siden vi
> startede. Strækning C er nu 9 gange så lang som A oprindelig var. Det
> tager lyset 9 mia år at løbe C igennem. Sammenlagt har det taget lyset
> 1+3+9 = 13 mia år at nå os. Udregningen gav ovenikøbet det rigtige
> resultat!
>
Den tror jeg fes ind på lystavlen.

> Det er både rigtigt og forkert. Det er altid rigtigt, hvis man overholder
> de spilleregler, der er i den specielle relativitetsteori. Men den kan
> desværre ikke bruges, når man taler om hele Universet. Her bruger man den
> almene relativitetsteori. En diskussion om dette kan blive meget teknisk.
>
Så lad os droppe den.

> Det gælder kun "lokalt." I kosmologi vælger man af praktiske hensyn at
> omdefinere tid og sted lidt i forhold til normal klassisk fysik. Det er
> tilladt, fordi at relativitetsteorien garanterer, at målbare størrelser
> ikke forvanskes. Igen er dette meget teknisk. Men det betyder, at du ikke
> kan bruge dit udsagn til at argumentere for, at big bang teorien er
> regnemæssigt forkert. Det er den ikke, når man bruger de rigtige
> regneregler, som man får fra almen relativitetsteori.
>
Jeg har ikke de store problemer med at acceptere BB.

Det der volder mig vanskeligheder er der stigende acceleration. Og hvad
der får den til at ske.
Den med hvad der sker med lyset i et univers der udvider sig har du og
Henning vistnok fået forklaret så jeg tror jeg er nogenlunde med.

--
MVH Finn
To, på hinnanden følgende, undertekster i en udsendelse om
operasangere; "Jeg laver selv min mad" "og beder ofte til Gud"

Per Rønne (29-10-2004)
Kommentar
Fra : Per Rønne


Dato : 29-10-04 06:22

Finn Guldmann <n@a.invalid> wrote:

> Jamen, jamen. Hvordan kan det så være kun 3,3 mia. lysår gammelt.

Hvis du nu skrev »kun 33 Pm«, petameter, blev du måske mindre forvirret
over sammenblandingen af tid og afstand.
--
Per Erik Rønne

Finn Guldmann (29-10-2004)
Kommentar
Fra : Finn Guldmann


Dato : 29-10-04 15:31

Per Rønne wrote:

>>Jamen, jamen. Hvordan kan det så være kun 3,3 mia. lysår gammelt.
> Hvis du nu skrev »kun 33 Pm«, petameter, blev du måske mindre forvirret
> over sammenblandingen af tid og afstand.
>
Kun over at møde en ny betegnelse for det samme.
1 lysår = 10 petameter, selvom det jo så ikke passer.

- 1 Pm is equal to 6685 AU
- 1 light-year A light year, is equal to 9.46 Pm
- 1 parsec is equal to 30.86 Pm
- light travels the distance of 1 Pm in 38.6 days (38 d 14 h 34 min 1 s)
(Sakset fra "The Free Dictionary")

--
MVH Finn
To, på hinnanden følgende, undertekster i en udsendelse om
operasangere; "Jeg laver selv min mad" "og beder ofte til Gud"

Per Rønne (29-10-2004)
Kommentar
Fra : Per Rønne


Dato : 29-10-04 16:07

Finn Guldmann <n@a.invalid> wrote:

> Per Rønne wrote:
>
> >>Jamen, jamen. Hvordan kan det så være kun 3,3 mia. lysår gammelt.
> > Hvis du nu skrev »kun 33 Pm«, petameter, blev du måske mindre forvirret
> > over sammenblandingen af tid og afstand.
> >
> Kun over at møde en ny betegnelse for det samme.
> 1 lysår = 10 petameter, selvom det jo så ikke passer.
>
> - 1 Pm is equal to 6685 AU
> - 1 light-year A light year, is equal to 9.46 Pm
> - 1 parsec is equal to 30.86 Pm
> - light travels the distance of 1 Pm in 38.6 days (38 d 14 h 34 min 1 s)
> (Sakset fra "The Free Dictionary")

Som huskeregel er 1 lysår = 10 Pm nu meget god.
--
Per Erik Rønne

Sven Nielsen (01-11-2004)
Kommentar
Fra : Sven Nielsen


Dato : 01-11-04 22:23

In article <clsep4$25bm$1@news.cybercity.dk>, n@a.invalid says...

> Hvilken hændelse skulle det være der har igangsat denne øgede
> acceleration fornylig? Den burde være til at finde spor efter.

Nej, der er såmænd ikke sket noget for nylig, der har igangsat
accelerationen. Den har været indprogrammeret lige fra starten.

Det simplest mulige forklaring er flg.: Den accelererende udvidelse
skyldes vakuumenergi. Der er vakuumenergi overalt i Universet med en
energitæthed på: 6*10e-30 g/cm^3. Det er den værdi, man kommer frem til
på baggrund af observationer af fjerne supernovaer. Denne værdi antages
at have været konstant siden brøkdele af et sekund efter big bang. Den
ændres ikke når Universet udvider sig.

Tætheden af masse menes i dag at være på ca. 2*10e-30 g/cm^3 (dvs. 1/3
af vakuumenergien). Men denne tæthed ændres med Universets skalafaktor
i minus tredie potens. Dvs. at da afstanden mellem to galakser engang
var den halve af, hvad den er i dag, da var tætheden 8 gange så stor
(1,6 *10e-29 g/cm^3). Vi skal altså ikke så lang tid tilbage, før
masseenergien i Universet var så stor i forhold til vakuumenergien, at
vakuumenergien ingen betydning havde. Omvendt skal vi ikke så langt ind
i fremtiden, før masserne i Universet ingen betydning har for
udvidelsen.

Det egentlige mysterium er faktisk, hvorfor vi er her og ser på
Universet netop nu, hvor masseenergi og vakuumenergi nogenlunde
balancerer. Hvorfor er vakuumenergien ikke f.eks. 10e50 g/cm^3, så
Universet havde været domineret af vakuumenergi allerede efter den
første brøkdel af et sekund? Det er et af de spørgsmål, som man håber
på at finde svaret på.

Med venlig hilsen Sven.

Pongo (23-10-2004)
Kommentar
Fra : Pongo


Dato : 23-10-04 17:46

Finn Guldmann wrote:
> Jens Olsen wrote:
>
>>>> Hvis universet udvider sig som en ballon (kugleskal) må der jo
>>>> være et centrum af denne kugle. Et centrum som alting har bevæget
>>>> sig væk fra i 13,7 mia år. Mens det har ændret form og fysisk
>>>> tilstand.
>> Ok, det er fordi du ikke helt har forstået ballonanalogien så. Det er
>> jo netop kun en analogi.

Her er en anden analogi, som sikkert også har mangler.
Forestil dig Universet som en stor kasse som er fuldstændigt pakket med
oppustede balonner. Som tiden går, siver luften langsomt ud af balonnerne,
så afstanden mellem dem bliver større og større. Ballonnerne tiltrækker
hinanden, så med tiden samles de i små klumper. Klumperne samler sig til
større klumper osv.
Hvis vi nu forestiller os en observatør der er skabt af disse balonner, og
som iagtager det hele nede fra kassen. Denne observatør kan måle verden i
ballon-diametre, da det er den eneste reference der er til rådighed. Han
mener derfor at afstanden til de andre balonner bliver større og større.
Lad nu balonnerne være f.eks atomer, antag at kassen er uendelig stor og
ikke kan betragtes udefra. Så har vi et billede på et uendeligt stort
Univers hvor afstandene bliver større og større (dvs Udvidelse). Der er
intet centrum for udvidelsen, og intet centrum for Universet.

/Klaus



Finn Guldmann (25-10-2004)
Kommentar
Fra : Finn Guldmann


Dato : 25-10-04 06:50

Pongo wrote:
>>>Ok, det er fordi du ikke helt har forstået ballonanalogien så. Det er
>>>jo netop kun en analogi.
> Her er en anden analogi, som sikkert også har mangler.
> Forestil dig Universet som en stor kasse som er fuldstændigt pakket med
> oppustede balonner. Som tiden går, siver luften langsomt ud af balonnerne,
> så afstanden mellem dem bliver større og større. Ballonnerne tiltrækker
> hinanden, så med tiden samles de i små klumper. Klumperne samler sig til
> større klumper osv.
> Hvis vi nu forestiller os en observatør der er skabt af disse balonner, og
> som iagtager det hele nede fra kassen. Denne observatør kan måle verden i
> ballon-diametre, da det er den eneste reference der er til rådighed. Han
> mener derfor at afstanden til de andre balonner bliver større og større.
> Lad nu balonnerne være f.eks atomer, antag at kassen er uendelig stor og
> ikke kan betragtes udefra. Så har vi et billede på et uendeligt stort
> Univers hvor afstandene bliver større og større (dvs Udvidelse). Der er
> intet centrum for udvidelsen, og intet centrum for Universet.
>
Det får jeg til at udvidelsen af universet er "fake". Det ser blot sådan
ud for os, fordi det vi ser på bliver mindre og mindre.

--
MVH Finn
To, på hinnanden følgende, undertekster i en udsendelse om
operasangere; "Jeg laver selv min mad" "og beder ofte til Gud"

Pongo (25-10-2004)
Kommentar
Fra : Pongo


Dato : 25-10-04 17:36

Finn Guldmann wrote:
> Det får jeg til at udvidelsen af universet er "fake". Det ser blot
> sådan ud for os, fordi det vi ser på bliver mindre og mindre.

Er der forskel på følgende 2 udsagn ?
"1: Universet bliver større og større i forhold til os !"
"2: Vi bliver mindre og mindre i forhold til Universet!"

Hvorfor mener du det ene er fake og det andet er virkelighed, og hvordan vil
du undersøge hvad der er hvad ?

I analogien leger vi at Universet kan betragtes udefra og ses som en lukket
kasse. Det kan man ikke, og er kun et tankeeksperiment der letter
forståelsen. Hvis der eksisterede en over-virkelighed, hvori Universet
eksistere og kan betragtes udefra, kan man begynde at diskutere om det er os
der bliver mindre eller om Universet bliver større, men set fra vores
virkelighed bliver Universet større.
Det er vel alene et spørgsmål om hvilke referencer man har.
/Klaus



Finn Guldmann (25-10-2004)
Kommentar
Fra : Finn Guldmann


Dato : 25-10-04 20:47

Pongo wrote:

> Finn Guldmann wrote:
>
>>Det får jeg til at udvidelsen af universet er "fake". Det ser blot
>>sådan ud for os, fordi det vi ser på bliver mindre og mindre.
>
>
> Er der forskel på følgende 2 udsagn ?
> "1: Universet bliver større og større i forhold til os !"
> "2: Vi bliver mindre og mindre i forhold til Universet!"
>
> Hvorfor mener du det ene er fake og det andet er virkelighed, og hvordan vil
> du undersøge hvad der er hvad ?
>
> I analogien leger vi at Universet kan betragtes udefra og ses som en lukket
> kasse. Det kan man ikke, og er kun et tankeeksperiment der letter
> forståelsen. Hvis der eksisterede en over-virkelighed, hvori Universet
> eksistere og kan betragtes udefra, kan man begynde at diskutere om det er os
> der bliver mindre eller om Universet bliver større, men set fra vores
> virkelighed bliver Universet større.
> Det er vel alene et spørgsmål om hvilke referencer man har.
>
Ved den med "kassen" og ballonnerne bliver der ændrer på objekternes
størrelse, men de holder afstanden til os.

Ved udvidelser beholder objekterne deres størrelse, men deres afstand
til os bliver størrere.

Umiddelbart ser det ens ud for os, men praktiske målinger kan afgøre om
det er det ene eller det andet.

--
MVH Finn
To, på hinnanden følgende, undertekster i en udsendelse om
operasangere; "Jeg laver selv min mad" "og beder ofte til Gud"

Pongo (25-10-2004)
Kommentar
Fra : Pongo


Dato : 25-10-04 21:33

Finn Guldmann wrote:

> Ved den med "kassen" og ballonnerne bliver der ændrer på objekternes
> størrelse, men de holder afstanden til os.

Kun for en observatør der står udenfor kassen og som kan bruge kassens
størrelse som reference.

> Ved udvidelser beholder objekterne deres størrelse, men deres afstand
> til os bliver størrere.
>
> Umiddelbart ser det ens ud for os, men praktiske målinger kan afgøre
> om det er det ene eller det andet.

Praktiske målinger i kassen bliver foretaget i enheder baseret på
ballonernes størrelse. Derfor bliver kassen større og større set indefra
kassen. Man kunne have valgt at måle i enheder baseret på kassens størrelse,
og så ville man finde at balonnerne blev mindre og mindre. Det ændre dog
ikke noget ved virkeligheden, men blot ved de tal man må nedskrive.

Valgte vi at opmåle objekter i Universet i en enhed som var bygget på
brøkdele af Universets absolutte størrelse, ville vi finde at vi optog en
mindre og mindre andel af Universets rumfang. Vi ville sige at vi blev
mindre og mindre. Vi vælger istedet at benytte enheder baseret på ting vi
kan observere i Universet, og derved er det Universet som synes at blive
større og større.
Af praktiske grunde vælger vi at sige at Universet udvider sig og at
enhederne vi måler i, er uforanderlige. Valgte vi Universet som den
uforanderlig størrelse, skulle vi jo omskrive alle måleresultater hele
tiden, da enheden jo var blevet mindre siden målingen blev foretaget.
Natur-"konstanterne" ville også skulle omdøbes til Natur-"tællere", da de jo
hele tiden skulle justeres.
Så længe vi ikke har noget referencesystem udenfor Universet, som vi kan
antage har en fast uforanderlig størrelse, kan vi ikke afgøre om det ene
bliver mindre eller om det andet bliver større. Antager vi, at der slet ikke
findes et sådant referencesystem, er det jo også blot en strid om ord.
Måske begyndte Tiden at gå meget hurtigere forleden nat. Vi opdagede det
bare ikke, for vi vil kun kunne konstatere det, hvis vi havde et magisk ur
der ikke er påvirket af Universets Tid, men af en eller anden grundtid .
Fandt vi sådan et ur, vil vi nok blive enige om at det måtte være det
magiskeur der var begyndt at gå langsommere, og ikke Universets Tid der gik
hurtigere.
/Klaus



Finn Guldmann (25-10-2004)
Kommentar
Fra : Finn Guldmann


Dato : 25-10-04 22:26

Pongo wrote:

>>Ved den med "kassen" og ballonnerne bliver der ændrer på objekternes
>>størrelse, men de holder afstanden til os.
> Kun for en observatør der står udenfor kassen og som kan bruge kassens
> størrelse som reference.
>>Ved udvidelser beholder objekterne deres størrelse, men deres afstand
>>til os bliver størrere.
>>Umiddelbart ser det ens ud for os, men praktiske målinger kan afgøre
>>om det er det ene eller det andet.
> Praktiske målinger i kassen bliver foretaget i enheder baseret på
> ballonernes størrelse. Derfor bliver kassen større og større set indefra
> kassen. Man kunne have valgt at måle i enheder baseret på kassens størrelse,
> og så ville man finde at balonnerne blev mindre og mindre. Det ændre dog
> ikke noget ved virkeligheden, men blot ved de tal man må nedskrive.
>
Jeg tænkte nu mere i de baner at man stod som betragter på den ene
ballon og betragtede den anden.

Det man umiddelbart kunne se, i begge tilfælde var at den blev mindre.
Ud fra det alene kunne man jo ikke afgøre om det var fordi den fysisk
blev mindre eller om det var fordi den fysisk flyttede sig længere væk.

Men det ville en måling af afstanden kunne afgøre.


> Valgte vi at opmåle objekter i Universet i en enhed som var bygget på
> brøkdele af Universets absolutte størrelse, ville vi finde at vi optog en
> mindre og mindre andel af Universets rumfang. Vi ville sige at vi blev
> mindre og mindre. Vi vælger istedet at benytte enheder baseret på ting vi
> kan observere i Universet, og derved er det Universet som synes at blive
> større og større.
> Af praktiske grunde vælger vi at sige at Universet udvider sig og at
> enhederne vi måler i, er uforanderlige. Valgte vi Universet som den
> uforanderlig størrelse, skulle vi jo omskrive alle måleresultater hele
> tiden, da enheden jo var blevet mindre siden målingen blev foretaget.
> Natur-"konstanterne" ville også skulle omdøbes til Natur-"tællere", da de jo
> hele tiden skulle justeres.
> Så længe vi ikke har noget referencesystem udenfor Universet, som vi kan
> antage har en fast uforanderlig størrelse, kan vi ikke afgøre om det ene
> bliver mindre eller om det andet bliver større. Antager vi, at der slet ikke
> findes et sådant referencesystem, er det jo også blot en strid om ord.
> Måske begyndte Tiden at gå meget hurtigere forleden nat. Vi opdagede det
> bare ikke, for vi vil kun kunne konstatere det, hvis vi havde et magisk ur
> der ikke er påvirket af Universets Tid, men af en eller anden grundtid .
> Fandt vi sådan et ur, vil vi nok blive enige om at det måtte være det
> magiskeur der var begyndt at gå langsommere, og ikke Universets Tid der gik
> hurtigere.
>
Jeg kan ikke se noget forkert i at vi udtrykker vores resultater udfra
det vi kan observere.

For at afgøre din problemstilling skulle vi jo finde en måde at måle os
frem til universets "kant". Og derefter kunne måle om den flytter sig i
forhold til universets modstående "kant".

Og så langt var vist end ikke "Star Wars" kommet.

--
MVH Finn
To, på hinnanden følgende, undertekster i en udsendelse om
operasangere; "Jeg laver selv min mad" "og beder ofte til Gud"

Pongo (25-10-2004)
Kommentar
Fra : Pongo


Dato : 25-10-04 22:58

Finn Guldmann wrote:
>> Derfor bliver kassen større og større set
>> indefra kassen. Man kunne have valgt at måle i enheder baseret på
>> kassens størrelse, og så ville man finde at balonnerne blev mindre
>> og mindre. Det ændre dog ikke noget ved virkeligheden, men blot ved
>> de tal man må nedskrive.
> Jeg tænkte nu mere i de baner at man stod som betragter på den ene
> ballon og betragtede den anden.

Jo men denne observatør der står på balonnen bliver jo også selv mindre og
mindre. Derfor kan han ikke konstater om balonnerne bliver mindre, men blot
at der bliver mere plads i kassen.
Lokalt kan balonnerne jo godt samle sig i grupper , men de balonner der er
længst væk, skal man bruge flere og flere ballonner for at kunne skabe en
bro til.

> Jeg kan ikke se noget forkert i at vi udtrykker vores resultater udfra
> det vi kan observere.

Nej - og det er netop når vi gør det, at vi når frem til at det må være
Universet der bliver større, og ikke det vi observere der bliver mindre.

> For at afgøre din problemstilling skulle vi jo finde en måde at måle
> os frem til universets "kant". Og derefter kunne måle om den flytter
> sig i forhold til universets modstående "kant".
>
> Og så langt var vist end ikke "Star Wars" kommet.

Det er rigtigt, så vi kan ikke bruge Universets størrelse som reference. Vi
kan se at der bliver mere og mere plads i Universet, og da vi ikke synes det
er praktisk at tale om at vi bliver mindre og mindre, så siger vi istedet at
Universet bliver større og større.
/Klaus



Carsten Svaneborg (24-10-2004)
Kommentar
Fra : Carsten Svaneborg


Dato : 24-10-04 01:20

Finn Guldmann wrote:
> Det giver mig to problemer.
> For det første; Hvad så med det jeg kan opfatte i det tredie plan (3D)?
> Og hvad forhold har det plan (2D) du beskriver i forhold til
> udvidelsesretningen?

Ok. I stedet for at forestille dig 2D overfladen af en ballon i 3D,
så prøv at forestille dig en 3D overflade (universet) af en ballon i 4D.

Jeg vil nok tro at det giver dig et endnu større problem. ;*)

> Og så kommer det store spørgsmål; Hvilken kraft får hastigheden til at
> stige?

Hvilket er et godt spørgsmål.

--
Mvh. Carsten Svaneborg
http://www.softwarepatenter.dk

Finn Guldmann (25-10-2004)
Kommentar
Fra : Finn Guldmann


Dato : 25-10-04 07:31

Carsten Svaneborg wrote:

>>Det giver mig to problemer.
>>For det første; Hvad så med det jeg kan opfatte i det tredie plan (3D)?
>>Og hvad forhold har det plan (2D) du beskriver i forhold til
>>udvidelsesretningen?
> Ok. I stedet for at forestille dig 2D overfladen af en ballon i 3D,
> så prøv at forestille dig en 3D overflade (universet) af en ballon i 4D.
> Jeg vil nok tro at det giver dig et endnu større problem. ;*)
>
Hvad jeg ikke lige forstår er hvorfor man skal "glemme" den ene dimension.

Men hvis jeg så opfatter det som at ("materiale-")tykkelsen af ballonnen
øges med samme hastighed som ballonnen udvides?

--
MVH Finn
To, på hinnanden følgende, undertekster i en udsendelse om
operasangere; "Jeg laver selv min mad" "og beder ofte til Gud"

Jens Olsen (25-10-2004)
Kommentar
Fra : Jens Olsen


Dato : 25-10-04 15:10

Finn Guldmann <n@a.invalid> wrote in message news:<cli6mf$16hm$1@news.cybercity.dk>...
> Hvad jeg ikke lige forstår er hvorfor man skal "glemme" den ene dimension.

Fordi den ikke er en del af universet (i dette tilfælde den 2
dimensionelle ballonanalogi).
Ballonuniversets beboer er 2-dimensionelle skabninger der laver på den
2-dimensionelle ballonoverfladen. De kan slet ikke forstå hvad du
mener med en tredie dimension, lige så lidt som du kan
se/opfatte/forstå en fjerde rumlig dimension.

> Men hvis jeg så opfatter det som at ("materiale-")tykkelsen af ballonnen
> øges med samme hastighed som ballonnen udvides?

Så har du (bevidst?) misforstået det igen. Vi taler i analogien om et
2-dimensionelt univers,- altså en 2-dimensionel flade, - så vores
ballon-analogi-univers har slet ingen tykkelse.

Jeg har en lumsk mistanke om at du hellere vil "have ret" end prøve at
forstå. Hvis det virkelig er tilfældet kan du så ikke lige sige det,
så vi ikke behøver at besvare flere af dine indlæg.

J.O.

Finn Guldmann (25-10-2004)
Kommentar
Fra : Finn Guldmann


Dato : 25-10-04 19:27

Jens Olsen wrote:

>>Hvad jeg ikke lige forstår er hvorfor man skal "glemme" den ene dimension.
> Fordi den ikke er en del af universet (i dette tilfælde den 2
> dimensionelle ballonanalogi).
>
Men den tredie dimension er en del af det virkelige liv. Hvorfor må den
så ikke tages med i beregninger, teorier, m.v.?

Bliver det for svært at forklare? Eller er det fordi teorierne så ikke
vil holde vand?

> Ballonuniversets beboer er 2-dimensionelle skabninger der laver på den
> 2-dimensionelle ballonoverfladen. De kan slet ikke forstå hvad du
> mener med en tredie dimension, lige så lidt som du kan
> se/opfatte/forstå en fjerde rumlig dimension.
>
Så er det da heldig at det ikke er det univers vi lever i. Får så ville
jeg jo komme i konflikt med de lærde, da jeg kan opfatte mig selv og
mine omgivelser i tre dimensioner.

>>Men hvis jeg så opfatter det som at ("materiale-")tykkelsen af ballonnen
>>øges med samme hastighed som ballonnen udvides?
> Så har du (bevidst?) misforstået det igen. Vi taler i analogien om et
> 2-dimensionelt univers,- altså en 2-dimensionel flade, - så vores
> ballon-analogi-univers har slet ingen tykkelse.
>
Se ovenfor.

> Jeg har en lumsk mistanke om at du hellere vil "have ret" end prøve at
> forstå. Hvis det virkelig er tilfældet kan du så ikke lige sige det,
> så vi ikke behøver at besvare flere af dine indlæg.
>
Jeg er ikke ude efter at "få ret".

Derimod er jeg ude efter at finde nogle forklaringer der kan stå mål med
det jeg kan opfatte.

Og da jeg ikke kun kan opfatte ting i nord, syd, øst og vest, men også
op og ned, kan jeg ikke få et 2D univers til at stemme overens med det
jeg kan se. (Og forstå)

Hvad jeg heller ikke kan forstå er hvorfor det skal være nødvendig at
kunne opfatte sine 3D omgivelser i 2D for at kunne regnes for at mene
det man skriver om emnet.

For mig er det helt i orden at teoretikerne bruger alternative modeller
i udviklingen af deres teorier. Men jeg finder det påkrævet at de,
såfrem de ikke selv kan, finder nogen der kan omsætte slutproduktet af
deres teorier til noget almindelige mennesker kan fatte.

Men så langt er jeg kommet at det, blandt dem mener de ved noget om det,
er blevet en slags religion der har forskellige retninger der er uenige
om hvad der passer. Bl.a. fordi deres forskellige teorier ikke harmonerer.

--
MVH Finn
To, på hinnanden følgende, undertekster i en udsendelse om
operasangere; "Jeg laver selv min mad" "og beder ofte til Gud"

Henning Makholm (25-10-2004)
Kommentar
Fra : Henning Makholm


Dato : 25-10-04 21:05

Scripsit Finn Guldmann <n@a.invalid>
> Jens Olsen wrote:

> >>Hvad jeg ikke lige forstår er hvorfor man skal "glemme" den ene dimension.
> > Fordi den ikke er en del af universet (i dette tilfælde den 2
> > dimensionelle ballonanalogi).

> Men den tredie dimension er en del af det virkelige liv. Hvorfor må
> den så ikke tages med i beregninger, teorier, m.v.?

Det skal den naturligvis også. Det er kun i den populariserende model
at man ikke tager den med. Den populariserende model er ikke teorien
selv, blot et pædagogisk virkemiddel for at forklare et enklet af
teoriens aspekter. (Efter min erfaring virker ballonmodellen endda
ikke særlig godt som pædagogisk virkemiddel, hvilket din forvirring er
et eksempel på).

> For mig er det helt i orden at teoretikerne bruger alternative
> modeller i udviklingen af deres teorier. Men jeg finder det påkrævet
> at de, såfrem de ikke selv kan, finder nogen der kan omsætte
> slutproduktet af deres teorier til noget almindelige mennesker kan
> fatte.

Ballonmodellen er ikke teorien. Det er en af de "forklaringer for
almindelige mennesker" du efterlyser. Den er ikke særlig vellykket,
men den er desværre populær.

--
Henning Makholm "Nej, hvor er vi altså heldige! Længe
leve vor Buxgører Sansibar Bastelvel!"

Finn Guldmann (25-10-2004)
Kommentar
Fra : Finn Guldmann


Dato : 25-10-04 22:06

Henning Makholm wrote:


>>Men den tredie dimension er en del af det virkelige liv. Hvorfor må
>>den så ikke tages med i beregninger, teorier, m.v.?
> Det skal den naturligvis også. Det er kun i den populariserende model
> at man ikke tager den med. Den populariserende model er ikke teorien
> selv, blot et pædagogisk virkemiddel for at forklare et enklet af
> teoriens aspekter. (Efter min erfaring virker ballonmodellen endda
> ikke særlig godt som pædagogisk virkemiddel, hvilket din forvirring er
> et eksempel på).
>
Er ved at få hold på at den ikke kan bruges. Men mangler at få fat i
hvad der så kan bruges i stedet.

--
MVH Finn
To, på hinnanden følgende, undertekster i en udsendelse om
operasangere; "Jeg laver selv min mad" "og beder ofte til Gud"

Carsten Svaneborg (25-10-2004)
Kommentar
Fra : Carsten Svaneborg


Dato : 25-10-04 17:37

Finn Guldmann wrote:
> Hvad jeg ikke lige forstår er hvorfor man skal "glemme" den ene dimension.

Fordi ellers giver analogen slet ikke nogen mening.

--
Mvh. Carsten Svaneborg
http://www.softwarepatenter.dk

Finn Guldmann (25-10-2004)
Kommentar
Fra : Finn Guldmann


Dato : 25-10-04 20:48

Carsten Svaneborg wrote:

> Finn Guldmann wrote:
>
>>Hvad jeg ikke lige forstår er hvorfor man skal "glemme" den ene dimension.
> Fordi ellers giver analogen slet ikke nogen mening.
>
Men burde den ikke det? Om ikke andet så i forhold til det fysiske
univers vi lever i?

--
MVH Finn
To, på hinnanden følgende, undertekster i en udsendelse om
operasangere; "Jeg laver selv min mad" "og beder ofte til Gud"

Carsten Svaneborg (26-10-2004)
Kommentar
Fra : Carsten Svaneborg


Dato : 26-10-04 13:12

Finn Guldmann wrote:
>> Fordi ellers giver analogen slet ikke nogen mening.
> Men burde den ikke det?
> Om ikke andet så i forhold til det fysiske univers vi lever i?

Nej. Intet forhindrer dig i at opfinde en 2D verden med 2D væsner,
for at illusterer nogle aspekter som vi ikke kan visualiserer i 3D.

Analogien viser at hvis du lever i 2D verdenen på overfladen af
en kugle (som du ikke selv kan erkende med din 2D hjerne), så vil
alle afstande vokse linært også uden at der findes noget centrum.

Det er let at tegne og relativt let at forklare, når man indser
at fra en 2D verden borgers perspektiv, findes der ikke nogen kugle.

Om man så lærer mere af Ballon analogien end at kigge på udregningen
af Robinson-walker metrikken afhænger så af om man foretrækker simple
men måske misvisende analogier ifht. en dosis differentialgeometri
og tensor matematik.

--
Mvh. Carsten Svaneborg
http://www.softwarepatenter.dk

Finn Guldmann (26-10-2004)
Kommentar
Fra : Finn Guldmann


Dato : 26-10-04 14:48

Carsten Svaneborg wrote:

>>>Fordi ellers giver analogen slet ikke nogen mening.
>>Men burde den ikke det?
>>Om ikke andet så i forhold til det fysiske univers vi lever i?
> Nej. Intet forhindrer dig i at opfinde en 2D verden med 2D væsner,
> for at illusterer nogle aspekter som vi ikke kan visualiserer i 3D.
> Analogien viser at hvis du lever i 2D verdenen på overfladen af
> en kugle (som du ikke selv kan erkende med din 2D hjerne), så vil
> alle afstande vokse linært også uden at der findes noget centrum.
> Det er let at tegne og relativt let at forklare, når man indser
> at fra en 2D verden borgers perspektiv, findes der ikke nogen kugle.
> Om man så lærer mere af Ballon analogien end at kigge på udregningen
> af Robinson-walker metrikken afhænger så af om man foretrækker simple
> men måske misvisende analogier ifht. en dosis differentialgeometri
> og tensor matematik.
>
Jeg syntes nu stadig at det må være mere relevant at forsøge at forklare
det univers som faktisk findes. I stedet for at skulle opfinde noget som
vi har svært ved at sætte os ind i. Og som derfor, imho, blot gør
forståelsesproblemet størrere.

--
MVH Finn
To, på hinnanden følgende, undertekster i en udsendelse om
operasangere; "Jeg laver selv min mad" "og beder ofte til Gud"

Carsten Svaneborg (26-10-2004)
Kommentar
Fra : Carsten Svaneborg


Dato : 26-10-04 16:14

Finn Guldmann wrote:
> Jeg syntes nu stadig at det må være mere relevant at forsøge at forklare
> det univers som faktisk findes. I stedet for at skulle opfinde noget som
> vi har svært ved at sætte os ind i. Og som derfor, imho, blot gør
> forståelsesproblemet størrere.

Det er bedrer med en simplificeret forklaring der måske giver et vagt
indtryk af sandheden, end at bede folk om at sætte sig ind i differential
geometri og tensor calculus, der for 99% af befolkningen ikke giver en
bedrer forklaring end den de kan finde i det ugentlige horoskop.

--
Mvh. Carsten Svaneborg
http://www.softwarepatenter.dk

Finn Guldmann (27-10-2004)
Kommentar
Fra : Finn Guldmann


Dato : 27-10-04 09:27

Carsten Svaneborg wrote:

>>Jeg syntes nu stadig at det må være mere relevant at forsøge at forklare
>>det univers som faktisk findes. I stedet for at skulle opfinde noget som
>>vi har svært ved at sætte os ind i. Og som derfor, imho, blot gør
>>forståelsesproblemet størrere.
> Det er bedrer med en simplificeret forklaring der måske giver et vagt
> indtryk af sandheden, end at bede folk om at sætte sig ind i differential
> geometri og tensor calculus, der for 99% af befolkningen ikke giver en
> bedrer forklaring end den de kan finde i det ugentlige horoskop.
>
Det kan kun være godt ud fra den betragtning at de der 99% ikke
interesserer sig en snus for det alligevel.

--
MVH Finn
To, på hinnanden følgende, undertekster i en udsendelse om
operasangere; "Jeg laver selv min mad" "og beder ofte til Gud"

Mogens Michaelsen (26-10-2004)
Kommentar
Fra : Mogens Michaelsen


Dato : 26-10-04 16:59

Finn Guldmann skrev:

> Carsten Svaneborg wrote:
>
>>>>Fordi ellers giver analogen slet ikke nogen mening.
>>>Men burde den ikke det?
>>>Om ikke andet så i forhold til det fysiske univers vi lever i?
>> Nej. Intet forhindrer dig i at opfinde en 2D verden med 2D
>> væsner,
>> for at illusterer nogle aspekter som vi ikke kan visualiserer
>> i 3D.
>> Analogien viser at hvis du lever i 2D verdenen på overfladen
>> af
>> en kugle (som du ikke selv kan erkende med din 2D hjerne), så
>> vil
>> alle afstande vokse linært også uden at der findes noget
>> centrum.
>> Det er let at tegne og relativt let at forklare, når man
>> indser
>> at fra en 2D verden borgers perspektiv, findes der ikke nogen
>> kugle.
>> Om man så lærer mere af Ballon analogien end at kigge på
>> udregningen
>> af Robinson-walker metrikken afhænger så af om man foretrækker
>> simple
>> men måske misvisende analogier ifht. en dosis
>> differentialgeometri
>> og tensor matematik.
>>
> Jeg syntes nu stadig at det må være mere relevant at forsøge at
> forklare det univers som faktisk findes. I stedet for at skulle
> opfinde noget som vi har svært ved at sætte os ind i. Og som
> derfor, imho, blot gør forståelsesproblemet størrere.
>
> --
> MVH Finn
> To, på hinnanden følgende, undertekster i en udsendelse om
> operasangere; "Jeg laver selv min mad" "og beder ofte til Gud"
>

Jeg har fornylig læst en utroligt velskreven bog om emnet,
nemlig:
Steen Hannestad: "Universet - fra superstrenge til stjerner", fra
Aarhus Universitetsforlag.
Den er udgivet i 2003 og har derfor det nyeste med, er ret præcis
uden at indeholde en masse
matematik (som jeg - heller ikke - har forudsætninger for at
forstå). Jeg lånte den på folkebiblioteket.

MVH
Mogens Michaelsen



Finn Guldmann (27-10-2004)
Kommentar
Fra : Finn Guldmann


Dato : 27-10-04 09:59

Mogens Michaelsen wrote:

> Jeg har fornylig læst en utroligt velskreven bog om emnet,
> nemlig:
> Steen Hannestad: "Universet - fra superstrenge til stjerner", fra
> Aarhus Universitetsforlag.
> Den er udgivet i 2003 og har derfor det nyeste med, er ret præcis
> uden at indeholde en masse
> matematik (som jeg - heller ikke - har forudsætninger for at
> forstå). Jeg lånte den på folkebiblioteket.
>
Fandt den ikke på det lokale bibliotek.
Men måske man skulle prøve en boghandel i stedet. Er den 200,- kr værd?


--
MVH Finn
To, på hinnanden følgende, undertekster i en udsendelse om
operasangere; "Jeg laver selv min mad" "og beder ofte til Gud"

Mogens Michaelsen (27-10-2004)
Kommentar
Fra : Mogens Michaelsen


Dato : 27-10-04 18:47

Finn Guldmann skrev:

> Mogens Michaelsen wrote:
>
>> Jeg har fornylig læst en utroligt velskreven bog om emnet,
>> nemlig:
>> Steen Hannestad: "Universet - fra superstrenge til stjerner",
>> fra Aarhus Universitetsforlag.
>> Den er udgivet i 2003 og har derfor det nyeste med, er ret
>> præcis uden at indeholde en masse
>> matematik (som jeg - heller ikke - har forudsætninger for at
>> forstå). Jeg lånte den på folkebiblioteket.
>>
> Fandt den ikke på det lokale bibliotek.
> Men måske man skulle prøve en boghandel i stedet. Er den 200,-
> kr værd?
>
>
> --
> MVH Finn
> To, på hinnanden følgende, undertekster i en udsendelse om
> operasangere; "Jeg laver selv min mad" "og beder ofte til Gud"
>

Du kan altid reservere bogen på folkebiblioteket (også online).
Men ellers vil jeg helt sikkert mene, at den er et par hunde
værd.
Bogen er også rimeligt godt egnet til opslag, og indeholder bl.a.
ordforklaringer.


MVH
Mogens Michaelsen

P.S. Jeg laver selv min mad - og har en hund (endnu)



Finn Guldmann (27-10-2004)
Kommentar
Fra : Finn Guldmann


Dato : 27-10-04 22:35

Mogens Michaelsen wrote:

>>Fandt den ikke på det lokale bibliotek.
>>Men måske man skulle prøve en boghandel i stedet. Er den 200,-
>>kr værd?
> Du kan altid reservere bogen på folkebiblioteket (også online).
> Men ellers vil jeg helt sikkert mene, at den er et par hunde
> værd.
> Bogen er også rimeligt godt egnet til opslag, og indeholder bl.a.
> ordforklaringer.
>
Ok, jeg vil gå på jagt efter den.

> P.S. Jeg laver selv min mad - og har en hund (endnu)
>
Jeg har så meget pli at jeg undlader at lave mad andre skal spise.

--
MVH Finn
To, på hinnanden følgende, undertekster i en udsendelse om
operasangere; "Jeg laver selv min mad" "og beder ofte til Gud"

Pettersen; Roald (27-10-2004)
Kommentar
Fra : Pettersen; Roald


Dato : 27-10-04 20:38

Jens Olsen:
....
> Altså universet er ballonens overflade, og KUN overfladen.
> Altså et todimensionelt univers.
> Ballonens indre er slet ikke en del af universet
> eller i det hele taget relevant i sammenhængen.
>
> Lige gyldigt hvad punkt på ballonens overflade du står på, når
> ballonens pustes op, så vil du se alle andre punkter bevæge sig væk
> fra dig. Du kan ikke sige, at et bestemt punkt på ballonens overflade
> er centrum for ekspansionen.
>
> Hjælper det dig til at forstår det.

Kan jeg sige, at når jeg i modellen betragter universet,
så _ser_ jeg ikke universet som hele ballonen,
men nærmere som en mindre ballon med ca. den halve diameter,
og den tangerer det punkt, som jeg selv befinder mig i?

En ballon har jo et centrum, selvom det ikke ligger på ballonen.
Den synlige ballon, som er det univers jeg ser lige nu,
indeholder så stadig det centrum hvorfra det hele begyndte.
Det er det punkt, der ligger modsat mig på den lille ballon,
og jeg ser det i baggrunden i alle retninger.

Hvis jeg prøver at nærme mig dette centrum, så kan jeg ikke,
selv hvis tiden stod stille. Jeg vil istedet flytte mig rundt det,
med den følge, at jeg nu ser en tilsvarende lille ballong
bare fra et andet punkt på den store ballon.

Henning Makholm (27-10-2004)
Kommentar
Fra : Henning Makholm


Dato : 27-10-04 21:53

Scripsit roald@nomail.invalid (Pettersen; Roald)

> Kan jeg sige, at når jeg i modellen betragter universet,
> så _ser_ jeg ikke universet som hele ballonen,
> men nærmere som en mindre ballon med ca. den halve diameter,
> og den tangerer det punkt, som jeg selv befinder mig i?

Øh, nej, det giver vist ikke meget mening.

> En ballon har jo et centrum, selvom det ikke ligger på ballonen.

Jo, men sådan et centrum findes ikke i de faktiske universmodeller som
ballonen skal illustrere et enkelt aspekt af. (Det er hovedgrunden til
at jeg mener ballonmodellen er mere vild- en vejledende).

--
Henning Makholm # good fish ...
# goodfish, goodfish ...
# good-good FISH! #

Sven Nielsen (23-10-2004)
Kommentar
Fra : Sven Nielsen


Dato : 23-10-04 00:37

In article <clblun$10mi$1@news.cybercity.dk>, n@a.invalid says...

> De skriver "en plet, der er mindre end et atom". Det tolker jeg som et
> bestemt sted. Et sted der har et geografisk forhold til det univers vi
> har i dag.

Det er så en forkert tolkning fra din side. Det er HELE Universet, der
ifølge standardmodellen på et tidspunkt har været meget kompakt -
mindre end et atom. Det Univers er siden blevet blæst op til den
størrelse, det har i dag.

Du vil aldrig kunne forstå modellen, før du opgiver tanken om, at big
bang var en eksplosion af et uratom ud i et univers, der allerede
eksisterede. Sådan er modellen ikke.

Mvh Sven.

Carsten Svaneborg (22-10-2004)
Kommentar
Fra : Carsten Svaneborg


Dato : 22-10-04 19:10

Finn Guldmann wrote:
> Men da teorien hævder at [..]

Nej. Den beskrivelse du har læst af teorien hævder,
og derfor hævder du at ...

Dette er blot forkert, og har ikke noget med Big Bang teorien at gøre.
Desuden udtaler Big Bang teorien sig ikke om hvad der skete ved t=0 og
blot kort tid efter Big Bang øjeblikket.

--
Mvh. Carsten Svaneborg
http://www.softwarepatenter.dk

Finn Guldmann (22-10-2004)
Kommentar
Fra : Finn Guldmann


Dato : 22-10-04 21:49

Carsten Svaneborg wrote:

> Nej. Den beskrivelse du har læst af teorien hævder,
> og derfor hævder du at ...
>
Jeg hævder ikke noget. Jeg forsøger at finde en forklaring på noget der
irriterer min logiske sans.

> Dette er blot forkert, og har ikke noget med Big Bang teorien at gøre.
> Desuden udtaler Big Bang teorien sig ikke om hvad der skete ved t=0 og
> blot kort tid efter Big Bang øjeblikket.
>
Det hævder de ellers på DR's videnskabsredaktion.

--
MVH Finn
To, på hinnanden følgende, undertekster i en udsendelse om
operasangere; "Jeg laver selv min mad" "og beder ofte til Gud"

Per Rønne (23-10-2004)
Kommentar
Fra : Per Rønne


Dato : 23-10-04 18:46

Finn Guldmann <n@a.invalid> wrote:

> Carsten Svaneborg wrote:

> > Dette er blot forkert, og har ikke noget med Big Bang teorien at gøre.
> > Desuden udtaler Big Bang teorien sig ikke om hvad der skete ved t=0 og
> > blot kort tid efter Big Bang øjeblikket.

> Det hævder de ellers på DR's videnskabsredaktion.

Fordi DRs videnskabsredaktion forsøger at give en populær fremstilling
af sagen.
--
Per Erik Rønne

Finn Guldmann (25-10-2004)
Kommentar
Fra : Finn Guldmann


Dato : 25-10-04 07:04

Per Rønne wrote:

>>>Dette er blot forkert, og har ikke noget med Big Bang teorien at gøre.
>>>Desuden udtaler Big Bang teorien sig ikke om hvad der skete ved t=0 og
>>>blot kort tid efter Big Bang øjeblikket.
>>Det hævder de ellers på DR's videnskabsredaktion.
> Fordi DRs videnskabsredaktion forsøger at give en populær fremstilling
> af sagen.
>
Måske for at sådan nogen som mig har en chance for at forstå (noget af) det.

Er der nogen der kan give et link til BB-teorien, som den oprindelig
blev fremsat? (Så jeg kan læse den ufortolket)

--
MVH Finn
To, på hinnanden følgende, undertekster i en udsendelse om
operasangere; "Jeg laver selv min mad" "og beder ofte til Gud"

Per Rønne (25-10-2004)
Kommentar
Fra : Per Rønne


Dato : 25-10-04 07:52

Finn Guldmann <n@a.invalid> wrote:

> Per Rønne wrote:
>
> >>>Dette er blot forkert, og har ikke noget med Big Bang teorien at gøre.
> >>>Desuden udtaler Big Bang teorien sig ikke om hvad der skete ved t=0 og
> >>>blot kort tid efter Big Bang øjeblikket.
> >>Det hævder de ellers på DR's videnskabsredaktion.
> > Fordi DRs videnskabsredaktion forsøger at give en populær fremstilling
> > af sagen.
> >
> Måske for at sådan nogen som mig har en chance for at forstå (noget af) det.
>
> Er der nogen der kan give et link til BB-teorien, som den oprindelig
> blev fremsat? (Så jeg kan læse den ufortolket)

Tjae, jeg har en gammel bog af Dr John Gribbin, »Genesis«, isbn
0-19-283035-X. Fra 1982, Oxford University Press. Før jeg købte den
lånte jeg den fra et folkebibliotek.

Når det drejer sig om de helt store linier, tvivler jeg på at alderen på
22 år er så afgørende, og der er tale om en populær fremstilling.

http://bookshop.blackwell.co.uk/bobuk/scripts/display_product_info.jsp?B
V_SessionID=@@@@1449584310.1098686580@@@@&BV_EngineID=ccciadcmljkjgffcef
eceegdfigdffo.0&productid=019283035X
--
Per Erik Rønne

Carsten Svaneborg (25-10-2004)
Kommentar
Fra : Carsten Svaneborg


Dato : 25-10-04 17:42

Finn Guldmann wrote:
> Er der nogen der kan give et link til BB-teorien, som den oprindelig
> blev fremsat? (Så jeg kan læse den ufortolket)

http://xxx.lanl.gov/find

Klik i "Search all", og alle år.

Søg efter "Big Bang review" i abstract.

Du får 110 artikler.

--
Mvh. Carsten Svaneborg
http://www.softwarepatenter.dk

Stig Johansen (22-10-2004)
Kommentar
Fra : Stig Johansen


Dato : 22-10-04 19:03

Finn Guldmann wrote:

> Det har længe været mig til undren at man snakker om at man kan se så og
> så mange mia. år tilbage i tiden.

Det samme her.
En af de ting, der også undrer mig er, at man betragter det ud fra en lineær
model.
Lyset bliver jo afbøjet af gravitationsfelter, og man kan ligeså godt
forestille sig, at vi i nogle tilfælde ser vores egen galakses 'ekko'.

--
Med venlig hilsen
Stig Johansen

Carsten Svaneborg (22-10-2004)
Kommentar
Fra : Carsten Svaneborg


Dato : 22-10-04 20:50

Stig Johansen wrote:
> En af de ting, der også undrer mig er, at man betragter det ud fra en
> lineær model.
Nu er almen relativitetsteori særdeles ikke-lineær.

> Lyset bliver jo afbøjet af gravitationsfelter, og man kan ligeså godt
> forestille sig, at vi i nogle tilfælde ser vores egen galakses 'ekko'.

Hvilket ville rejse spørgsmålet i hvilken retning skulle vi kunne se
vores egen galakse? Da alle retninger jo er lige gode.

Anyways, så er universets topologi ikke et utrivielt spørgsmål. Dvs.
dvs. at dele af den kosmiske baggrundstråling der fra vores synspunkt
syntes at komme fra forskellige områder af stjernehimlen i virkeligheden
pga. en ikke triviel topologi kunne komme fra samme sted i rummet.

Sådanne effekter ville man kunne observere ved at udregne
korrelationsfunktioner for den kosmiske baggrundstråling.

Se http://astro.uchicago.edu/home/web/olinto/courses/A18200/nbower.htm
og referencer.

--
Mvh. Carsten Svaneborg
http://www.softwarepatenter.dk

Stig Johansen (23-10-2004)
Kommentar
Fra : Stig Johansen


Dato : 23-10-04 06:56

Carsten Svaneborg wrote:

> Hvilket ville rejse spørgsmålet i hvilken retning skulle vi kunne se
> vores egen galakse? Da alle retninger jo er lige gode.

Ja, eller det jeg har lidt svært ved at forstå er, hvordan ved vi at det
*ikke* er vores egen galaxe, vi observerer?

Jeg tager nok grueligt fejl, men jeg forestiller mig et meget simplificeret
tilfælde.

Hvis vi antager, at vores galaxe udsendte lys for eks. 10 mia år siden.
Dette lys har undervejs mødt så mange gravitationsfelter, så den samlede
afbøjning er 180 grader. Endvidere er den samlede rejsetid også 10 mia år.

Kan man så ikke forestille sig, at vi i virkeligheden ser os selv for 10 mia
år siden, og ikke en galaxe, der ligger 10 mia lysår væk?

--
Med venlig hilsen
Stig Johansen

Carsten Svaneborg (24-10-2004)
Kommentar
Fra : Carsten Svaneborg


Dato : 24-10-04 01:29

Stig Johansen wrote:
> Hvis vi antager, at vores galaxe udsendte lys for eks. 10 mia år siden.
> Dette lys har undervejs mødt så mange gravitationsfelter, så den samlede
> afbøjning er 180 grader. Endvidere er den samlede rejsetid også 10 mia år.

Det kan jeg godt forestille mig. Men jeg kan ikke forestille mig at
under nogle omstændigheder kan få et SKARPT billed af galaksen, som det
man ser på Hubble deep field. Selv i idelle situationer hvor man ser
Einstein ringe er disse uskarpe.

Tænk på det som et stort ping-pong spil, lyset udsendes af galaksen i
alle retninger, en god del af disse fotoner skal igennem et ping-pong
spil af tyngdefelter og så ende med at blive sendt tilbage i samme
retning som de kom fra. Bare en lille ændring af vinklen lyset udsendes
i, vil betyde at lyset følger en helt anden bane igennem universet.
Det betyder at alt udtværes.

--
Mvh. Carsten Svaneborg
http://www.softwarepatenter.dk

Stig Johansen (24-10-2004)
Kommentar
Fra : Stig Johansen


Dato : 24-10-04 07:10

Carsten Svaneborg wrote:

> Stig Johansen wrote:
>> Hvis vi antager, at vores galaxe udsendte lys for eks. 10 mia år siden.
>> Dette lys har undervejs mødt så mange gravitationsfelter, så den samlede
>> afbøjning er 180 grader. Endvidere er den samlede rejsetid også 10 mia
>> år.
>
> Det kan jeg godt forestille mig. Men jeg kan ikke forestille mig at
> under nogle omstændigheder kan få et SKARPT billed af galaksen, som det
> man ser på Hubble deep field. Selv i idelle situationer hvor man ser
> Einstein ringe er disse uskarpe.
>
> Tænk på det som et stort ping-pong spil, lyset udsendes af galaksen i
> alle retninger, en god del af disse fotoner skal igennem et ping-pong
> spil af tyngdefelter og så ende med at blive sendt tilbage i samme
> retning som de kom fra. Bare en lille ændring af vinklen lyset udsendes
> i, vil betyde at lyset følger en helt anden bane igennem universet.
> Det betyder at alt udtværes.

Ja - den tror jeg er feset ind på lystavlen.
Tak.

--
Med venlig hilsen
Stig Johansen

Preben Riis Sørensen (24-10-2004)
Kommentar
Fra : Preben Riis Sørensen


Dato : 24-10-04 15:48


"Carsten Svaneborg" <zqex@sted.i.tyskland.de> skrev Bare en lille ændring
af vinklen lyset udsendes
> i, vil betyde at lyset følger en helt anden bane igennem universet.
> Det betyder at alt udtværes.

Og det er netop ikke det man ser. Altså er der noget der halter med teorien.
Rent faktisk ligger opdagelsen af gravitationslinser ikke langt tilbage i
tiden, og det er vist ret få der kendes. De skulle være overalt i en stor
udtværing, som du skriver.
--
M.V.H.
Preben Riis Sørensen
preben@esenet.dk




Finn Guldmann (22-10-2004)
Kommentar
Fra : Finn Guldmann


Dato : 22-10-04 21:47

Stig Johansen wrote:
>>Det har længe været mig til undren at man snakker om at man kan se så og
>>så mange mia. år tilbage i tiden.
> Det samme her.
> En af de ting, der også undrer mig er, at man betragter det ud fra en lineær
> model.
> Lyset bliver jo afbøjet af gravitationsfelter, og man kan ligeså godt
> forestille sig, at vi i nogle tilfælde ser vores egen galakses 'ekko'.
>
Warp? Det var måske en forklaring på hvor "The Missing Light" bliver af?

--
MVH Finn
To, på hinnanden følgende, undertekster i en udsendelse om
operasangere; "Jeg laver selv min mad" "og beder ofte til Gud"

Lars Kristensen (09-11-2004)
Kommentar
Fra : Lars Kristensen


Dato : 09-11-04 18:08

Hej Finn,

det jeg nu vil fortælle dig, vil få alle de andre her i gruppen til at
rejse sig i stolen og sige »Åh, nej - ikke igen!«

Problemer er, at jeg hele tiden siger, at universet ikke ekspanderer
og at der ikke har eksisteret et Big Bang. At rødforskydningen af
lyset fra fjerne himmelobjekter kan være opstået ved andre faktorer
end en bevægelse bort fra os (hinanden - universets ekspansion).

Det andet problem er, at jeg ikke kan fremkomme med beviser for min
påstand og så vil de faneme ikke tale mere om det. For vil det jeg
siger være rigtigt, er hele det forrige århundredes videnskabelige
verdensbillede være forkert og det kan det jo ikke være, vel?

Der er ingen der bryder sig om, at der stilles spørgsmålstegn ved
deres opfattelse af universet og dets mulige opståen/skabelse og slet
ikke de etablere lærde. Sådan har det altid været og sådan er det også
i dag. Den der ikke er lærd, er dum, for denne ved ikke hvad der er
rigtigt og hvad der er forkert.

Desværre er det tit og ofte sådan, at de lærde gør sig blinde for
andre muligheder og derfor kommer man til at føre sig selv og alle
andre ud på en vej som man har svært ved at vende om fra, fordi man nu
engang er kørt ud ad den.

Med venlig hilsen

Lars Kristensen

Carsten Svaneborg (09-11-2004)
Kommentar
Fra : Carsten Svaneborg


Dato : 09-11-04 19:28

Lars Kristensen wrote:
> Problemer er, at jeg hele tiden siger, at universet ikke ekspanderer
> og at der ikke har eksisteret et Big Bang.

Hvad du ikke syntes er at forstå er at naturvidenskab handler
om at beskrive naturen ved at lave eksperimenter. Teorier kan
være er interessante for sin egen skyld, men før eller siden
ender de i skrældespanden hvis de ikke kan eftervises med
eksperimenter.

Sålænge du ikke har formuleret en teori, og kommer med eksperimentel
evidens for denne, er det spild af tid at diskuterer om den er rigtig
eller ej. Det er blot snak.

Forskellen på fysik og filosofi er at fysikere har en skrældespand
til teorier der ikke virker, nemmeligt dem der ikke passer med
eksperimenter, mens filosoffer kan have endeløse diskussioner fordi
de ikke har en skrældespand.

> For vil det jeg siger være rigtigt, er hele det forrige
> århundredes videnskabelige verdensbillede være forkert og
> det kan det jo ikke være, vel?

Det kan meget vel vise sig at være tilfældet, og det ville være
særdeles interessant hvis det skete. Men så følger det ikke af
kaffesnak i dk.videnskab, eller af at du følger dig forsmået fordi
du hverken har teorie eller eksperiment.

Derfor SKRÆLDESPAND

--
Mvh. Carsten Svaneborg
http://www.softwarepatenter.dk

Per Abrahamsen (10-11-2004)
Kommentar
Fra : Per Abrahamsen


Dato : 10-11-04 10:45

lars@hjertensfryd.dk (Lars Kristensen) writes:

> Det andet problem er, at jeg ikke kan fremkomme med beviser for min
> påstand og så vil de faneme ikke tale mere om det.

Det er fordi det så ikke er videnskab længere. En teori kan sagtens
være rigtig og uvidenskabelig, ligesom den kan være forkert og
videnskabelig. God videnskab handler ikke om rigtigt eller forkert,
men om hvad man kan fremlægge empirisk evidens for.

Du bør gå til livssynsgrupperne hvis du vil diskutere rigtigt og
forkert uden empirisk belæg.

> For vil det jeg siger være rigtigt, er hele det forrige århundredes
> videnskabelige verdensbillede være forkert og det kan det jo ikke
> være, vel?

Det halve af forrige århundrede, Big Bang teorien er fra 50'erne. Og
vi laver konstant om i vores kosmologiske modeller, når ny empirisk
evidens påkræver det. Det har været store ændringer bare i dette
årtusinde, vi er gået fra et endeligt (krumt) til et uendeligt (fladt)
univers som bedste bud.

Morten Guldager (10-11-2004)
Kommentar
Fra : Morten Guldager


Dato : 10-11-04 21:40

2004-11-10 Per Abrahamsen wrote
> God videnskab handler ikke om rigtigt eller forkert,
> men om hvad man kan fremlægge empirisk evidens for.

Kan du med ord som ikke kræver en fremmedordbog
forklare hvad "empirisk evidens" betyder?

> Det halve af forrige århundrede, Big Bang teorien er fra 50'erne. Og
> vi laver konstant om i vores kosmologiske modeller, når ny empirisk
> evidens påkræver det. Det har været store ændringer bare i dette
> årtusinde, vi er gået fra et endeligt (krumt) til et uendeligt (fladt)
> univers som bedste bud.

Betyder et "uendeligt (fladt) univers" en uendelig tynd skive?

Eller taler vi bare om en i kosmologisk forstand flad ting, altså
stadig mere end en håndfuld lysår tyk?

Eller noget helt tredje?


/Morten

Henning Makholm (10-11-2004)
Kommentar
Fra : Henning Makholm


Dato : 10-11-04 22:31

Scripsit Morten Guldager <spamtrap@mogul.dk>
> 2004-11-10 Per Abrahamsen wrote

> > God videnskab handler ikke om rigtigt eller forkert,
> > men om hvad man kan fremlægge empirisk evidens for.

> Kan du med ord som ikke kræver en fremmedordbog
> forklare hvad "empirisk evidens" betyder?

Observationer der stemmer med teorien, men ikke på de alternative
teorier man på et givet tidspunkt finder det relevant at overveje.

> > Det har været store ændringer bare i dette årtusinde, vi er gået
> > fra et endeligt (krumt) til et uendeligt (fladt) univers som
> > bedste bud.

> Betyder et "uendeligt (fladt) univers" en uendelig tynd skive?

Nej, det betyder et univers uden intern krumning - altså et hvor
geometrien i stor skala er euklidisk: Afstanden mellem krydsende linjer
er direkte propoprtional med hvor langt fra krydsningen man måler,
parallelle linjer har samme afstand til hinanden uanset hvor langt ud
af dem man går, og så videre.

--
Henning Makholm "In my opinion, this child don't
need to have his head shrunk at all."

Søg
Reklame
Statistik
Spørgsmål : 177501
Tips : 31968
Nyheder : 719565
Indlæg : 6408527
Brugere : 218887

Månedens bedste
Årets bedste
Sidste års bedste